Вы находитесь на странице: 1из 117

CRIMINAL LAW

AND PROCEDURE
Case Summaries

This document has been prepared by the author according to the NCA’s official examination rules
(http://www.flsc.ca/en/nca/nca-resources/exam-schedules-and-policies/#Exam rules): “Starting with the
NCA examinations scheduled for August 2011, candidates are permitted to use any reference materials
during the examination, including personal hand-written or typed notes.”

This collection of case summaries adheres to the November 2013 FLSC/NCA syllabus, as accessed on
5/6/2014
The Sources of Criminal Law | <<Criminal Law and Procedure> 2
TABLE OF CONTENTS
Criminal Law and Procedure ............................................................................................................................................. 1
Table of Contents ................................................................................................................................................................ 3
General Overview and Preliminary Matters .................................................................................................................... 9
The Sources of Criminal Law ......................................................................................................................................... 9
Frey v. Fedoruk, [1950] S.C.R. 517 ...................................................................................................................... 9
R. v. Jobidon, [1991] 2 S.C.R. 714 ....................................................................................................................... 9
The Power to Create Criminal Offences and Rules of Criminal Procedure ......................................................... 10
Constitutional Division of Powers Introduced ..................................................................................................... 10
R. v. Malmo-Levine, 2003, SCC 74 ................................................................................................................... 10
The Canadian Charter of Rights and Freedoms .................................................................................................. 12
R. v. Heywood, [1994] 3 S.C.R. 761.................................................................................................................. 13
R. v. Oakes, [1986] 1 S.C.R. 103....................................................................................................................... 13
R. v. Labaye, [2005] 3 S.C.R. 728 ..................................................................................................................... 13
The Classification of Offences .................................................................................................................................... 14
Summary Offence .................................................................................................................................................... 14
Indictable Offence .................................................................................................................................................... 14
Interpreting Criminal Provisions ................................................................................................................................ 15
Definitions ................................................................................................................................................................. 15
Strict Construction ................................................................................................................................................... 16
R. v. Pare, [1987] 2 S.C.R. 17 ............................................................................................................................ 16
Purposive Interpretation ......................................................................................................................................... 16
French/English ......................................................................................................................................................... 16
R. v. Mac, [2002] 1 S.C.R. 856 .......................................................................................................................... 16
R. v. Collins, [1987] 1 S.C.R. 265 ...................................................................................................................... 17
R. v. D.(J.), 2002 CanLII 16805 (ON CA) .......................................................................................................... 18
The Charter ............................................................................................................................................................... 18
Canadian Foundation for Children, Youth & the Law v. Canada (A.G.), [2004] 1 S.C.R. 76 .................... 18
The Elements of a Criminal or Regulatory Offence..................................................................................................... 19
The Actus Reus ............................................................................................................................................................. 20
Acts and Statutory Conditions ............................................................................................................................... 21
R v J.(D.), [2002] O.J. No. 4916 (Ont. C.A.) ....................................................................................................... 22
R. v. Gunning [2005] 1 S.C.R. 627 .................................................................................................................... 22
Acts Must Be “Voluntary” or “Willed” .................................................................................................................... 23
The “Act” of Possession .......................................................................................................................................... 24

3 Criminal Law and Procedure | FLSC/NCA


R. v. York (2005), 193 C.C.C. (3d) 331 (B.C.C.A.)............................................................................................ 24
R. v. Marshall, [1969] 3 C.C.C. 149 (Alta. C.A.) ............................................................................................... 24
R. v. Terrence, [1983] 1 S.C.R. 357 .................................................................................................................. 25
R. v. Morelli, 2010 SCC 8, [2010] 1 S.C.R. 253 .............................................................................................. 25
Consent as an Element of the Actus Reus ........................................................................................................... 26
R. v. Jobidon, [1991] 2 S.C.R. 714 .................................................................................................................... 26
R. v. J.A., 2011 SCC 28 ....................................................................................................................................... 26
R. v. Mabior, 2012 SCC 47 ................................................................................................................................. 28
Causation .................................................................................................................................................................. 28
R. v. Smithers, [1978] 1 S.C.R. 506 .................................................................................................................. 29
R v Harbottle 1993 SCC ...................................................................................................................................... 29
R. v. Nette, [2001] 3 S.C.R. 488 ........................................................................................................................ 29
R. v. Williams [2003] 2 S.C.R. 134 ................................................................................................................... 30
R. v. Maybin 2012 SCC 24 ................................................................................................................................. 30
R. v. Reid, 2003 NSCA 104 ................................................................................................................................ 30
Omissions .................................................................................................................................................................. 31
R. v. Moore, [1979] 1 S.C.R. 195 ...................................................................................................................... 31
R. v. Peterson, [2005] O.J. No. 4450 (Ont. C.A.) ............................................................................................. 32
R v. Browne (1997), 116 C.C.C. (3d) 183 (Ont. C.A.) ...................................................................................... 32
Subjective Mens Rea ................................................................................................................................................... 33
Intention, and Ulterior Mens Rea .......................................................................................................................... 35
R. v. Vandergraff, [1994] M.J. No. 503 (Man. C.A.) ......................................................................................... 36
R. v. Murray, [2000] O.J. No. 2182 (Ont. S.C.J.) .............................................................................................. 36
R. v. J.S.R., 2008 O.N.C.A. 544 .......................................................................................................................... 36
R. v. Roks, 2011 ONCA 526 ............................................................................................................................... 37
Subjective Mens Rea with Objective Features .................................................................................................... 39
R. v. Theroux, [1993] 2 S.C.R. 5......................................................................................................................... 39
R. v. Chase, [1987] 2 S.C.R. 293 ....................................................................................................................... 39
Knowledge ................................................................................................................................................................ 40
R. v. Ewanchuk, [1999] 1 S.C.R. 330 ............................................................................................................... 40
R. v. Levigne, [2010] 2 S.C.R. 3 ......................................................................................................................... 40
R. v. Beaver, [1957] S.C.R. 531 ......................................................................................................................... 41
R. v. ADH, 2013 SCC 28 ..................................................................................................................................... 41
Wilful Blindness........................................................................................................................................................ 41
R. v. Currie (1975), 24 C.C.C. (2d) 292 (Ont.C.A.) ........................................................................................... 42
R. v. Vinokurov, 2001 ABCA 113 (Alta C.A.) .................................................................................................... 42

The Sources of Criminal Law | Table of Contents 4


R. v. Briscoe, 2010 SCC 13 ................................................................................................................................ 42
Recklessness ............................................................................................................................................................ 42
R. v. Theroux, [1993] 2 S.C.R. 5......................................................................................................................... 43
R. v. Buzzanga and Durocher (1979), 25 O.R. (2d) 705 (Ont. C.A.) ............................................................. 43
Objective Mens Rea and True Crimes ....................................................................................................................... 43
R. v. Martineau, [1990] 2 S.C.R. 633................................................................................................................ 44
R. v. Creighton, [1993] 3 S.C.R. 3 ..................................................................................................................... 44
R. v. Beatty, 2008 SCC 5 .................................................................................................................................... 45
Regulatory Offences .................................................................................................................................................... 45
R. v. Sault Ste. Marie, [1978] 2 S.C.R. 1299 ................................................................................................... 45
Reference re Section 94(2) of the Motor Vehicle Act (B.C.), [1985] 2 S.C.R. 486 .................................... 46
R. v. Wholesale Travel Inc. [1991] 3 S.C.R. 154 ............................................................................................. 46
R. v. Raham, 2010 ONCA 206 (C.A.) ................................................................................................................ 46
Lévis (City) v. Tétreault; Lévis (City) v. 2629-4470 Québec inc..................................................................... 47
Extensions of Criminal Liability ...................................................................................................................................... 47
Aiding and Abetting ..................................................................................................................................................... 47
R. v. Dunlop and Sylvester, [1979] 2 S.C.R. 881 ............................................................................................ 47
R. v. Logan, [1990] 2 S.C.R. 731 ....................................................................................................................... 48
R. v. Briscoe, 2010 SCC 13 ................................................................................................................................ 48
R. v. Thatcher, [1987] 1 S.C.R. 652 .................................................................................................................. 49
R. v. JF, 2013 SCC 12 .......................................................................................................................................... 49
R. v. Gauthier, 2013 SCC 32 .............................................................................................................................. 50
Counselling .................................................................................................................................................................... 51
R. v. Hamilton, [2005] 2 S.C.R. 432 ................................................................................................................. 51
Attempts ........................................................................................................................................................................ 52
R. v. Ancio, [1984] 1 S.C.R. 225 ........................................................................................................................ 52
R. v. Deutsch, [1986] 2 S.C.R. 2 ........................................................................................................................ 53
R. v. Déry, 2006 SCC 53 ..................................................................................................................................... 54
Corporate and Association Liability........................................................................................................................... 54
Select Criminal Defences ................................................................................................................................................ 54
Involuntariness ............................................................................................................................................................. 55
Mental Disorder ............................................................................................................................................................ 55
R. v. Cooper, [1980] 1 S.C.R. 1149 ................................................................................................................... 57
R. v. Kjeldsen, [1981] 2 S.C.R. 617 .................................................................................................................. 58
R. v. Oommen, [1994] 2 S.C.R. 507 ................................................................................................................. 58
Automatism and Involuntary Acts “Negativing” the Actus Reus .......................................................................... 58

5 Criminal Law and Procedure | FLSC/NCA


R. v. Swaby, [2001] O.J. No. 2390 (Ont.C.A.) ................................................................................................... 60
R. v. Parks, [1992] 2 S.C.R. 871........................................................................................................................ 61
R. v. Stone, [1999] 2 S.C.R. 290 ....................................................................................................................... 61
R. v. Fontaine, [2004] 1 S.C.R. 702 .................................................................................................................. 63
R. v. Luedecke, 2008 ONCA 716 ....................................................................................................................... 64
Intoxication.................................................................................................................................................................... 65
Simple Intoxication ...................................................................................................................................................... 66
The Queen v. George, [1960] S.C.R. 871 ......................................................................................................... 66
R. v. Robinson, [1996] 1 S.C.R. 683 ................................................................................................................. 67
Extreme Intoxication .................................................................................................................................................... 67
R. v. Daviault, [1994] 3 S.C.R. 63 ..................................................................................................................... 67
R. v. Bouchard-Lebrun, 2011 SCC 58 ............................................................................................................... 68
Defence of the Person ................................................................................................................................................. 68
R. v. Lavallee, [1990] 1 S.C.R. 852 ................................................................................................................... 70
Necessity ....................................................................................................................................................................... 70
Perka v. The Queen. [1984] 2 S.C.R. 232 ........................................................................................................ 71
R. v. Latimer, [2001] 1 S.C.R. 3 ......................................................................................................................... 72
Duress ............................................................................................................................................................................ 72
R .v. Ryan, 2013 SCC 3 ....................................................................................................................................... 73
Provocation ................................................................................................................................................................... 75
R. v. Tran, [2010] 3 S.C.R. 350 .......................................................................................................................... 76
Entrapment ................................................................................................................................................................... 76
R. v. Mack, [1988] 2 S.C.R. 903 ........................................................................................................................ 76
R. v. Barnes, [1991] 1 S.C.R. 449 ..................................................................................................................... 77
Ignorance of the Law ................................................................................................................................................... 78
Lilly v. The Queen, [1983] 1 SCR 794 .............................................................................................................. 78
R. v. Jones, [1991] 3 SCR 110 ........................................................................................................................... 78
Lévis (City) v. Tétreault; Lévis (City) v. 2629-4470 Québec inc..................................................................... 79
The Adversarial Proceeding ............................................................................................................................................ 79
The Adversarial Process .............................................................................................................................................. 79
The Presumption of Innocence and the Ultimate Standard of Proof .............................................................. 79
R. v. Lifchus, [1997] 3 S.C.R. 320 ..................................................................................................................... 79
R. v. Starr [2000] 2 S.C.R. 144 .......................................................................................................................... 81
R. v. J.H.S., 2008 SCC 30 .................................................................................................................................... 82
Other Burdens........................................................................................................................................................... 83
R. v. Oakes, [1986] 1 S.C.R. 103....................................................................................................................... 83

The Sources of Criminal Law | Table of Contents 6


R. v. Cinous, [2002] 2 S.C.R 3 ............................................................................................................................ 84
R. v. Fontaine, [2004] 1 S.C.R. 702 .................................................................................................................. 84
The Neutral Impartial Trier ..................................................................................................................................... 85
R. v. Gunning, [2005] 1 S.C.R 627 .................................................................................................................... 85
R. v. Hamilton, [2004] O.J. No. 3252 (Ont. C.A.) ............................................................................................. 85
The Role of the Prosecutor ..................................................................................................................................... 86
Boucher v. The Queen, [1955] S.C.R. 16 .......................................................................................................... 86
Krieger v. Law Society of Alberta, [2002] 2 S.C.R. 372 ................................................................................. 87
R. v. Nixon, 2011 SCC 34 ................................................................................................................................... 87
The Role of the Defence ......................................................................................................................................... 88
Getting to the Trial: The Criminal Investigation ........................................................................................................... 88
Police Powers ............................................................................................................................................................... 88
Search and Seizure .................................................................................................................................................. 88
Detention ................................................................................................................................................................... 91
R. v. Grant, 2009 SCC 32.................................................................................................................................... 93
R. v. Suberu, 2009 SCC 33 ................................................................................................................................. 95
R. v. Aucoin, 2012 SCC 66 ................................................................................................................................. 95
R. v. Cole, 2012 SCC 53 ...................................................................................................................................... 96
Getting to the Trial: Taking Control Over the Accused ................................................................................................ 96
Securing Jurisdiction over the Accused and Interim Release ............................................................................... 96
R. v. Hall, [2002] 3 S.C.R. 309 ........................................................................................................................... 98
Getting Ready for Trial ..................................................................................................................................................... 98
Disclosure ...................................................................................................................................................................... 98
R. v. Stinchcombe, [1991] 3 S.C.R. 326 .......................................................................................................... 99
R. v. O’Connor, [1995] 4 S.C.R. 411 ................................................................................................................. 99
R. v. McNeil 2009 SCC 3 ................................................................................................................................... 100
Preliminary Inquiries ................................................................................................................................................. 101
R. v. Arcuri [2001] 2 S.C.R 828 ....................................................................................................................... 102
The Jury Trial ............................................................................................................................................................... 102
R. v. Williams, [1998] 1 S.C.R. 1128 .............................................................................................................. 102
R. v. Find, 2001 SCC 32 .................................................................................................................................... 103
R. v. Yumnu, 2012 SCC 73 ............................................................................................................................... 103
Pre-Trial Motions ........................................................................................................................................................ 104
Sentencing....................................................................................................................................................................... 105
General Principles of Sentencing ............................................................................................................................ 105
R. v. Nasogaluak [2010] 1 S.C.R. 206. No. 6 ................................................................................................ 106

7 Criminal Law and Procedure | FLSC/NCA


R v. C.A.M., [1996] 1 S.C.R. 500 ..................................................................................................................... 107
R. v. Gladue [1999] 1 S.C.R. 688 .................................................................................................................... 108
R. v. Ferguson 2008 SCC 6 .............................................................................................................................. 109
R. v. Morrisey, 2000 SCC 39 ............................................................................................................................ 110
R. v. Pham, 2013 SCC 15 ................................................................................................................................. 111
Appeals and Review....................................................................................................................................................... 111
Appeals of Final Decisions and Judicial Review of Interim Decisions ............................................................... 111
Appendix .......................................................................................................................................................................... 112
Problem Flowchart ..................................................................................................................................................... 112
Exclusion of Evidence ................................................................................................................................................ 113
Sample Legal Memoranda ....................................................................................................................................... 114
Short Legal Memorandum ................................................................................................................................... 114
Long Legal Memorandum .................................................................................................................................... 115
Model Factum ......................................................................................................................................................... 115

The Sources of Criminal Law | Table of Contents 8


GENERAL OVERVIEW AND PRELIMINARY MATTERS
THE SOURCES OF CRIMINAL LAW
With the exception of contempt of court, criminal offences are created in Canada by statute. Most criminal
offences are created by the Criminal Code but it is not the only statutory source. Drug trafficking, for
example, is made a criminal offence by the Controlled Drugs and Substances Act. The common law cannot
be used to create offences in Canada because of concerns related to the principle of legality, and the notion
that criminal offences should be clear, certain, and should pre-exist the act being prosecuted. As will be seen
below, many rules of criminal procedure are created in the Criminal Code, and many other rules of procedure
are common law based1.

For further information on the above, refer to: Criminal Code, section 9, Roach, page 5.

FREY V. FEDORUK, [1950] S.C.R. 517


RATIO: Common law offences are no longer recognized. No one can be convicted of a crime that isn’t
already an offence under the Criminal Code or other federal criminal statute (exception: contempt of
court).

FACTS: Frey was caught by Fedoruk looking into window of woman’s house.

ISSUES: Can someone be convicted of a crime that is not recognized as an offence under the
Criminal Code? Should courts continue to expand list of common law offences?

DECISION: Courts will no longer use their authority to expand list of offenses at common law because
courts aren’t democratically elected and shouldn’t create laws based on values. Parliament is where
laws are created.

PRECEDENT: Later, amendments to the Criminal Code created: offence of trespassing at night
(1953) and voyeurism (2005)

PITHY: “…no one shall be convicted of a crime unless the offence with which he is charged is
recognized as such in the provisions of the Criminal Code, or can be established by the authority of
some reported case as an offence known to the law. I think that if any course of conduct is now to be
declared criminal, which has not up to the present time been so regarded, such declaration should
be made by Parliament and not by the Courts.”2

While common law offences are not allowed, common law defences are available under Canadian criminal
law and can still be created by the courts. As will be seen below, the Supreme Court of Canada recognized a
common law defence in Levis (City) v. Tetrault, [2006] 1 S.C.R. 420 (officially induced error) and R. v. Mack,
[1988] 2 S.C.R. 903 (entrapment). Moreover, the common law can deeply influence the way that statutory
criminal offences are interpreted, particularly the mental elements 3.

For further information on the above, refer to: Criminal Code, section 8, Roach, pages 104-105.

R. V. JOBIDON, [1991] 2 S.C.R. 714

1 Section 9 of the Criminal Code; Roach, page 5.


2 Cartwright J for the majority, at page 530.
3 Section 8, Criminal Code; Roach, pages 104-105.

9 Criminal Law and Procedure | FLSC/NCA


RATIO: At common law, you are not allowed to consent to serious bodily harm. Consent is not a
defence to assault. (used here to show how words in CCC get interpreted at CL)

FACTS: Jobidon got into a consensual fight with the victim outside a bar. Jobidon’s first punch
rendered the victim unconscious and after a subsequent flurry of punches, the victim lay limp and
later died of contusions to the head. The trial judge acquitted b/c says there was no unlawful act
because the victim consented to the fight; the appeal court overruled this and convicted Jobidon of
manslaughter. The accused appealed.

CHARGE: manslaughter 222.5 culpable homicide resulting in death by a) unlawful death b) crim.
negligence

ISSUE: Is consent a defence to assault?

DECISION: No. Conviction upheld

REASONS: Gonthier - There are common law limitations on consent—consent between adults is
vitiated in the case of fist fights (Jobidon) but this doesn’t apply to sporting events if actions are
within the rules and regulations of the game. Also, must think about what victim consented to—no
way you can give consent when unconscious.

DISSENT: Sopinka - No new offences are supposed to be made at common law and majority’s
application creates a new offence – intentional application of force with the consent of the victim.
Use of CL to eliminate an element of the offence that is required by statute is more than just
interpretation

THE POWER TO CREATE CRIMINAL OFFENCES AND RULES OF CRIMINAL


PROCEDURE
CONSTITUTIONAL DIVISION OF POWERS INTRODUCED
Both the Federal Government and Provincial governments have jurisdiction to create non-criminal offences
(regulatory offences) and to use jail to enforce those regulatory offences, but only the Federal Government
can create “criminal” offences, or “true crimes”, pursuant to its powers under s. 91 (27) of the Constitution
Act, 1867. The principles that apply to true crimes differ from those that apply to regulatory offences. These
principles will be examined below when regulatory offences are discussed.

Curiously, while they cannot create criminal offences, Canadian provinces do have jurisdiction over the
administration of justice within the province under s. 92(14) of the Constitution Act, 1867. For example, the
provinces have set up the lowest level of criminal court where the vast majority of cases are actually
prosecuted (the provincial courts); it is the provincial Attorneys General who prosecute most offences,
including serious offences; and the provinces have passed statutes setting out juror eligibility within the
province. The procedure during criminal hearings, however, is governed by federal rules and by the common
law4.

For further information on the above, refer to: Roach, pages 6-7, 24-30.

R. V. MALMO -LEVINE, 2003, SCC 74

4 Roach, pages 6-7, 24-30.


The Power to Create Criminal Offences and Rules of Criminal Procedure | General Overview and 10
Preliminary Matters
RATIO: The "harm principle" is not a fundamental aspect of our justice system and need not be
included in crimes for them to be constitutionally valid.

FACTS: Challenge to the criminalization of marijuana. Malmo-Levine argues that criminal law is
limited to conduct that causes harm and that the harm principle should be a principle of
fundamental justice.

ISSUE: Must all crimes adhere to the harm principle; is the prohibition of marijuana constitutionally
valid?

REASONS: This claim is ungrounded as there is no principle stating that all laws must adhere to the
"harm principle" and further that there are several crimes in the Code that do not harm others. They
decide that the "harm principle" is not a fundamental aspect of our justice system, and therefore its
violation does not lead to the prohibitions being unconstitutional.

PITHY: RE: The Criminal Law Power

73 The federal criminal law power is “plenary in nature” and has been broadly construed:

A crime is an act which the law, with appropriate penal sanctions, forbids; but as
prohibitions are not enacted in a vacuum, we can properly look for some evil or injurious or
undesirable effect upon the public against which the law is directed. That effect may be in
relation to social, economic or political interests; and the legislature has had in mind to
suppress the evil or to safeguard the interest threatened.

(Reference re Validity of Section 5(a) of the Dairy Industry Act, [1949] S.C.R. 1 (the
“Margarine Reference”), at p. 49)

In the present case the “evil or injurious or undesirable effect” is the harm attributed to the non-
medical use of marihuana.

74 For a law to be classified as a criminal law, it must possess three prerequisites: a valid criminal
law purpose backed by a prohibition and a penalty (Reference re Firearms Act (Can.), [2000] 1
S.C.R. 783, 2000 SCC 31, at para. 27). The criminal power extends to those laws that are designed
to promote public peace, safety, order, health or other legitimate public purpose. In RJR-MacDonald
Inc. v. Canada (Attorney General), [1995] 3 S.C.R. 199, it was held that some legitimate public
purpose must underlie the prohibition. In Labatt Breweries, supra, in holding that a health hazard
may ground a criminal prohibition, Estey J. stated the potential purposes of the criminal law rather
broadly as including “public peace, order, security, health and morality” (p. 933). Of course
Parliament cannot use its authority improperly, e.g. colourably, to invade areas of provincial
competence: Scowby v. Glendinning, [1986] 2 S.C.R. 226, at p. 237.

75 In various instances members of this Court have upheld the constitutionality of the NCA on the
basis of the criminal power: Industrial Acceptance Corp., supra; Hauser, supra, per Dickson J.,
dissenting on this point, at p. 1060; and Schneider v. The Queen, [1982] 2 S.C.R. 112, per Laskin
C.J., at p. 115. Other courts interpreting the Opium Act and its successors have also reached this
conclusion. See, e.g., Dufresne v. The King (1912), 5 D.L.R. 501 (Que. K.B.), and Ex p. Wakabayashi,
[1928] 3 D.L.R. 226 (B.C.S.C.).

76 The purpose of the NCA fits within the criminal law power, which includes the protection of
vulnerable groups: Rodriguez v. British Columbia (Attorney General), [1993] 3 S.C.R. 519, at p. 595.
See also R. v. Morgentaler, [1988] 1 S.C.R. 30, at pp. 74-75, in which s. 251 of the Criminal Code

11 Criminal Law and Procedure | FLSC/NCA


prohibiting abortions except in therapeutic situations was held to have a valid objective, namely
protecting the life and health of pregnant women, although it failed the s. 1 test on other grounds.
On somewhat related issues arising under the Charter, the protection of vulnerable groups has also
been upheld under s. 1 as a valid federal objective of the exercise of the criminal law power. In R. v.
Sharpe, [2001] 1 S.C.R. 45, 2001 SCC 2, we upheld s. 163.1(4) of the Criminal Code prohibiting the
possession of child pornography, noting that the prevention of harm threatening vulnerable
members of society is a valid limit on freedom of expression. Similarly in R. v. Butler, [1992] 1 S.C.R.
452, at p. 497, we concluded that “legislation proscribing obscenity is a valid objective which
justifies some encroachment on the right to freedom of expression”. In so doing, we emphasized the
impact of the exploitation of women and children, depicted in publications and films, which can in
certain circumstances, lead to “abject and servile victimization”. In R. v. Keegstra, [1995] 2 S.C.R.
381, we held that the restrictions on free speech imposed by the hate speech provision in the
Criminal Code was a justifiable limit under s. 1 because of potential attacks on minorities.

77 The protection of vulnerable groups from self-inflicted harms does not, as Caine argues, amount
to no more than “legal moralism”. Morality has traditionally been identified as a legitimate concern
of the criminal law (Labatt Breweries, supra, at p. 933) although today this does not include mere
“conventional standards of propriety” but must be understood as referring to societal values beyond
the simply prurient or prudish: Butler, supra, at p. 498; R. v. Murdock (2003), 11 C.R. (6th) 43 (Ont.
C.A.), at para. 32. The protection of the chronic users identified by the trial judge, and adolescents
who may not yet have become chronic users, but who have the potential to do so, is a valid criminal
law objective. In R. v. Hydro-Québec, [1997] 3 S.C.R. 213, the Court held at para. 131 that
“Parliament has for long exercised extensive control over such matters as food and drugs by
prohibitions grounded in the criminal law power”. See also Berryland Canning Co. v. The Queen,
[1974] 1 F.C. 91 (T.D.), at pp. 94-95; Standard Sausage Co. v. Lee (1933), 60 C.C.C. 265 (B.C.C.A.),
supplemented by addendum at (1934), 61 C.C.C. 95. In our view, the control of a “psychoactive
drug” that “causes alteration of mental function” clearly raises issues of public health and safety,
both for the user as well as for those in the broader society affected by his or her conduct.

78 The use of marihuana is therefore a proper subject matter for the exercise of the criminal law
power. Butler held, at p. 504, that if there is a reasoned apprehension of harm Parliament is entitled
to act, and in our view Parliament is also entitled to act on reasoned apprehension of harm even if
on some points “the jury is still out”. In light of the concurrent findings of “harm” in the courts below,
we therefore confirm that the NCA in general, and the scheduling of marihuana in particular, properly
fall within Parliament’s legislative competence under s. 91(27) of the Constitution Act, 1867.

79 Prior to the enactment of the Charter in 1982, that finding, which validates the exercise of the
criminal law power, would have ended the appellants’ challenge. Now, of course, Parliament must
not only find legislative authority within the Constitution Act, 1867, but it must exercise that
authority subject to the individual rights and freedoms guaranteed by the Charter.

THE CANADIAN CHARTER OF RIGHTS AND FREEDOMS


The Canadian Charter of Rights and Freedoms (the “Charter”) imposes limits on the jurisdiction of all
governments, subject to s. 1, the “reasonable limitations” clause, and the seldom-used s. 33
“notwithstanding clause.” Since its passage in 1982, the Charter has had such a profound impact on criminal
law and procedure that all criminal practitioners need to develop expertise in its operation.

The Power to Create Criminal Offences and Rules of Criminal Procedure | General Overview and 12
Preliminary Matters
Section 52 of the Charter can be used by courts to invalidate offences that Parliament has created, and
courts have done so on a number of occasions, but this is not common. It has also been used to strike down
rules of criminal procedure, although this too is uncommon 5.

For further information on the above, refer to: Roach, pages 31-77.

R. V. HEYWOOD, [1994] 3 S.C.R. 761


RATIO: The law is over-broad; captured more than it needed to in trying to achieve the goal of public
safety for children from sex offenders.

FACTS: Accused was a convicted sex offender charged with vagrancy, by being a person who had
been convicted of a sexual offence and found “loitering at or near a school ground, playground,
public park, or bathing area” which violated s. 179(1)(b) of CCC. He was also observed with camera
at playground, taking pictures.

ISSUE: Is the law overbroad?

REASONS: Cory – does not think that this violation is saved by s.1. The way the section is structured
limits offenders' freedom in ways that go beyond what is necessary for justice. He gives four main
reasons:

1. its geographical limits are too far reaching (includes areas without children);
2. the lifetime ban is unreasonable, and should be subject to review;
3. it limits all sex offenders, when it should only limit those who abused children; and
4. it puts the prohibition in place without notifying the offender first.

DISSENT: Gonthier – Law is not overbroad; should read into the law “activities with a malevolent or
ulterior purpose related to reoffending”

R. V. OAKES, [1986] 1 S.C.R. 103


RATIO: To test if a section is saved under s.1:

1. the section must fulfill an objective related to concerns which are pressing and substantial
in a free and democratic society; and
2. the means chosen must be reasonable and demonstrably justified.

FACTS: Oakes challenged s. 8 of the Narcotics Control Act, which held that if the court finds the
accused in possession of a narcotic, he is presumed to be in possession for purpose of trafficking.
Oakes was found was weed and hash oil. Oakes argued that NCA violates right to be presumed
innocent under s. 11(d) of the Charter because law places burden on accused to prove that s/he is
not in possession for purpose of trafficking.

R. V. LABAYE, [2005] 3 S.C.R. 728


RATIO: Morality is of no use in determining whether activities are indecent; only objective standards
of decency established in Canadian law. What is indecent under Criminal Code is that which is
contrary to principles in the constitution or other important laws.

FACTS: Group sex club.

CHARGE: s. 210(1) - keeping common bawdy-house, for purposes of acts of indecency.

5 Roach 31-77

13 Criminal Law and Procedure | FLSC/NCA


ISSUE: Is the operation of a bawdy-house indecent? Does it cause ‘harm’ to Canadian society?

REASONS: Acts of group sex at a swingers' club were not indecent within the meaning of s. 197(1) of
the Criminal Code of Canada because the acts were relatively private and did not degrade
participants. Therefore, the club was not indecent within the meaning of s. 210(1) of the Code.

THE CLASSIFICATION OF OFFENCES


In Canada, criminal offences are divided into two general categories: “indictable offences” and “summary” (or
“summary conviction”) offences. Offences can be “hybrid” in the sense that the prosecutor has the right to
elect whether to treat the offence as “indictable” or “summary.” The classification of offences has important
implications for the penalties that are possible, and for the procedure that will be used, including the mode of
trial. For example, jury trials are not available for criminal offences prosecuted by summary conviction and
are also precluded for indictable offences listed in s.553 of the Code as being in the absolute jurisdiction of
provincial court judges.

For further information on the above, refer to: Coughlan, pages 34-42.

SUMMARY OFFENCE
 lowest form of offence
 punishable by up to 6 months of jail and/or up to $2000 fine (s. 787 of CCC)
 Super Summary Conviction Offence - can be summary conviction offences, but Parliament decreed
punishment can be up to 18 months in jail
 Can only be tried in some courts

INDICTABLE OFFENCE
 more serious offence (depends on crime, but up to life)
 Can be tried in any court (except jury trial—only superior court)
 Crown elect/dual procedure/hybrid - for some offences, crown can elect whether to proceed by
indictment or summarily

The Classification of Offences | General Overview and Preliminary Matters 14


INTERPRETING CRIMINAL PROVISIONS
Interpreting the Criminal Code and related enactments is not unlike interpreting other statutes. There are
special considerations that operate, however. For example:

DEFINITIONS
The Criminal Code has definitions for many of the terms used but they are not always easy to locate. Section
2 contains definitions that apply throughout the Code. The Code is divided into Parts, and at the beginning of
each Part, there will be a definition section that applies solely to that Part. Sometimes definitions are found
in or around the relevant statutory provision to be interpreted. See, for example, ss.348(3) and 350, which
apply to offences in s. 348(1) (i.e., breaking and entering).

15 Criminal Law and Procedure | FLSC/NCA


STRICT CONSTRUCTION
Historically, criminal statutes were interpreted strictly in favour of the liberty of the accused. In other words,
the accused would get the benefit of the doubt or ambiguity in matters of interpretation. This principle
continues to apply but has been heavily modified by the purposive interpretation.

For further information on the above, refer to: Roach, pages 84-89.

R. V. PARE, [1987] 2 S.C.R. 17


RATIO: Strict construction is used only if ambiguity still exists after purposive methods of
interpretation. Must first look at the intention of parliament and the acts the law is designed to cover
in determining how to interpret ambiguous language.

FACTS: The accused indecently assaulted a boy, threatened to kill him, and killed him 2 minutes
later.

ISSUE: Did accused kill victim “while committing” indecent assault?

REASONS: Wilson –look at purpose of statute, interpreting “while” as exactly simultaneous is


problematic.

PURPOSIVE INTERPRETATION
Canadian law makes liberal use of purposive interpretation, in which the language that is used in the
provision being construed is interpreted harmoniously with the statute as a whole, with the underlying
purpose of the provision in mind so as to best accomplish its underlying purpose, always bearing in mind that
the limit on purposive interpretation is that damage cannot be done to the language employed. R. v. Pare is
an example. Be on the lookout throughout the decisions included in this list for examples of purposive
interpretations.

FRENCH/ENGLISH
Federal laws like the Criminal Code are passed in both of Canada’s official languages. Each version is equally
authoritative, and ambiguities in one language can be clarified by the other.

For further information on the above, refer to: Roach, page 87.

R. V. MAC, [2002] 1 S.C.R. 856


RATIO: The Criminal Code is a bilingual statute of which both the English and French versions are
equally authoritative. In his Interpretation of Legislation in Canada (3rd ed. 2000), at p. 327, Pierre-
André Côté reminds us that statutory interpretation of bilingual enactments begins with a search for
the shared meaning between the two versions. Where the words of one version may raise an
ambiguity, courts should first look to the other official language version to determine whether its
meaning is plain and unequivocal.

In this case, any ambiguity arising from the English version is resolved by the clear and unambiguous
language of the French version of s. 369(b).6

FACTS: The accused was charged with being in possession of machines and materials adapted and
intended to be used in forging credit cards.

6 at 5-6

Interpreting Criminal Provisions | General Overview and Preliminary Matters 16


ISSUE: What is the proper interpretation of the word ”adapted” in s. 369 of the Criminal Code?

R. V. COLLINS , [1987] 1 S.C.R. 265


RATIO: Collins Test: to determine whether admission of evidence would bring admin of justice into
disrepute:

1. Would admitting the evidence adversely affect the fairness of the trial?
a. Is the evidence conscriptive? (yes, then go to b; no, then go to 2)
b. Is the evidence discoverable? (yes, then go to 2; no, then admission of evidence will
render trial unfair and it will generally be excluded w/o considering stages 2 and 3).
2. How serious was the Charter violation?
3. What would be the effect of excluding the evidence on the administration of justice?

FACTS: RCMP drug squad had Ruby Collins under surveillance as part of an investigation into a
“heroin problem”. One of the officers approached her in a local pub, told her that he was a police
officer, and then grabbed her by the throat and in the process dragged her down to the floor in what
is known as a “throat hold” used to prevent suspects from swallowing drug filled balloons. The officer
then told her to let go of a heroin filled balloon she had in her hand, and she did so. The officer then
arrested Collins for drug possession.

ISSUE: When is it OK to admit evidence obtained through a Charter breach?

LANGUAGE: There are two reasons why the threshold for exclusion under s. 24(2) is lower…

The second reason is based on the language of s. 24(2). Indeed, while both the English text of s.
24(2) and Rothman use the words "would bring the administration of justice into disrepute", the
French versions are very different. The French text of s. 24(2) provides "est susceptible de
déconsidérer l'administration de la justice", which I would translate as "could bring the
administration of justice into disrepute". This is suppportive of a somewhat lower threshold than the
English text. As Dickson J. (as he then was) wrote in Hunter v. Southam Inc., supra, at p. 157:

Since the proper approach to the interpretation of the Charter of Rights and Freedoms is a
purposive one, before it is possible to assess the reasonableness or unreasonableness of the
impact of a search or of a statute authorizing a search, it is first necessary to specify the
purpose underlying s. 8: in other words, to delineate the nature of the interests it is meant to
protect.

As one of the purposes of s. 24(2) is to protect the right to a fair trial, I would favour the
interpretation of s. 24(2) which better protects that right, the less onerous French text. Most courts
which have considered the issue have also come to this conclusion (see Gibson, supra, at pp. 63 and
234‑ 35). Section 24(2) should thus be read as "the evidence shall be excluded if it is established
that, having regard to all the circumstances, the admission of it in the proceedings could bring the
administration of justice into disrepute". This is a less onerous test than Rothman, where the French
translation of the test in our reports, "ternirait l'image de la justice", clearly indicates that the resort
to the word "would" in the test "would bring the administration of justice into disrepute" means just
that7.

7 para 42-3

17 Criminal Law and Procedure | FLSC/NCA


R. V. D.(J.), 2002 CANLII 16805 (ON CA)
FACTS: See below.

ISSUE: See below.

RATIO: See below.

PITHY: [10] The French and English versions of s. 72 are equally authoritative. They should be read
together to discern their shared meaning: R. v. Jarvis, supra, at para. 79; R. v. Mac 2002 SCC 24
(CanLII), (2002), 163 C.C.C. (3d) 1 (S.C.C.); Schreiber v. Canada (Attorney General) (2002), SCC 62 at
para. 54.

THE CHARTER
As indicated, the Charter can have an important influence on the way statutory provisions are interpreted
because of the presumption that statutes were intended to be constitutionally valid. You have observed this
in R. v. Labaye, [2005] 3 S.C.R 728.

See, for example, Canadian Foundation for Children, Youth & the Law v. Canada(A.G.), [2004] 1 S.C.R. 76
where a Charter challenge encouraged the Court to read significant content into the concept of “reasonable
corrective force.” Examine this decision not only for what it shows about legal technique, and the rule of law
doctrine of “void for vagueness,” but also for what it says about the operation of the defence of corrective
force.

CANADIAN FOUNDATION FOR CHILDREN , YOUTH & THE LAW V. CANADA (A.G.), [2004] 1
S.C.R. 76
FACTS: The Foundation sought a declaration that s. 43 of the Criminal Code, which allows parents
and teachers to use force to correct a child’s behavior, was in violation of ss. 7, 12 and 15(1) of the
Charter.

DECISION: The majority opinion was written by Chief Justice McLachlin with Gonthier, Iacobucci,
Major, Bastarache and LeBel JJ. Concurring:

Section 7: Section 7 protects individuals from violation of their personal security. McLachlin found
that there was no violation of section. The Crown had conceded that the law adversely affected the
child's security of person, so the issue was whether the violation offended a principle of fundamental
justice. The Foundation proposes three claims as mentioned above. McLachlin rejected the first
claim that it failed to give procedural protection as children receive all the same protection as
anyone else. On the second claim, she rejects that the "best interests of the child" is a principle of
fundamental justice as there is no "consensus that it is vital or fundamental to our societal notion of
justice."

On the third claim she rejects the claim that the law is vague and overbroad on grounds that the law
"delineates a risk zone for criminal sanction". She examines the meaning of "reasonable under the
circumstances" stating that it includes only "minor corrective force of a transitory and trifling nature",
but it does not include "corporal punishment of children under two or teenagers", or "degrading,
inhuman or harmful conduct" such as "discipline by the use of objects", "blows or slaps to the head"
or acts of anger. The test is purely objective, McLachlin claims.

Section 12: Section 12 prevents "cruel and unusual punishment". Citing the standard of showing
cruel and unusual punishment from R. v. Smith [1987] 1 S.C.R. 1045 as "so excessive as to outrage

Interpreting Criminal Provisions | General Overview and Preliminary Matters 18


standards of decency", McLachlin rejects the claim as the section only permits "corrective force that
is reasonable" thus cannot be excessive by definition.

Section 15(1): Section 15(1) is the equality guarantee that protects individuals from discrimination.
McLachlin examines the claim using the analytical framework from Law v. Canada.

When identifying from whose perspective the analysis must be, McLachlin notes that rather than
take the perspective of a young child, which would prove too difficult, it must be viewed from the
perspective of a "reasonable person acting on behalf of a child" and apprised of the law.

McLachlin says that the claim hinges on demonstrating the lack of "correspondence between the
distinction and the claimant's characteristics or circumstances" (the second contextual factor from
the Law v. Canada test). On this point she acknowledges that children need to be protected, and in
furtherance of this goal parents and teachers require protection as well. Section 43 decriminalizes
"only minimal force of transient or trivial impact" and to remove such protection would be dangerous
as it would criminalize acts such as "placing an unwilling child in a chair for a five-minute "time-out""
which would risk destroying the family.

THE ELEMENTS OF A CRIMINAL OR REGULATORY


OFFENCE
Each criminal offence has “elements” that must be present before a conviction is possible. Indeed, all
elements of the offence must be present at the same time, or there will be no crime (see R. v. Williams
below). As is the case internationally, it is convenient to think of the elements of an offence as:

 The physical elements or actus reus of the offence (the act that must be performed or omission that
is proscribed, the circumstances or conditions in which the act must occur, and any consequence
that must be caused by the act); and
 The mental or mens rea elements of the offence.

In Canadian law, the mental elements normally describe the actual or “subjective” state of mind of the
accused (things such as intent, or planning and premeditation, or knowledge, or willful blindness or
recklessness.). It is becoming increasingly common, however, to produce offences that have an objective
mens rea, such as negligence. Objective mens rea is determined not according to the state of mind of the
accused (the subject), but according to what a reasonable person in the position of the accused would have
known or foreseen.

As a general proposition of interpretation, a true crime will be interpreted as requiring subjective mens rea
unless it is clear that Parliament wished to impose objective liability. Identifying what the elements of an
offence are is a challenging enterprise, turning on interpretation of the offence and familiarity with relevant
precedents and principles. It is not possible or desirable to attempt here to “teach” the elements of every
offence. Instead, some offences will be selected for their illustrative value in demonstrating the key actus
reus and mens rea concepts.

Applicants are expected to be able to demonstrate interpretive and application skills for all criminal offences,
whether included in these reading materials or not. Meaning, candidates are expected to develop the ability
to review an offence provision and analyze it in such a way as to be able to discern its essential elements
(actus reus, mens rea etc.). Sometimes this exercise will involve being cognizant of definitions or
presumptions that may be included in the offence provision or elsewhere in the Criminal Code. A candidate is

19 Criminal Law and Procedure | FLSC/NCA


not expected to have conducted such an analysis with respect to every offence in the Criminal Code prior to
the exam. Nevertheless, the candidate must be able to quickly carry out an analysis of an offence that is put
at issue in an exam question, even if he/she has not previously dealt with that offence in the readings.

THE ACTUS REUS


ACTUS REUS
Elements of AR 1. A physically voluntary
2. Act or omission;
3. Sometimes in certain circumstances;
4. And sometimes causing certain consequences.
1. Voluntariness  Physical voluntariness must be element of Actus Reus of any offence (Larssoneur). Key
issue is autonomy. “author of own misfortune”
 Punishing involuntary actions is unjust because it conflicts with the assumption in criminal
law that individuals are autonomous and freely choosing agents (Ruzic).
2. Omissions  A person will not be criminally liable for failing to act unless he or she is under a legal
duty to act. Two basic circumstances:
1. Specific Omission Offences -- Where the offence specifies and criminalizes a particular
omission to exercise a legal duty.
 S. 50(b) – failure to report high treason
 S. 127 – failure to obey court order
 S. 129(b) – omitting to assist a police officer when requested
 S. 252(1) – failing to stop and assist after involved in accident
 S. 254(5) – failing to provide breath sample.

2. General Omission Offences -- Where offence states generally that it’s predicated on breach
or failure to preform a legal duty.
 S. 180 – common nuisance
 S. 219 – criminal negligence (causing death or bodily harm)
 “Fails to discharge legal duty” or “duty to do” is a general duty to act that creates
obligations. Must then find a legal duty to plug into the general placeholder.

Possible sources of duties:


1. Statutory Law (Browne, Thornton)
2. Provincial Statute (e.g. In QC, duty to help someone if won’t be harmed)
3. Common Law (Coyne, Popen, Nixon, Thornton)
 Duty imposed by common law could “snap in” to common nuisance or criminal
negligence.

Duties that ground liability for general omission offences (imposed by Criminal Code)
 S. 79 – DOC handling explosives
 S. 215(1) – Duty to provide necessities of life
 S. 216 – DOC for those who undertake to administer surgical or medical treatment or other
lawful act that may endanger lives of others to have and use reasonable knowledge, skill
and care in doing so. (Thornton)
 S. 217 – Duty for those who undertake an act to actually do it, if omission can be
dangerous to life. (Browne)
 S. 217.1 – Duty of those directing work to take reasonable steps prevent bodily harm
3. Circumstances  Some acts require that certain circumstances exist

The Actus Reus | The Elements of a Criminal or Regulatory Offence 20


 Circumstances in actus reus are the things that turn otherwise non-criminal or less serious
conduct into criminal or more serious criminal offences. (e.g. consent is the lynchpin of
assault)
4. Consequences  For some offenses, the actus reus requires causing of certain consequences: assault
causing bodily harm (267), crim negl causing harm/death (220/221), all homicides
(222, 224-228), wilful damage to property (430), arson (433), dangerous operation of
a vehicle causing bodily harm/death (249 (3)(4)), and impaired driving (255(2)(3)).
 As part of the Actus Reus, Crown will have to prove the consequence beyond a reasonable
doubt.
Causation
Factual Cause: Was there a causal link between the accused’s actions and the prescribed consequence?
(Winning)
Legal Causation: Asks whether the factual cause is significant enough to justify criminal liability. “legal
causation… is a narrowing concept which funnels a wider range of factual causes into those
which are sufficiently connected to a harm to warrant legal responsibility.” (Maybin, para. 16)
Is the crime first Yes  Harbottle causation
degree murder under Not  Smithers causation
s. 231(5)?
Smithers Test for Crown must establish that the accused’s actions are a “contributing cause of death,
Legal Causation outside the de minimus range.” (ie. whether or not the accused’s actions were a “non-trivial”
(affirmed in Cribbin) or “not insignificant” cause of the prescribed result. )
 Thin skull rule applies (Smithers)
 significant contributing cause” rather than “non trivial cause”, “not insignificant” or “de
minimis” can be used to mean the same (Nette).
Harbottle Test for  In first degree murder cases under s. 231(5) and other sections where the language “caused
Legal Causation by that person” is used, legal causation test is: the actions of the accused must form an
essential, substantial, and integral part of the killing of the victim (Harbottle)
 Will usually involve actual physical involvement in the killing (Harbottle).
 The higher standard used for first degree murder under 231(5) and similar language ONLY.
Otherwise, Smithers standard applies (Nette).
Multiple Contributg  Multiple causes are not a problem. All that matters is if the accused’s actions were a
Causes contributing cause to the outcome. Smithers standard still applies.
Intervening Causes  Generally where actions of intervener are a direct result of the accused’s actions, it will not
sever the chain of action. It is only where the accused’s actions do not precipitate the direct
cause. (Pagett – pregnant gf, Blaue – JW blood transfusion)
TEST:
 Were the actions of the accused a significant contributing outcome? (Maybin) Two
approaches for analyzing the intervening cause:
a) Reasonable foreseeability: Was the general nature of the intervening act
reasonably foreseeable? (Maybin)
b) Independence: Was the intervening act sufficiently independent and
overwhelming to sever the impact of the accused’s actions? (Maybin)

ACTS AND STATUTORY CONDITIONS


The act must be the act of the accused. The act must also be the kind of act described in the relevant
provision. Further, the act must be committed under the circumstances or conditions specified in the offence.
For example, an accused cannot be convicted of the offence of break and enter with intent to commit a
criminal offence pursuant to s. 348 (1) (a) unless he “breaks” and “enters” something that qualifies as a
“place” according to the Criminal Code, with the relevant mens rea.

21 Criminal Law and Procedure | FLSC/NCA


See, for an example of the interpretation of acts and actus reus conditions:

R V J.(D.), [2002] O.J. NO. 4916 (ONT. C.A.)


FACTS: With the officers and their dog in close pursuit, the appellant walked to the front door of 157
Horseley Hill Drive, the residence of Violet Bernard. The appellant knocked and Mark, Ms. Bernard’s
twelve year old son, answered the door. He recognized the appellant as a person he had spoken to
in the park on previous occasions. The appellant said “Hi” and Mark let him into the house. The
appellant then said “Pretend I live here”. He walked to the back door, found it to be blocked by a
couch, and after observing a police officer waiting outside at the back of the house, the appellant
proceeded upstairs.

Ms. Bernard knew the appellant by name and knew that he was an acquaintance of two of her
daughters. He had been in her home before. She was not surprised that the appellant was in her
house on that day, although she did not know he was present until after the police had entered her
home.

ISSUE: On this appeal, the court must interpret s. 72(1) of the Criminal Code, R.S.C. 1985, c. C-46.
The section creates the longstanding, but seldom prosecuted, offence of forcible entry.

REASONS: An interpretation of the provisions which requires a taking of possession in the sense of
an interference with the peaceable possession of the person in actual possession, but does not
require an intention to take over possession of the property is consistent with the French and English
versions of the sections and the purpose of the section. For example, an intruder who forces his or
her way into a home over the objection of the person in actual possession intending only to run
through the house and out the back door would have no intention of taking over possession of the
residence in any permanent sense. The intruder’s conduct would, however, interfere, albeit briefly,
with the owner’s peaceable possession of the residence.

The appellant’s entry into Ms. Bernard’s residence was not accompanied by any force, violence, or
threat of force or violence. He had been in the residence before and was allowed into the residence
by a person who had authority to grant him entry. The appellant was not even a trespasser as long
as he was not asked to leave by Ms. Bernard. On these facts, there is no basis for concluding that
the appellant took possession of the residence when he entered it in the sense that he interfered in
any way with the peaceable possession of the residence by Ms. Bernard and her family.

There is also no evidence that the manner in which the appellant entered the Bernard residence was
such as to create any possibility that his entry or presence in the residence would be resisted by
anyone thereby resulting in a breach of the peace or a reasonable apprehension of a breach of the
peace. There was a possibility of a breach of the peace if the police entered the Bernard residence in
pursuit of the appellant and he resisted any attempt to remove him. Had that breach of the peace
eventuated, it would not have been a product of the manner in which the appellant entered the
Bernard residence, but would instead have been the consequences of events that occurred after his
entry and unrelated to the Bernards’ continued peaceable possession of the property.

R. V. GUNNING [2005] 1 S.C.R. 627


FACTS: The accused fatally shot C, a person unknown to the accused who had entered his home
uninvited during a party. The accused denied that he intended to kill C. Although his memory was
sketchy due to his consumption of alcohol, he testified that C had assaulted him and refused to
leave his house after they had argued. He claimed that he was scared, so he took out and loaded the
shotgun to intimidate C into leaving. He testified that the gun discharged accidentally. The focus of
the trial was on whether the shooting was intentional or accidental. The trial judge, however,

The Actus Reus | The Elements of a Criminal or Regulatory Offence 22


instructed the jury that the offence of careless use of a firearm had been made out and he refused to
instruct the jury on defence of property. Later in his charge, he purported to correct the impugned
instruction on careless use of a firearm. The accused was convicted of second degree murder. The
Court of Appeal upheld the conviction.

ISSUE: Whether trial judge exceeded his proper function by directing jury that offence of careless use
of firearm had been made out and by failing to instruct the jury on the defence of property.

DECISION: The appeal should be allowed. The conviction should be set aside and a new trial ordered.

REASONS: The trial judge erred in instructing the jury that the Crown had proven the “unlawful act”
necessary to prove murder or manslaughter and his recharge did not cure the error. It is a basic
principle of law that the jury is to decide whether an offence has been proven on the facts. The judge
is entitled to give an opinion on a question of fact but not a direction. A trial judge has no duty or
entitlement to direct a verdict of guilty and the duty to keep from the jury affirmative defences
lacking an evidential foundation does not detract from this principle. In this case, if the jury was
satisfied that the accused intended to kill C, the unlawful act that caused the death would be the
shooting itself and the accused would be guilty of murder. If the Crown failed to prove an intent to
kill, the accused would be guilty of manslaughter only if he was guilty of the unlawful act of careless
use of a firearm. If the jury had a reasonable doubt on this question, he was entitled to an acquittal.
In finding that the accused’s use of the firearm was careless within the meaning of s. 86 of the
Criminal Code, and an unlawful act that caused the death of C, the trial judge encroached on the
exclusive domain of the jury. That issue, together with the question of intent to kill, were central in
this trial. It was incumbent upon the trial judge to instruct the jury on the law in respect of the
careless use of a firearm, including any defences that arose on the evidence, and to leave for the jury
the application of the law to the facts. [5] [21‑22] [35]

The trial judge also erred in failing to instruct the jury on the defence of house or property under s.
41 of the Criminal Code. The accused advanced the defence in respect of his use of the firearm prior
and up to what he alleged to have been an accidental shooting. On the evidence, this defence raised
a real issue for the jury to decide, but the jurors were never told that a person is entitled at law to
forcibly remove a trespasser from his home, so long as he uses no more force than necessary. They
were directed, as a matter of law, that the accused’s conduct before the shooting was the unlawful
act of careless use of a firearm. The trial judge effectively determined the merits of the defence, a
matter that was for the jury to resolve. [6] [22] [37‑38]

In view of the fact that the jury was not properly instructed in respect of matters fundamental to the
defence, reliance cannot be placed on the verdict to conclude that there is no reasonable possibility
that the verdict would have been different without these errors. [7]

ACTS MUST BE “VOLUNTARY” OR “WILLED”


The act described by the offence must be “voluntary” in the sense that it must be the willed act of the
accused. For example, a man in the throes of a seizure does not “will” his movements; it would be no assault
on his part even if his arm was to strike another without the other’s consent. It would have been possible to
deal with this kind of issue using the mens rea concept by suggesting that he did not intend to strike the
other, but Canadian law has also accepted that unless a physical motion is willful, it is not fair to call it an act
of the accused person. This is the foundation for the automatism defence, discussed below. It is easier to
understand the concept of voluntariness together with automatism authorities, so this discussion will be
deferred until the voluntariness-based defences are discussed below.

23 Criminal Law and Procedure | FLSC/NCA


For further information on the above, refer to: Roach, page 117-119.

THE “ACT” OF POSSESSION


At times part of the actus reus for an offence has an inherent mental element to it, as it does with the
important element, common to many offences, of “possession.” This concept demonstrates that the divide
between the actus reus and mens rea is not always a solid one. What matters is that lawyers appreciate what
the elements are, regardless of how they are characterized.

For further information on the above, refer to: Criminal Code, section 4(3).

R. V. YORK (2005), 193 C.C.C. (3D) 331 (B.C.C.A.)


FACTS: Two trailers containing furniture shipments were stolen from a yard controlled by PCE Ltd –
shipments total value exceeded 28 000. The appellant received a telephone call from Mr. Shannon.
Mr. Shannon told him that there was "a bunch of stuff" in the warehouse. He asked the appellant if
he had put it there. The appellant replied that he had not. They then drove to the warehouse where
the appellant noted that there was a trailer backed up into one of the bay doors with no tractor unit
hooked to it. When they entered the warehouse, the appellant said he was surprised to see a lot of
furniture and lumber and that he did not know where they came from. He telephoned Mark Rogers,
the manager, and asked him if he knew anything about the goods in the warehouse. Mr. Rogers told
the appellant that he knew about the goods but refused to disclose where they came from. At that
time, the appellant realized that the goods were probably stolen. He said he did not think through
what he was going to do regarding the goods; he simply wanted to get rid of them. The appellant
then borrowed a truck, hooked up the trailer and dropped it off at a location on Carpenter Street, not
far from the warehouse, and he was arrested.

ISSUE: Did the trial judge err in finding that the evidence established, beyond a reasonable doubt,
that the appellant possessed the necessary mens rea for the offence of possession of stolen
property?

HELD: The judge convicted the appellant on the grounds that the appellant knew that the goods were
stolen, and that he exercised physical control over them without notifying the police or the rightful
owners. The blameworthiness of this conduct fell short of that required for a conviction for a crime
of dishonesty. Appeal allowed.

REASONS: The appellant testified. He knew the goods were stolen. He also exercised physical
control over the goods. However, there was no evidence that he had any intention to deprive the
rightful owner of the stolen goods, which is an essential requirement for possession in law.

A brief handling of stolen goods with full knowledge of their character solely for the purpose of
getting rid of them does not constitute possession, for example.

This is because conduct may be characterized as criminal only where the Crown proves the existence
of a blameworthy state of mind.

R. V. MARSHALL, [1969] 3 C.C.C. 149 (ALTA. C.A.)


RATIO: Marshall Inquiry showed that outcomes of criminal justice system turn on small, mundane
decisions. Criminal justice system is fragile and mistakes compound over time, which can lead to
these terrible outcomes. Major lesson from inquiry was that much of these issues resulted from
systemic discrimination and bias against Marshall, an Aboriginal person.

The Actus Reus | The Elements of a Criminal or Regulatory Offence 24


FACTS: Marshall, Aboriginal man, charged and convicted with stabbing death of Sandy Seale. Tried
and convicted and appeal dismissed.

DECISION: Marshall ultimately acquitted and an inquiry held.

REASONS: Police presumed guilt of Marshall, they didn’t investigate witnesses, had “tunnel vision;
Crown didn’t fully disclose all the information they had, including inconsistent statements; shaped
the testimony of witnesses; Defence counsel didn’t do anything to defend Marshall bc they felt he
was guilty; Appeal judge didn’t point out error of law (they tried to blame Marshall for not telling what
happened), trial judge made evidentiary rulings that were incorrect; Other witnesses not pushed in
the way they should have been.

R. V. TERRENCE, [1983] 1 S.C.R. 357


RATIO: In determining possession, per s.4(3) of the Criminal Code, there must be evidence of control
to prove the unlawful act.

FACTS: Terrence goes for ride with another person in their stolen car. Terrence is in the passenger
seat, while the other person drives. There was no evidence to Terrence that it had been stolen.

ISSUE: What is necessary to meet the requirements of possession in s.4(3) of the Criminal Code
(Canada)?

ANALYSIS: Terrence was not driving. He therefore did not meet the requirement of control.

HOLDING: Decision in favour of Terrence.

COMMENTS: There are three types of possession defined in s 4(3):

1. Personal;
2. Constructive (e.g. illegal material in a locker; control over something); or
3. Joint (possession with someone else).

To be in possession, requires:

1. Knowledge of the criminality associated with the item;


2. Consent (per Marshall v R (1969)); and
3. Control (per R v Terrence (1983)).

R. V. MORELLI, 2010 SCC 8, [2010] 1 S.C.R. 253


FACTS: A computer technician arrived at the accused’s house, unannounced, to find him home alone
with his younger daughter. On inspection of the accused’s computer, the technician noticed several
links to both adult and child pornography sites in the taskbar’s favourites list. As well, in the same
room, he noticed home videos and, on a tripod, a webcam that was connected to a videotape
recorder and was pointed at the toys and at the child. When he returned the following morning, he
found that everything had been “cleaned up”. Worried, the technician reported what he saw to a
social worker, who reported this to the RCMP. This led to the drafting of an information to obtain a
search warrant (ITO), which led to a warrant being issued pursuant to s. 487 of the Criminal Code to
search the accused`s computer. Pornographic pictures involving children were found on the
computer and the accused was charged with possession of child pornography contrary to s. 163.1(4)
of the Criminal Code. At trial, he unsuccessfully challenged the validity of the search warrant under
s. 8 of the Canadian Charter of Rights and Freedoms. The trial judge convicted the accused and the
majority of the Court of Appeal upheld the conviction.

25 Criminal Law and Procedure | FLSC/NCA


ISSUE: This case concerns the right of everyone in Canada, including the appellant, to be secure
against unreasonable search and seizure. And it relates, more particularly, to the search and seizure
of personal computers.

DECISION: With Deschamps, Charron and Rothstein JJ. dissenting, it was held that the appeal was to
be allowed. The accused’s conviction was quashed, and an acquittal was entered.

REASONS: Per McLachlin C.J. and Binnie, Fish and Abella JJ.: The ITO is limited to allegations of
possession of child pornography contrary to s. 163.1(4) of the Criminal Code and does not involve
allegations of accessing child pornography pursuant to s. 163.1(4.1). Merely viewing in a Web
browser an illegal image stored in a remote location on the Internet does not establish the level of
control necessary to find possession. Neither does creating a “favourite” or an “icon” on one’s
computer. In order to commit the offence of possession, as opposed to the offence of accessing of
child pornography, one must knowingly acquire the underlying data files and store them in a place
under one’s control. It is the underlying data file that is the stable “object” that can be transferred,
stored, and possessed. The automatic caching of a file to the hard drive does not, without more,
constitute possession. While the cached file might be in a “place” over which the computer user has
control, in order to establish possession it must be shown that the file was knowingly stored and
retained through the cache. An ITO seeking a warrant to search for evidence of possession, rather
than accessing, must therefore provide reasonable grounds to believe that the alleged offender
possesses (or has possessed) digital files of an illegal image, and that evidence of that possession
will be found in the place to be searched at the time the warrant is sought. Here, the search and
seizure of the accused’s computer infringed his right under s. 8 of the Charter. Even when corrected
and amplified on review, the ITO was insufficient to permit any justice of the peace, acting
reasonably, to find adequate grounds for the search. The ITO did not allege the distinct and separate
offence of accessing child pornography and, stripped of its defects and deficiencies, all that really
remained were two Internet links, seen four months earlier in the “Favourites” menu of a Web
browser on a computer that was subsequently formatted, deleting both links. The prior presence of
the two “Lolita” links supports a reasonable inference that the accused browsed a Web site that
contained explicit images of females under the age of 18, but this does not suffice to establish
possession.

CONSENT AS AN ELEMENT OF THE ACTUS REUS


Often the question of absence of consent by the victim is an important actus reus condition that must be
present for offences to occur. Consent is a complex idea, animated by statute and the common law.

For further information on the above, refer to: Criminal Code, section 265(3); Roach, page 101-106.

R. V. JOBIDON, [1991] 2 S.C.R. 714


RATIO: At common law, you are not allowed to consent to serious bodily harm. Consent is not a
defence to assault. (used here to show how words in CCC get interpreted at CL)

FACTS: See above.

ISSUE: See above.

REASONS: See above.

R. V. J.A., 2011 SCC 28

The Actus Reus | The Elements of a Criminal or Regulatory Offence 26


FACTS: One evening, in the course of sexual relations, J.A. placed his hands around the throat of his
long-term partner K.D. and choked her until she was unconscious. At trial, K.D. estimated that she
was unconscious for “less than three minutes”. She testified that she consented to J.A. choking her,
and understood that she might lose consciousness. She stated that she and J.A. had experimented
with erotic asphyxiation, and that she had lost consciousness before. When K.D. regained
consciousness, her hands were tied behind her back, and J.A. was inserting a dildo into her anus.
K.D. gave conflicting testimony about whether this was the first time J.A. had inserted a dildo into her
anus. J.A. removed the dildo ten seconds after she regained consciousness. The two then had
vaginal intercourse. When they finished, J.A. cut K.D.’s hands loose.

K.D. made a complaint to the police two months later and stated that while she consented to the
choking, she had not consented to the sexual activity that had occurred. She later recanted her
allegation, claiming that she made the complaint because J.A. threatened to seek sole custody of
their young son. The trial judge convicted J.A. of sexual assault. A majority of the Court of Appeal
allowed the appeal, set aside the conviction and dismissed the charges against J.A.

ISSUE: Does consent for the purpose of sexual assault require an active mind during the sexual
activity in question?

MAJORITY (McLachlin C.J.): The majority reviewed the definition of consent for sexual assaults found
in section 273.1 of the Criminal Code of Canada, and concluded that: "Parliament viewed consent as
the conscious agreement of the complainant to engage in every sexual act in a particular encounter."
Ultimately, the majority concluded that Parliament intended for a person to have an active mind
during the sexual activity in question.

In coming to their conclusion, the majority noted the following:

1. Consent in advance is not a defense, as a person must be able to withdraw their consent
during the sexual activity in question.
2. The rule only applies to consent in cases of sexual assault.
3. Although this may lead to an odd interpretation, such as one partner kissing the other
partner while they are asleep, the majority found that this was Parliament's intention, and it
cannot be overruled without a constitutional challenge.

As a result, J.A. was guilty of sexual assault.

DISSENT (Fish J.): The dissent found a number of problems with the majority's interpretation:

1. It would deprive women of their freedom to engage in sexual activity that does not result in
bodily harm.
2. It would mean that cohabiting partners, including spouses, risk having one partner commit a
sexual assault when that partner kisses or caresses their sleeping partner, even with that
sleeping partner's prior express consent.

The dissent found that absent a clear prohibition in the Criminal Code, a conscious person can
consent in advance to sexual activity to take place while they are unconscious, provided there is no
bodily harm, and provided the sexual activity did not go beyond what was agreed to.

CRITICISM: Elizabeth Sheehy, a law professor at the University of Ottawa who represented LEAF at
the Supreme Court, said that the decision protects women who are vulnerable to sexual exploitation
because they are asleep, medicated, have episodic disabilities, or are drunk. Sheehy noted that the

27 Criminal Law and Procedure | FLSC/NCA


decision upheld that "unconscious women are not sexually available." 8 Melanie Randall, who drafted
some of the legal arguments for LEAF, said that the decision does not change the law and merely
reaffirms the law that has been in place since 19839.

Martha Shaffer, a law professor at the University of Toronto, said that the Criminal Code was explicit
that a person could not consent if they were unconscious, but it was not expressly clear whether prior
consent could be given. "Now the law is clear: The notion that you give prior consent is not
recognized in Canadian law," Shaffer said10.

Rosie DiManno, a columnist with The Toronto Star, criticized the decision, saying that it "infantilizes"
women11.

R. V. MABIOR, 2012 SCC 47


FACTS: Accused undergoing antiretroviral therapy and having protected and unprotected sexual
relations knowing he was HIV-positive.

ISSUE: Whether approach outlined in R. v. Cuerrier, [1998] 2 S.C.R. 371, remains valid in
determining whether fraud vitiates consent to sexual relations — Whether non-disclosure of HIV
status in circumstances where no realistic possibility of transmission exists can constitute fraud
vitiating consent.

REASONS: Here, the four complainants all consented to sexual intercourse with M, and testified that
they would not have had sex with him had they known he was HIV-positive. M had intercourse by
vaginal penetration with the four complainants, during which he ejaculated. At the time of
intercourse with the complainants S.H., D.C.S. and D.H., M had a low viral load but did not use a
condom. Consequently, those convictions should be maintained. As regards K.G., the record shows
that M’s viral load was low. When combined with condom protection, this did not expose K.G. to a
significant risk of serious bodily harm. This conviction must accordingly be reversed.

CAUSATION
Where the relevant offence prescribes a “consequence” that must occur before the offence is complete, the
Crown prosecutor must prove that the accused caused the consequence to occur, beyond a reasonable
doubt. As Williams shows, if causation is not proved, the accused cannot be convicted of an offence that
requires his act to produce a prohibited consequence. Nette discusses the need for both “factual causation”
and legal causation, as well as discussing the higher causation standard for first degree murder. Smithers
illustrates the legal causation principle of the “thin skull” and Reid demonstrates the need in some contexts
to consider whether intervening events have broken the relevant chain of causation. These cases illustrate
how most imputable causation principles explain why blame can be assigned in criminal cases, in spite of
arguments that might, in civil cases, reduce or even eliminate civil liability.

For further information on the above, refer to: Criminal Code, sections 224-226; Roach, pages 108-115.

8 Mike Blanchfield (27 May 2011). "Woman can’t consent to sex while unconscious, Supreme Court rules".
The Toronto Star. Retrieved 30 May 2011; Alison Crawford (27 May 2011). "No consent in unconscious sex
case: Supreme Court". Canadian Broadcasting Corporation. Retrieved 30 May 2011.
9 Kazi Stastna (27 May 2011). "AnalysisSupreme Court decision on sexual consent". Canadian Broadcasting

Corporation. Retrieved 30 May 2011.


10 Kazi Stastna (27 May 2011). "AnalysisSupreme Court decision on sexual consent". Canadian Broadcasting

Corporation. Retrieved 30 May 2011.


11 Rosie DiManno (29 May 2011). "DiManno: Supreme Court’s consent ruling infantilizes women". The

Toronto Star. Retrieved 30 May 2011.

The Actus Reus | The Elements of a Criminal or Regulatory Offence 28


R. V. SMITHERS, [1978] 1 S.C.R. 506
RATIO: Legal cause is established wherever the accused’s actions are a contributing cause of death,
outside of the de minimus range. Otherwise put, the normal test for legal causation is whether or not
the accused’s actions were a “non-trivial” or “not insignificant” cause of the prescribed result.

FACTS: Smithers kicked victim in stomach and died within five minutes. Autopsy revealed he died
from malfunctioning epiglottis – choking on vomit.

ISSUE: Were Smithers’ actions a significant cause of Cobby’s death, outside of the de minimus
range? Did Smithers cause the death or was it the malfunctioning epiglottis?

REASONS: Dickson, writing for the court, said that the kick had to be an "operating clause outside of
the de minimis range" in order for it to be deemed the cause of death, that is, the Crown had to
prove that the kick caused the vomiting, and that the vomiting caused the death. He accepts the
Crown's argument that this was outside the de minimis range, as the thin skull rule applies in
criminal law and therefore the kick led to the victim's reaction that resulted in death. This is a lower
standard than the Smith test, which is what the defence wanted to be used. The Smithers test needs
a lower threshold of causation to be proven in order to get the conviction.

R V HARBOTTLE 1993 SCC12


RATIO: In first degree murder cases under s. 235(1) and other sections where language of “caused
by that person” is used in the provision, the test for legal causation is: “the actions of the accused
must form an essential, substantial, and integral part of the killing of the victim.”. The higher
standard will usually involve actual physical involvement in the killing.

FACTS: Harbottle and friend forcibly confined a woman, sexually assaulted her and killed her.
Harbottle held her legs down while other strangled her.

ISSUE: Can Harbottle be found guilty of first degree murder pursuant to s. 231(5)?

DECISION: Guilty.

REASONS: Cory –For crimes 231(5) and with language of “caused by that person”, for specifically
first degree murder, the test for causation is higher: Actions of the form an essential, substantial,
and integral part of the killing of the victim. Here it was impossible to distinguish bw strangler and
person who held down legs.

R. V. NETTE, [2001] 3 S.C.R. 488


RATIO: Smithers standard of causation is the one standard in criminal with one narrow exception of
Harbottle for s. 231(5) and sections with the associated language for first degree murder.

FACTS: Nette robbed 95yr woman. After he left, she fell off bed and suffocated.

ISSUE: Does the Smithers causation apply? Should Harbottle apply to second degree murder as well?

DECISION: Conviction of second degree murder upheld. Smithers test applies.

REASONS: Unanimous – Smithers test is one test of causation for criminal law. Court split on how to
articulate the Smithers standard. Majority (Arbour) –preferable to phrase the standard of causation
in positive terms using a phrase such as “significant contributing cause.

12 Not in syllabus.

29 Criminal Law and Procedure | FLSC/NCA


R. V. WILLIAMS [2003] 2 S.C.R. 134
RATIO: An attempt is proven by demonstrating that significant portion of the actus reus has been
completed even though not all of the elements of actus reus have been proven. The absence of
consent is subjective and determined by reference to the complainant’s subjective internal state of
mind towards the touching, at the time it occurred.

FACTS: Sexual affair lasted for 18 months. Condoms were used on occasion, however the usual
precautions against pregnancy were not used because the respondent told her that he had a
vasectomy. The respondent went to a clinic, and he tested positive for HIV. He was given counseling.
He chose to follow none of the recommendations. Complainant eventually got tested once and
tested negative then again and tested positive for HIV. She confronted respondent and he denied
ever being tested. She testified saying she wouldn’t have had sex with him if she knew he was HIV
positive.

ISSUES: Can the accused be convicted of aggravated assault rather then attempted aggravated
assault? Was consent vitiated by consent by fraud?

ANALYSIS: The mens rea for aggravated assault is the mens rea for assault (intent to apply force
intentionally or recklessly or being willfully blind to the fact that the victim does not consent) plus
objective foresight of the risk of bodily harm (R v Godin). An accused is guilty of an attempt if he
intends to commit a crime and takes legally sufficient steps towards its commission.

DECISION: Convicted of attempted aggravated assault.

COMMENT: Judge says he is just applying the Ceurrier principle - But he did not apply the 2 steps of
the Theroux test using and objective standard.

R. V. MAYBIN 2012 SCC 24


RATIO: The governing question will always be the same: were the dangerous, unlawful acts of the
accused a significant contributing cause of the death [or prohibited behaviour]?

FACTS: Maybin beat victim who falls unconscious onto pool table. Bouncer then punches him in back
of head while unconscious. Cause of death was cranial damage.

ISSUE: Was there sufficient link between the Maybins’ assault and the victim’s death to hold them
criminally liable? Trial judge didn’t think so; Court of Appeal thought there was but asked, further:
How to determine if an intervening act ought to break the chain of causation?

REASONS: Two approaches to analyzing intervening causes (Karakatsanis J)

1. Reasonable foreseeability: was the intervening act reasonably foreseeable?


2. Independence: Was the intervening act sufficiently independent and overwhelming to sever
the impact of the accused’s actions?

In this case, the court ruled that the intervening act was reasonably foreseeable and insufficiently
independent to overwhelm the acts of the accused.

R. V. REID, 2003 NSCA 104


RATIO: Did the accused’s unlawful acts in fact amount to a significant contributing cause of the
victim’s death, by being so connected to the death? And if so, did any intervening cause which
resulted in the victim’s death occurring between the accused’s acts and the victim’s death, interrupt
the chain of causation?

The Actus Reus | The Elements of a Criminal or Regulatory Offence 30


FACTS: A few minutes of stupidity after a reckless night of drinking left a young man dead and two
others convicted of manslaughter for killing him.

ISSUE: This appeal raises a most serious point concerning the requirement in law that in order for a
culpable homicide to have been committed, the person accused of the crime must be shown to have
caused the death of a human being.

REASONS: CPR was an intervening cause.

CRITICISM: The argument seems wrong since they do not look at the fact that it is foreseeable that
the choke could have caused death.

OMISSIONS
Some offences do not require a positive act by the accused. Rather, they can be committed by a showing that
the accused failed to act, or omitted to act. Whether an offence can occur by “omission” is a question of
construction.

To be guilty by omission:

1. the offence must contemplate guilt for omissions,


2. the accused must be placed under a legal duty to act either by the provision charging him or by some
incorporated provision, and
3. the omission in question must be a failure to fulfill that legal duty.

For further information on the above, refer to: Roach, pages 115-117.

R. V. MOORE, [1979] 1 S.C.R. 195


RATIO: Omission to act in a particular way will give rise to criminal liability only where a duty to act arises
at common law or is imposed by statute, or by contract.

FACTS: Officer said M ran light on bike. Tried to pull him over. M wouldn’t pull over. Charged with
obstructing justice s. 129 (b) – omission to help a public officer in the execution of his duty13.

ISSUE: Is M guilty of obstructing a police officer?

ANALYSIS: Narrow View: A person obstructs justice that satisfies s.129 (b) if he fails to identify himself to
an officer, when he was seen committing a crime.

Broader View: A person can obstruct justice, in the terms of s.129(b), if he fails to act in a way convenient
to an officer.

MAJORITY: Under the Motor Vehicle Act – every driver of a vehicle and every pedestrian shall obey the
instructions of an applicable traffic-control device. Say it is a major inconvenience (not in public interest
to inconvenience the police) – this is absurd – but is the law

DISSENT: Any duty to identify oneself must be found in common law or by statute. The fact that a police
officer has a duty to identify a person suspected of an offence says nothing about whether the person
has the duty to identify himself on being asked. Criminal law is no place within which to introduce

13 He could have arrested him and used - s.495(2)(d) – can arrest to establish the identity of the person – this
a less serious option.

31 Criminal Law and Procedure | FLSC/NCA


implied duties, unknown to statute and common law, breach which subjects a person to arrest and
imprisonment.

R. V. PETERSON, [2005] O.J. NO. 4450 (ONT. C.A.) 14


RATIO: In interpreting s. 215 as a whole, “under his charge” means the exercise of an element of control
by one person and a dependency on the part of another. One must also consider the existence of a
relationship of trust and the relative capabilities of the two parties.

FACTS: Dennis Peterson was convicted of failing to provide the necessaries of life to his father, Arnold
Peterson, thereby endangering Arnold’s life (s 215). D lived with father A (84 yrs old) in messed up
house. A wasn’t being looked after properly (food, clothes, housing); he fell sick many times etc.

ISSUE: This appeal requires us to consider when a parent is under the charge of a child thereby requiring
the child to provide necessaries of life to that parent pursuant to s. 215 of the Criminal Code.

REASONS: Subsection 215(2) imposes liability on an objective basis. The section indicates that a duty to
provide necessaries also includes a duty to act in an on-going relationship over time.

For a duty to arise under s. 215:

1. One person must be under the other’s charge


2. Person must be unable to withdraw from that charge
3. Person must be unable to provide self with necessities of life.

Criteria for “under his charge”:

 Dependency: Element of trust must be present


 Awareness: person taking care must be aware the person is dependant
 Control: control over the other’s living conditions
 Consciousness of parties: if the person under one’s charge will not cooperate the person taking
care must obtain help from a community agency.
 Responsibility: if the person takes legal or public responsibility for the dependant person

Test for breach of a s. 215 duty: Was there conduct showing a marked departure from the behaviour of
the reasonable person? (A higher standard than the civil standard, which is just the departure from
reasonable behaviour.)

Potential defences: Financial inability (difficult to maintain in the face of the number of community
agencies available.)

Contributory Negligence can only be invoked if the injuries are entirely attributable to the victim.

R V. BROWNE (1997), 116 C.C.C. (3D) 183 (ONT. C.A.) 15


RATIO: Must be an undertaking before there is a legal duty within s. 217. No pre-existing relationship that
creates the legal duty.

FACTS: Two drug dealers, Browne and Greiner. G swallowed bag with crack cocaine and bag ruptured.
Brown said he would “take her to hospital” by taxi. She died. He is going by taxi to be distanced from it in
some form. Trial -- in failing to do this he showed wanton and reckless endangerment of life.

14 Leave to appeal refused.


15 Leave to appeal refused.

The Actus Reus | The Elements of a Criminal or Regulatory Offence 32


ISSUE: Was there an undertaking? Did Browne have a legal duty arising from an “undertaking” within the
meaning of s. 217 to take her to hospital. If so, was he criminally negligent causing death for omitting to
do what was his duty to do?

DECISION: Not guilty. No undertaking and therefore no legal duty.

REASONS: Abella -- An undertaking has to be something upon which reliance could be placed and this
was not considered an undertaking – too casual, too quick. Saying “take you to hospital” is not a firm
enough undertaking.

CRITICISM: It could be said that he undertook and discharged a duty. One could say there is an
undertaking to save her life by taking her immediately to hospital and he failed in doing that. In so doing,
he caused the death of his companion.

SUBJECTIVE MENS REA


As indicated, subjective mens rea focuses on the actual state of mind of the subject of the prosecution,
namely, the accused. Since what someone thinks or wants or knows is personal to him unless
communicated, subjective mens rea ordinarily must be gleaned circumstantially, including by using the
common sense inference that persons usually intend the natural consequences of their acts. Since the state
of “knowledge” is not often manifested circumstantially the way apparent intent is, the law will assume that
the accused knew of the elements of the offence unless the so-called “defence of mistake of fact,” discussed
below, is made out. The close link between knowledge and mistake of fact makes it sensible to discuss the
“defence” together with this mens rea concept.

There are many states of mind described by the various Criminal Code provisions. For example, one form of
first degree murder requires proof of planning and deliberation (premeditation), while second degree murder
requires only that the accused intends to cause death, or intends to cause bodily harm that he knows is likely
to cause death.

Most offences require more than one mental state to exist. For example, to be guilty of murder, the accused
must know that the living thing he is killing is a human being and intend to cause death to that human being.
A sexual assailant must intend to touch the complainant, and know that she is not consenting (although as
indicated, that knowledge will be assumed absent a mistake of fact defence being raised successfully).

It is a close exercise of construction to see what mental states are required by a particular offence. If an
offence is explicit and specifies the relevant state of mind, then only that state of mind will suffice. This is
why “assault” contrary to section 265 requires “intentional” touching, and not simply reckless touching. Many
offences do not specify the relevant mental state. If a true crime is silent as to the mental state and the
offence requires a consequence, it is presumed that intention or “recklessness” in bringing out the
consequence will suffice (that presumption was rebutted for the offence charged in R. v. Buzanga and
Durocher below, requiring the Crown to prove actual intention to bring about the consequence).

Again, subject to exception (see for example C.C. s. 150.1) where an offence sets out conditions or
circumstances that have to exist, the accused must, as a general rule, know that those conditions or
circumstances exist before the offence can be committed, although the mens rea known as “willful
blindness” can substitute for full knowledge. Sometimes the mens rea is provided for in the relevant
provision, and should be construed according to relevant criminal law principles.

MENS REA
 Each different crime has a specific fault element which must be related to the actus reus of that crime.

33 Criminal Law and Procedure | FLSC/NCA


 Principle of Symmetry: the prescribed mens rea must attach to every element of the actus reus.
DETERMINING THE REQUISITE MENS REA
Statutory Provision

Is there EXPRESS language of mens rea? (e.g. intent, wilfully, etc)
 
Yes No
 
Language governs Is the offence a true crime or regulatory offence? (Prue
 and Baril)
1. Nature and seriousness of the offence
o Purpose of object of the
legislation?
o High stigma (mala in se -
deviant and bad)? Low stigma
(mala prohibita -- didn’t live up
to a standard of conduct)
2. Seriousness of the penalty
 
Is the prescribed mens rea unconstitutional? True Crime Regulatory Offence
  
Mens rea is presumptively The mens rea is
Yes if…. any subjective form of presumptively strict
mens rea (Buzzanga and liability (Sault Ste Marie)
  Desrocher)
Stigma offence True crime with Risk of loss of
with objective strict or absolute liberty with
mens rea liability absolute liability
(Vaillancourt) (Hundal) (BC Motor
Vehicles)

FORMS OF MENS REA


Subjective INTENT (“with intent”, “wilful”, Direct Intent – desired or sought the prescribed harm
Forms of Mens “for the purpose of”, “means”) Indirect Intent – desired or sought some other end, but it was
Rea certain or virtually certain that the prescribed harm would be
(accused subj had the result (subj foreseeable). (Hibbert)
guilty mind in
KNOWLEDGE (“knowing”) Has subjective knowledge of some fact or state of affairs
relation to
consequences) RECKLESSNESS Involves knowledge of a danger or risk and persistence in a
course of conduct which creates the risk that the prohibited
result will occur (Sansregret)
WILFUL BLINDNESS Arises where a person who has become aware of the need for
some inquiry declines to make that inquiry bc he does not wish
to know the truth. (Sansregret)
Where willful blindness is shown, that is the same as knowing.
When a crime specifies knowledge as a requirement, willful
blindness will also do (Briscoe)
If the statute is silent, the mens rea for a true crime is satisfied by any of the above (Buzzanga and Desrocher)
If the crime is a stigma offence, the mens rea cannot be below this line (Vaillancourt; Martineau; Finta)
Objective CRIMINAL NEGLIGENCE A marked and substantial departure from the conduct of a
Forms of Mens reasonable person which shows wanton and reckless disregard

Subjective Mens Rea | The Elements of a Criminal or Regulatory Offence 34


Rea for the lives or safety of others. (Tutton and Waite)
(reasonable person PENAL NEGLIGENCE A marked departure from the conduct of a reasonable person
in accused’s
(Hundal) See also Beatty, Roy
position would
have had the True crimes cannot have a mens reas below this line (Hundal)
required guilty STRICT LIABILITY Once the Crown proves actus reus beyond a reasonable doubt,
mind or would have If the statute is silent, this is the the accused can advance a due diligence defence.
acted differently) default mens rea for a regulatory
offence. (Sault Ste Marie)
ABSOLUTE LIABILITY Once the Crown proves actus reus beyond a reasonable doubt,
Not permitted when the liability attaches. No defence based on a an absence of mens
punishment involves a potential rea.
loss of liberty (BC Motor
Vehicles)
Stigma Offences  Constitutional requirement to have a subjective MR to justify the stigma associated with
the conviction and sentence of the offence (Vaillancourt)
 Stigma offences must have symmetry bw all elements of AR and MR the whole way
through (Vaillancourt)
 Rationale: it is a PFJ that there must be proportionality bw the moral blameworthiness of
the person and the seriousness of the conduct (Vaillancourt) therefore subj foresight is
required.
 Includes murder (Vaillancourt, Martineau), attempted murder, theft (Vaillancourt),
crimes against humanity (Finta)
 Manslaughter is NOT a stigma offence (Creighton)
Regulatory offences  Lower form of MR. Three phases of MR developed:
Phase 1: absolute liability
 No mental fault requirement. Upon proof by the Crown beyond a reasonable doubt that
the actus reus is made out, the accused is found guilty (Pierce Fisheries).
Phase 2: strict liability
 Middle standard bw subj MR and absolute liability.
 This is the presumption if the statute is silent (Sault Ste Marie)
 Upon the Crown proving the actus reus beyond a reasonable doubt, the burden shifts to
the accused to prove due diligence on a balance of probabilities (Sault Ste Marie)
Phase 3: constitutional constraints
 You can not be exposed to a loss of liberty without mens rea. Absolute liability and a
potential for a loss of liberty can no longer be combined. (BC Motor)
 Reverse burden on accused to show due diligence is constitutional (Wholesale Travel)
Is it a true crime or a 1. Nature and seriousness of the offence
regulatory offence? o Purpose of object of the legislation?
(Prue and Baril) o High stigma?
2. Seriousness of the penalty

In the cases included below, the most common mental states are identified and illustrated:

INTENTION, AND ULTERIOR MENS REA


Intention is a complex idea. The accused must have the very intention required by the relevant provision. For
example, Vandergraff intended to throw the object, but not to make contact with the victim. His “assault” was
not intended and he was not guilty. He could have been charged with criminal negligence causing injury, but
the wrong charge was laid. For his part Murray intended to hold the Bernardo tapes, but not for the purpose

35 Criminal Law and Procedure | FLSC/NCA


of obstructing justice. He was therefore not guilty. R.(J.S.) intended to shoot into a crowd, with intent to kill a
human being (albeit not the one killed), opening the door to his possible murder conviction during his
upcoming trial, depending on how the evidence comes out. In Roks, the Court of Appeal stresses the
importance of knowing that death is probable and warns of the dangers of reasoning backwards from the
fact that death occurred.

For further information on the above, refer to: Roach, pages 180-187.

R. V. VANDERGRAFF 16, [1994] M.J. NO. 503 (MAN. C.A.)


RATIO: The mens rea for assault with a weapon must be that of the underlying offence of assault
which requires force to be intentionally applied to another person.

FACTS: Charged with the offence of assault with a weapon (a jar of peanut butter). The TJ held that it
was the accused's jar that struck the complainant, hwr the accused did not intend to hit anybody. He
concluded, "that there was the intention to apply force in a general sense, and it happened to be
particularly against the complainant." The accused was convicted on this basis.

ISSUE: What is the intention required for assault with a weapon?

JUDGMENT: The conviction cannot stand. Without proof of an intention to apply force to the
complainant or to another person. The UR offence is an assault, and the intent that must be
established is the intention to apply force, directly or indirectly, to another person.

DECISION: Conviction set aside, acquittal directed.

R. V. MURRAY, [2000] O.J. NO. 2182 (ONT. S.C.J.)


RATIO: ‘Wilfully attempts’ interpreted by the court as the specific intention to obstruct justice, which
was not intention of the D. Therefore, the mens rea was not found.

For actus reus of s.139, we use “Tendency Test”: Would his actions have had the tendency to
obstruct justice? This element of the offence was found 17.

For the mens rea we have a specific intent offence. A specific intent is a more precise form of intent
that is usually linked to a purpose. It is the intention to do an act for a purpose, not just the intent to
do an act. The offence says that he must have intended a specific outcome (he must have intended
to obstruct justice). This was not found.

FACTS: Paul Bernardo’s lawyer (Murray) hiding sex tapes of his client. His expressed intention was to
use them to attack Homolka on witness stand, not to bury/hide them forever.

ISSUE: Did Murray have the mens rea to commit the offence of obstructing justice?

DECISION: Judge could not disbelieve accused BARD. S. 139 requires ‘wilfully attempting’ to obstruct
justice, imposing a mens rea to intend to obstruct. Crown did not prove that specific intention, so not
guilty.

R. V. J.S.R., 2008 O.N.C.A. 544

16Or Vandergraaf?
17Attempting to obstruct justice is construed as the doing of an act which has the tendency to pervert or
obstruct the course of justice (the actus reus).

Subjective Mens Rea | The Elements of a Criminal or Regulatory Offence 36


RATIO: Acts by a 3rd party who is not acting independently but is acting in a furtherance of a joint
activity undertaken by the accused and that 3rd party, will not sever the legal causal connection =
joint endeavour.

An intervening, independent act by a 3rd party that is a more direct cause of a victim’s death then
the prior act of an accused may sever the legal causation connection between the victim’s death and
the prior act of the accused even though the prior act remains a factual or ‘but for’ cause of the
victims death.

FACTS: Charged w 2nd degree murder. Gun battle on street. C died. There was a northbound and
southbound shooter.

ISSUE: Can we say “but for” the actions of J either shooting or preparing to shoot at the shooter
would have no shot and killed the girl? – factual causation.

Is the causal chain linking J to the death of the girl broken bc the last voluntary cause was the other
shooter shooting girl? – legal causation.

ANALYSIS: Where there are multiple wrongful acts it isn’t necessary to know whose wrongful act was
more detrimental to the forbidden consequence18.

If there is no reasonable basis upon which a jury could find that Js actions caused the girls’ death,
then J could not be committed of murder or manslaughter.

Legal causation issue satisfied by the finding that they were in engaged in a joint endeavour.

A reasonable jury could find that each shooter induced the other the engage in a gun fight on a
crowded street. “But for” the decision to engage in a gunfight on a crowded street and the resulting
exchange of bullets, girl would not have been killed.

DECISION: Judge did have a basis for concluding that there was reasonable evidence of causation to
put to jury. J is factually and legally linked to the forbidden consequence (girls’ death).

COMMENTS: Factual causation: But for test – factual chain of events culminating in the death of the
victim.

Legal causation: Is about determining who among those who have factually contributed to an event
should be held legally responsible for that event.

R. V. ROKS, 2011 ONCA 526


FACTS: The plan in this case involved arson, fraud and use of the proceeds of crime. The scheme
was simple enough. Burn down a building after removing anything of value from it. Submit a claim
to recover the proceeds of a policy of fire insurance on the building and its contents. And use the
proceeds of the insurance claim to pay those involved in the plan and its execution, and to finance a
new building on the same site.

The plan failed here when the execution began. The arsonists were too generous in their distribution
of the accelerant, gasoline, inside the building. The volatile mixture exploded. An inferno followed.

18 Both were involved in a joint endeavor, a mutual shootout – thus both are responsible for killing the girl, bc
it is only be chance that A kill C and not B.

37 Criminal Law and Procedure | FLSC/NCA


One arsonist, the principal, died. Another was severely injured and will carry the scars of his
involvement for the rest of his life.

A lengthy police investigation resulted in the arrest and prosecution of several alleged to have
participated in some way or other in the scheme. The charges varied. Adrian Roks was tried for what
the prosecutor alleged was a critical role in the scheme. A judge of the Superior Court of Justice,
sitting without a jury, found Roks guilty of conspiracy to commit arson, arson and second degree
murder.

ISSUE: Is the conviction of murder unreasonable? Did the trial judge make improper use of some
evidence and wrongly admit other evidence? Was the sentence imposed on the conviction of
conspiracy to commit arson erroneous in principle and outside the appropriate range of sentence for
the offence?

DECISION: Dismissed the appeal from the conviction of second degree murder and substituted a
conviction for manslaughter.

REASONS: [139] The unlawful object to which the appellant was a party was a fraud on Magno’s
insurers. A claim would be submitted to the insurers seeking compensation under the policy for the
loss of the building and its contents, nearly all of which had been removed and stored elsewhere,
caused by an unlawful act to which Magno was a party.

[140] The dangerous act in this case, clearly distinct from the unlawful object of defrauding Magno’s
insurers, was setting the store premises on fire. Setting the fire was at once a dangerous act,
committed intentionally, and one done in furtherance of but distinct from the unlawful object of
defrauding Magno’s insurer. The dangerous act was also unlawful.

[141] The deficit in the prosecutor’s proof of murder in this case resides in the failure of the evidence
as a whole to ground the conclusion that the appellant knew that the death of a human being would
likely occur from setting the fire at Woodbine.

[142] In the absence of any direct evidence on the foresight or knowledge issue, this element could
be proven beyond a reasonable doubt only if the only rational inference available from the
circumstantial evidence as a whole was that the appellant (actually) knew that setting the fire at
Woodbine would likely cause somebody’s death. See, R. v. Griffin, 2009 SCC 28 (CanLII), [2009] 2
S.C.R. 42, at para. 33.

[143] The trial judge founded her conclusion that the prosecutor had proven the fault or mental
element required under s. 229(c) beyond a reasonable doubt on four considerations:

i. the precautions taken in setting the fire by recruiting an arsonist with training in “fire
suppression”;
ii. that harm or death is a natural, even a likely consequence, of spreading accelerants in a
building and setting them on fire;
iii. the common sense inference of actual knowledge from constructive knowledge of the
probable consequences of dangerous conduct; and
iv. the scope of destruction planned, total destruction of the building and its remaining
contents.

[144] In my respectful view flaws in the trial judge’s analysis resulted in an unreasonable finding that
the evidence established the fault element required for murder under s. 229(c).

Subjective Mens Rea | The Elements of a Criminal or Regulatory Offence 38


[145] To determine whether the appellant’s conviction of murder is unreasonable, some context is
essential. The plan was to burn down Woodbine so that Magno could collect the proceeds of a fire
insurance policy. The principals in the conspiracy were to share in those proceeds. The appellant was
removed from the setting of the fire. He did not set the fire. He was not at Woodbine when the fire
was set. He was not told about how the fire would be set. He knew that the fire was to be set at
night when people were not around. He knew that someone with “fire suppression” training would
set the fire. It is in this context that we must consider the reasonableness of the conclusion that the
fault element in s. 229(c) was proven with the necessary certainty.

SUBJECTIVE MENS REA WITH OBJECTIVE FEATURES


Some criminal offences use standards to define criminal conduct. For example, some assaults are sexual in
their nature, and others are not. Some acts are dishonest, and others are not. It is not sensible to require the
accused to have a subjective appreciation that the relevant criminal standard has been met before a
conviction can follow since that would permit the content of offences to vary from offender to offender. For
example, the accused can commit fraud if he intends the relevant transaction, even if he does not appreciate
that a transaction of that nature is “dishonest.” If it were otherwise objective dishonest people would be held
to lower standards than the rest of us. Or an accused can commit sexual assault if he intends to touch
another, even if he does not believe that the contact is sexual in nature, so long as it is.

For further information on the above, refer to: Roach, pages 434-437, 454-457.

R. V. THEROUX, [1993] 2 S.C.R. 5


FACTS: Accused, being directing mind of company, made false representations in order to secure
unsecured deposits. Company later became insolvent, and investors lost deposit money. TJ
convicted accused.

ISSUE: Whether the fact that the accused honestly believed that the project would be completed
negates the mens rea of the offence of fraud. There is no doubt that the appellant deliberately
practised a deceitful act, constituting the actus reus of the offence of fraud.

REASONS: Fraud: The prohibited act is deceit, falsehood, or some other dishonest act. The prohibited
consequence is depriving another of what is or should be his, which may, as we have seen, consist in
merely placing another's property at risk. The mens rea would then consist in the subjective
awareness that one was undertaking a prohibited act (the deceit, falsehood or other dishonest act)
which could cause deprivation in the sense of depriving another of property or putting that property
at risk. If this is shown, the crime is complete

HELD: The appellant is guilty of fraud. The actus reus is established: the appellant committed
deliberate falsehoods, which caused deprivation. The mens rea is established: the appellant told the
depositors they had insurance protection when he knew that they did not (i.e. he knew this to be
false; and it may be inferred from his possession of this knowledge that the appellant knew that he
was placing the depositors’ money at risk).

R. V. CHASE, [1987] 2 S.C.R. 293


RATIO: An assault will be deemed a sexual assault when, viewed in the light of all the circumstances,
the sexual nature of the assault is evident to a reasonable observer.

FACTS: Chase grabbed his 15 yr old neighbour around shoulders and grabbed her breasts saying
“c’mon, i know you want it” and that he would tell ppl she had raped him. Charged with sexual
assault.

39 Criminal Law and Procedure | FLSC/NCA


ISSUE: Did the accused have the mental element, knowing there was no consent?

DECISION: Viewed objectively in the light of all the circumstances, it is clear that the conduct of the
respondent in grabbing the complainant's breasts constituted an assault of a sexual nature.

KNOWLEDGE
As indicated, bearing in mind what is said above about standards of criminality, the accused must generally
know that the conditions of the actus reus exist. For example, an accused cannot be convicted of assaulting a
police officer if she does not know the victim is a police officer. Generally, it is unrealistic to expect the Crown
to prove what the accused knows, so we presume the accused knows of the relevant conditions, unless the
accused presents a “mistake of fact defence.” In the sexual offence context, the mistake of fact defence is
heavily limited for policy reasons. A number of provisions deem knowledge where the accused has failed to
take “reasonable steps” to determine actual facts. This goes beyond the doctrine of willful blindness,
discussed below.

For further information on the above, refer to: Criminal Code, sections 265(4), 273.2; Roach, pages 187-189.

R. V. EWANCHUK, [1999] 1 S.C.R. 330


RATIO: Implied consent does not exist in context of sexual assault. The actus reus should be
determined from the perspective of the complainant, and particularly through a consideration of
whether the complainant, in her mind, wanted the sexual touching to take place. Consent for AR –
complainant, in her mind, consented to sexual touching. Consent for MR – complainant affirmatively
communicated by words or conduct an agreement to engage in sexual activity. Silence is not
consent.

FACTS: 17 yr old complainant was victim of sexual touching and said no four times. After each time,
he stopped. Trial judge found her to be credible. Accused argued that there was an implied consent
bc she went along with him, portrayed calm and didn’t leave the trailer.

ISSUE: Is there anything called implied consent?

DECISION: Guilty of sexual assault. She clearly did not consent and furthermore he did not take
reasonable steps to determine if consent was obtained after each time she said no.

REASONS: There is no such thing as implied consent. (see ratio). L’Heureux-Dube – takes up
McClung’s statements. Reinforces the myth that you’re less likely to consent based on how you’re
dressed. Feeds background assumption that this kind of conduct is natural or normal for women to
deal with.

R. V. LEVIGNE, [2010] 2 S.C.R. 3


RATIO: Presumption that accused believed interlocutor is underage — Reasonable steps requirement
to determine if interlocutor underage.

FACTS: Accused communicated by computer with police officer posing as 13 ‑ year ‑ old boy.
Communication indicated accused’s desire to engage in explicit sexual activity. “Boy” insisted he was
13 notwithstanding profile stating his age to be 18. Accused took no steps to ascertain interlocutor’s
real age. Arrangements were made to meet for anticipated sexual encounter. Accused was arrested
and charged with “luring a child”. Accused’s acquittals were overturned on appeal.

Subjective Mens Rea | The Elements of a Criminal or Regulatory Offence 40


ISSUE: Whether trial judge bound by combined effect of presumption of belief in s. 172.1(3) of
Criminal Code and reasonable steps requirement in s. 172.1(4) to find that accused believed he was
communicating by computer with an underage interlocutor.

DECISION: In this case, the accused’s convictions must be upheld.

REASONS: The “reasonable steps” invoked by the accused were in fact neither “reasonable” nor
“steps to ascertain the age of the person” with whom he was communicating by computer for the
avowed purpose of his own sexual gratification. Rather, they were circumstances which explain why
he in fact took no steps to ascertain the actual age of JG. And this despite the latter’s repeated
assertion that he was only 13.

R. V. BEAVER, [1957] S.C.R. 531


RATIO: True crimes we need proof of subjective MR. Mistake of fact is the denial of MR for the
defence – it is not a defence it is just not having the requisite MR to make up the offence.

FACTS: B agreed to sell heroin to an undercover RCMP officer. Defence was that H had told him it
wasn’t heroin.

DECISION: Not guilty of possession.

REASONS: A person cannot be said to possess that substance unless they knew the nature of the
substance. Here, since it was an honest belief, Beaver had no MR.

R. V. ADH, 2013 SCC 28


FACTS: Accused gave birth in a Wal-Mart bathroom, and left the newborn in the toilet. Accused
testified that she had not realized she was pregnant and that she believed child was born dead.

ISSUE: Whether fault element is subjective or objective — Criminal Code, R.S.C. 1985, c. C‑46, s. 214
“‘abandon’ or ‘expose’”, s. 218.

DECISION: The appeal should be dismissed.

REASONS: The legislative history of s. 218 further supports the conclusion that the fault element for
s. 218 is penal negligence. The provision has never included words of subjective intention, as
confirmed by the early English interpretation of the offence. Furthermore, neither the social stigma
associated with it nor the gravity of the offence of child abandonment require it to be treated
differently than its sister provision s. 215 (failure to provide necessaries), where penal negligence
was found to be the requisite fault element.

Under a penal negligence standard, a mistake of fact that is both honest and reasonable affords a
complete defence. Thus, an objective mens rea standard does not punish the morally blameless. In
the present circumstances, the trial judge found that the respondent honestly believed that her child
was dead at birth and that this belief was objectively reasonable. As such, she was entitled to be
acquitted based on the defence of honest and reasonable mistake of fact.

WILFUL BLINDNESS
Willful blindness is related to but distinct from recklessness. It is a subjective state of mind, requiring that the
accused personally sees the risk of a fact, but then willfully avoids confirmation so as to be able to deny
knowledge. This concept fits best when used as a substitute for knowledge, although courts (and Parliament
in CC s. 273.2) have an unfortunate habit of using “wilful blindness” terminology as interchangeable with

41 Criminal Law and Procedure | FLSC/NCA


recklessness. This leads to confusion. If the two concepts were indeed interchangeable willful blindness
would disappear because everyone who is willfully blind is necessarily reckless – if you suspect that a fact
exists but willfully avoid confirmation so as to be able to deny knowledge (and are willfully blind) then you
must necessarily be seeing and taking an unjustifiable risk that the fact may exist (and are reckless). The two
concepts are not the same and should not be equated.

For further information on the above, refer to: Roach, pages 189-191.

R. V. CURRIE (1975), 24 C.C.C. (2D) 292 (ONT.C.A.)


RATIO: Doctrine of wilful blindness: “The rule is that if a party has his suspicion aroused but then
deliberately omits to make further enquiries, because he wishes to remain in ignorance, he is
deemed to have knowledge. In other words, there is a suspicion which the defendant deliberately
omits to turn into certain knowledge”.

FACTS: The accused is asked to cash cheque for some money. He cashes the cheque that turns out
to be fraudulently endorsed.

ISSUES: Was accused wilfully blind?

DECISION: TJ wrong in conviction for wilful blindness. Conviction quashed.

R. V. VINOKUROV, 2001 ABCA 113 (ALTA C.A.)


RATIO: Cannot substitute wilful blindness for recklessness. Cannot be wilfully blind without being
reckless, however you can be reckless without being wilfully blind.

FACTS: Accused manager of a pawn shop. Bought stolen goods, claimed he didn’t know stolen.

ISSUE: Whether this was an issue of recklessness or wilful blindness?

REASON: While wilful blindness is imputed knowledge, recklessness is something greater than that.
Concern is that lowering standard of liability by using recklessness instead of wilful blindness.

R. V. BRISCOE, 2010 SCC 13


RATIO: In order to convict under s.21(1) for the MR the person must have the intent and knowledge
(that the principal was going to carry out an unlawful act) - wilful blindness satisfies this.

FACTS: B drove to golf course with other people. They had planned and started to beat and rape here.
B held her told her to be quiet. B stood by and watched them kill and rape her

ISSUE: Does willful blindness satisfy the MR?

ANALYSIS: s.21(1) makes perpetrators, aiders, and abettors equally liable. The person who provided
the gun, therefore, may be found guilty of the same offence as the one who pulls the trigger. Purpose
= intent and knowledge.

DECISION: B’s actions made him a party to the offence.

COMMENT: Significantly broadens liability – willful bliness satisifies knowledge in this case.

RECKLESSNESS
Recklessness is a subjective state of mind that requires the accused to act in spite of actually and personally
foreseeing the risk that if they do act, the prohibited consequence will be brought about. It therefore differs

Subjective Mens Rea | The Elements of a Criminal or Regulatory Offence 42


from negligence which can apply even if the actor does not personally see the risk, provided a reasonable
person would have. Still, recklessness is a subjective mens rea with objective features because it exists only
where it is objectively unjustifiable to take that risk the accused understood he was taking. The fact that the
accused may have felt the risk to be justifiable would be no answer. Recklessness will apply where the
provision creates a consequence, but does not, as a matter of construction, require some more limited kind
of mens rea.

For further information on the above, refer to: Roach, pages 191-192.

R. V. THEROUX, [1993] 2 S.C.R. 5


FACTS: See above.

ISSUE: See above.

RATIO: See above.

DECISION: See above.

R. V. BUZZANGA AND DUROCHER (1979), 25 O.R. (2D) 705 (ONT. C.A.)


RATIO: Mens rea is presumptively subjective mens rea. “Wilfully” means “intentionally” and doesn’t
include “recklessness”.

FACTS: Defendants put out a satirical document that appeared to be promoting hatred of French
Canadians. They intended the document to support building a French high school. The trial judge
treated testimony that they wished to create a “controversy, furor, and uproar” as admission that
they had a guilty mind. They were charged for wilfully promoting hatred s. 319(2) (281.2(2)).

ISSUE: What is the meaning of “wilfully”?

DECISION: New trial ordered.

REASONS: An intention to create “controversy, furor and uproar” is not the same thing as an
intention to promote hatred and it was an error to equate them.

OBJECTIVE MENS REA AND TRUE CRIMES


Negligence is judged objectively, according to what a reasonable person would know or understand or how a
reasonable person would act. The criminal law has long been uncomfortable with objective fault, as
historically the criminal law responded to an “evil” mind, and careless people may be dangerous but they are
not evil. Gradually the law has come to accept objective fault, but to adapt it to the criminal law by requiring a
marked departure standard from reasonable standards and to require it to be contextualized to reflect all the
circumstances, including after R. v. Beatty the accused’s explanation and state of mind. That said, the
ultimate issue is whether the accused can be said to have engaged in a marked departure from the standard
of care expected of the reasonable person. In R. v. Creighton the Court rejected the idea that mens rea
always has to relate to all aspects of the actus reus. In that case, the Court in a 5:4 decision held that the
fault for unlawful object manslaughter was objective foresight of bodily harm and that the objective test
should be based on a simple reasonable person standard with the personal characteristics of the accused not
being relevant unless they revealed an incapacity to appreciate the prohibited risk. Negligence is not,
however, accepted forfor murder, attempted murder and war crimes, where, as a matter of constitutional
law, convictions must be based on subjective mens rea in the form of full scale intention. For crimes using
objective fault as the mens rea, “penal negligence” - a more restricted form of negligence requiring a marked

43 Criminal Law and Procedure | FLSC/NCA


departure from reasonable standards of care - is generally required. One exception is with “predicate
offences,” those aggravated forms of offence that apply when serious consequences result, and that include
within their elements another complete but lesser offence, a “predicate” offence. For predicate offences the
consequence need not be brought about by “penal negligence.” It is enough if the accused commits the
underlying or predicate offence, and that the aggravated consequence that has been thereby caused was
objectively foreseeable. Also, for the specific offence of criminal negligence, the higher standard of a
“marked and substantial” departure must be proven.

For further information on the above, refer to: Roach, pages 193-202.

R. V. MARTINEAU, [1990] 2 S.C.R. 633


RATIO: Murder is a stigma offence. PFJ require that stigma offences need a subjective mens rea that
reflect the seriousness of the crime. Any form of murder that doesn’t require subjective mens rea all
the way through is unconstitutional.

FACTS: Felony/constructive murder case. Accused in a robbery when accomplice killed two ppl. He
didn’t intend to kill or foresee the death and thought he was participating only in a B&E.

ISSUE: Does felony murder under s. 213 violate ss. 7 and 11(d) of the Charter?

DECISION: Yes. S. 213 violates Charter and not justified under s. 1.

REASONS: Fundamental justice demands that moral blameworthiness is proportional to the stigma
and punishment of the offence. This degree of moral blameworthiness only arises where you have a
subjective mens rea. Here stigma and punishment attaching to murder should be reserved for those
who choose to intentionally cause death or who choose to inflict bodily harm that they know is likely
to cause death. To label and punish a person as a murderer who did not intend or foresee death
unnecessarily stigmatizes and punishes those whose moral blameworthiness is not that of a
murderer.

DISSENT: L’Heureux-Dube –

1. Misplaced compassion. These people are not morally innocent in the first place.
2. There is enough culpability in the act and that is enough to deal with the stigma.
3. If deterrence is ever going to have a role to play, it will be in cases like robberies where if a
person dies you will be labelled a murderer.

R. V. CREIGHTON, [1993] 3 S.C.R. 3


RATIO: Criminal negligence is an objective standard. Section 7 of the Charter does not require that
objective fault relate to all prohibited consequences of the offence (ie. not perfect symmetry). It’s
acceptable that there is only objective foresight of bodily harm, rather than relating to the death.

FACTS: Accused injected cocaine into victim causing death.

CHARGE: s. 222(5)(a) - manslaughter due to an unlawful act.

ISSUE: Does the CL definition of unlawful act manslaughter violate s. 7 of the Charter? Is
manslaughter a crime that should have subjective mens rea?

DECISION: Guilty. Appeal dismissed.

Objective Mens Rea and True Crimes | The Elements of a Criminal or Regulatory Offence 44
REASONS: McLachlin – CL requirement for mens rea for manslaughter (objective foreseeability of
bodily harm as opposed to death) is constitutional. Must be a marked departure from the standard of
care of a reasonable person. Fact that manslaughter is named differently from murder means it is
less blameworthy and less stigma attached. Only in cases of incapacity to appreciate the risk should
personal characteristics be considered.

DISSENT: Lamer – Standard for manslaughter should be objective foreseeability of death and
objective standard should account for person’s experiences and frailties.

R. V. BEATTY, 2008 SCC 5


RATIO: Mens rea for penal negligence is a marked departure from the conduct of a reasonable
person. Mens rea for dangerous driving is a modified objective test: marked departure from the
standard of care expected of a reasonable person in the circumstances of the accused.

FACTS: Drifted into oncoming traffic after momentary lapse and killed three passengers in other car.

CHARGE: Dangerous driving causing death (x3).

ISSUE: Is there a subjective element of fault required for dangerous driving?

DECISION: Not guilty. Appeal allowed.

REASONS: Momentary lack of attention was not a “marked departure”.

REGULATORY OFFENCES
Regulatory offences can be created by any level of government. Regulatory offences can be full mens rea
offences just as true crimes are, but a clear indication that mens rea is required is needed before regulatory
offences will be interpreted as having mens rea elements.. They are presumed to be “strict liability” offences
(offences that can be committed by simple, nonpenal negligence, with the accused bearing the burden of
proving an absence of negligence to avoid conviction). Some regulatory offences operate as absolute liability
offences that will be committed whenever the relevant actus reus is proved, provided this is clearly what the
legislators intended when establishing the offence. Absolute liability offences such as the requirement for
timely retraction in Wholesale Travel offend principles of fundamental justice and will violate s.7 of the
Charter if there is a possibility that they will result in imprisonment or otherwise violate rights to life, liberty or
security of the person. Strict liability offences that require the accused to establish a defence of due diligence
or reasonable mistake of fact on a balance of probabilities violate the presumption of innocence under
s.11(d) of the Charter but as in Wholesale Travel have been upheld as a reasonable limit on such rights given
that the accused has entered a regulated field. Given the different modes of interpretation used, it is
important to be able to distinguish true crimes from regulatory offences. The court has also recently
recognized a defence of officially induced error that can apply both to criminal and regulatory offences, but is
most relevant to regulatory offences.

For further information on the above, refer to: Roach, pages 213-229.

R. V. SAULT STE. MARIE, [1978] 2 S.C.R. 1299


RATIO: There are three different types of offences: true crimes, strict liability offences, and absolute
liability offences. Offences that are created in provincial statutes can only be absolute or strict
liability offences, because provinces have no jurisdiction to enact criminal law. In general, public
welfare offences are strict liability offences. For regulatory offence, the courts will presume the mens

45 Criminal Law and Procedure | FLSC/NCA


rea requirement is strict liability, UNLESS Parliament or the legislature explicitly makes it an
absolute liability offence.

FACTS: City of Sault Ste Marie charged with polluting a stream contrary to Water Resources Act.

ISSUE: What is a public welfare offence and is it a strict liability offence?

DECISION: Offence of causing pollution is a strict liability offence.

REASONS: Pollution offences are undoubtedly public welfare offences enacted in the interests of
public health. There is thus no presumption of full mens rea.

REFERENCE RE SECTION 94(2) OF THE MOTOR VEHICLE ACT (B.C.), [1985] 2 S.C.R. 486
RATIO: PFJ are substantive. Section 7 is the parent for ss 8-14. They are really just specifications of
the kinds of things that are required for a just system. It constitutionalizes Sault Ste. Marie. You will
not be exposed to a loss of liberty without mens rea. Absolute liability and a potential for a loss of
liberty can no longer be combined.

FACTS: BC government had an offence for driving with suspended license whether or not they know
it’s suspended.

ISSUE: Is it constitutional to have absolute liability for driving with a suspended license?

DECISION: No. Any regulatory offence that had absolute liability as a consequence and imprisonment
as a punishment is unconstitutional. Offends PFJ by allowing conviction of morally innocent.

REASONS: A law that has the potential to convict a person who had not really done anything wrong
offends the principles of fundamental justice and, if imprisonment is available as a penalty, such a
law then violates a person’s right to liberty under s. 7.

R. V. WHOLESALE TRAVEL INC. [1991] 3 S.C.R. 15419


RATIO: The reverse burden to show due diligence that is associated with strict liability is
constitutionally acceptable.

FACTS: Company charged with false or misleading advertising contrary to Competition Act for
wrongly advertising that its travel packages were “wholesale”. Accused challenged s. 36 of
Competition Act which created a statutory due diligence defence violated s 11(d) of the Charter.

ISSUE: Upon proof of the act, you are presumed guilty (ie. absolute liability). Was the reverse burden
a breach of the presumption of innocence?

DECISION: Split decision.

REASONS: Lamer – violates the presumption of innocence and cannot be justified under s. 1.
Ioaccobucci – The reverse burden does infringe s. 11(d) but is justified under s.1. For a regulatory
offences, the reversal of burden is justified bc the purpose is to try to get people to take care. Cory,
L’Heureux-Dube – no breach of presumption of innocence under s. 11(d).

R. V. RAHAM, 2010 ONCA 206 (C.A.)

19From syllabus: holding timely retraction requirements to be an unconstitutional form of absolute liability
but upholding strict liability offences that require the accused to establish a due diligence defence on a
balance of probabilities.

Regulatory Offences | The Elements of a Criminal or Regulatory Offence 46


RATIO: There is a presumption in favour of constitutionality, which requires offences to be interpreted
as strict liability offences where possible to construe as absolute or strict liability.

FACTS: The respondent was clocked at 131 kph in an 80 kph zone. The police officer charged her
with an offence under s. 172(1) of the Highway Traffic Act, R.S.O. 1990, c. H.8. That offence,
commonly referred to as stunt driving or racing, is punishable by a fine, a term of imprisonment or
both.

ISSUE: Does the offence of stunt driving by speeding, by being an absolute liability offence
punishable by imprisonment, violate s. 7 of the Charter?

DECISION: Offence does not violate s. 7 of Charter, as the due diligence defence could arise, making
it a strict liability offence, and not an absolute liability offence - speeding simpliciter is included
offence in stunt driving. A new trial was ordered to permit accused opportunity to raise the due
diligence defence.

LÉVIS (CITY) V. TÉTREAULT; LÉVIS (CITY) V. 2629-4470 QUÉBEC INC.20


RATIO: Most regulatory offences are presumed to be strict liability, rebuttable by the language
outlined in Sault Ste. Marie or other Parliamentary intention. The defence of due diligence requires
something more than passivity on the part of the accused.

FACTS: City appealing two separate acquittals: operating a car without having paid registration fees
and driving without a license. Respondents argue that provisions are strict liability and they exercised
due diligence.

ISSUE: Whether ss. 31.1 and 93.1 of Highway Safety Code create strict liability offences — If so,
whether the defence of due diligence is made out.

DECISION: The offences in question must be considered strict liability offences, but the accused have
not shown that they exercised due diligence. Moreover, the company did not make out the defence
of officially induced error.

EXTENSIONS OF CRIMINAL LIABILITY


AIDING AND ABETTING
It is not only the person who actually performs the actus reus (the “principal” offender) who can be convicted
of the offence. So too can those who aid (physically support) or abet (encourage) the accused to commit the
offence. Indeed, persons who aid and abet one offence can, in some circumstances, be convicted of offences
they did not intend to aid or abet, provided that offence is under s.21(2) a foreseeable outcome of the
offence they did intend to aid or abet.

For further information on the above, refer to: Criminal Code, section 21; Roach, pages 147-153.

R. V. DUNLOP AND SYLVESTER, [1979] 2 S.C.R. 881

20 From syllabus: recognizing defence of officially induced error, and summarizing the Court’s approach to
classification of regulatory offences and the due diligence defence.

47 Criminal Law and Procedure | FLSC/NCA


RATIO: Mere presence or passive acquiescence is not sufficient for liability w/o encouragement of
principal offender or act of facilitation. Mere presence itself cannot be interpreted to be
encouragement

FACTS: Motorcycle gang and B at park. About 18 guys had sex with B. D and S came with beer. They
said they were just dropping it off and saw B having sex someone but didn’t know what was going
on. B indentified D and S from lineup.

ISSUE: Does the fact that the accused were present for part of the assault and did nothing to assist
the victim amount to aiding and abetting?

ANALYSIS: Presence at the commission of an offence can be evidence of aiding and abetting if
accompanied by other factors – such as: prior knowledge of the offender’s intentions to commit the
offence, or attendance for the purpose of encouragement.

R v Salajko – girl raped, 3 people charged, S was near girl with pants down did not have intercourse
with her. Holding was the what S did was not encouragement 21.

R. V. LOGAN, [1990] 2 S.C.R. 731


RATIO: Where there is a constitutionally minimum required MR for the principle in those cases the
party must have the same level of MR.

FACTS: L charged with attempted murder during a robbery. L shot X in neck. X lived.

ISSUE: Does s.21(2) of the CC contravene s.7 and/or s.11(d) of the Charter?

DECISION: s.21(2) violates s.7 and cannot be justified under s.1.

ANALYSIS: To be convicted of a party to murder you must have subjective foresight of death.

Vaillancourt does preclude Parliament from providing for the conviction of a party to that offence on
the basis of a degree of MR below the constitutionally required min.

The question whether a party to an offence had the requisite MR to found a conviction under s.21(2)
is 2 steps:

 What is constitutionally required MR for principal? - Is there a min degree of MR required by


PFJ (constitutionally required min MR)? IF SO
 If PFJs do require a certain min degree of MR in order to convict the principal, then that min
degree of MR is constitutionally required to convict a party to that offence as well?

R. V. BRISCOE, 2010 SCC 13


RATIO: See above.

FACTS: See above.

ISSUE: See above.

DECISION: See above.

21Wilson J in Kirkness v. The Queen (1990) says she is troubled by decision in Salajko and that it is
anomalous and should not be followed.

Aiding and Abetting | Extensions of Criminal Liability 48


REASONS: See above.

R. V. THATCHER, [1987] 1 S.C.R. 652


RATIO: An accused can be found guilty as either an aider or abettor under section 21 of the Criminal
Code.

FACTS: Appellant was arrested and charged with causing the death of his ex‑wife. Following their
separation in 1979 after seventeen years of marriage, the relation between the estranged spouses
became increasingly bitter and acrimonious as they fought a long, hotly contested series of custody,
access and matrimonial property battles. On January 21, 1983, appellant's ex‑wife was ferociously
beaten and then shot to death.

ISSUE: Crown's theory that accused could be found guilty as principal or alternatively as aider or
abettor. Whether evidence to justify putting to the jury the Crown's alternative theory. Whether trial
judge failed to apply the legal principles of s. 21 of the Criminal Code to the evidence of the case.

DECISION: The appeal should be dismissed.

REASONS: Lamer J - It is true that the Crown presented two factually inconsistent theories: that the
appellant actually killed the deceased or that he aided and abetted the killer. The overwhelming
mass of the evidence against the appellant, however, was consistent with both theories and pointed
only to his participation in the murder. The jury could not have been convinced beyond a reasonable
doubt of one theory to the exclusion of the other, but must have been convinced beyond a
reasonable doubt that the appellant participated in the murder, either as principal or aider and
abettor. Since, as the Chief Justice points out, s. 21(1) of the Criminal Code makes the distinction
between participation as a principal and participation as aider and abettor legally irrelevant, it was
not necessary for the jury to decide on the form of his participation and the jury was correct in
convicting.

R. V. JF, 2013 SCC 12


RATIO: The Supreme Court of Canada stated that "the aiding and abetting of a conspiracy is an
offence known to Canadian law. The offence is made out where the accused aids or abets the actus
reus of conspiracy, namely the act of agreeing. Where a person with knowledge of a conspiracy does
(or omits to do) something for the purpose of furthering the unlawful object, with the knowledge and
consent of one or more of the existing conspirators, this provides powerful circumstantial evidence of
his membership in the conspiracy. ... Party liability to a conspiracy is limited to cases where the
accused encourages or assists in the initial formation of the agreement, or where he encourages or
assists new members to join a pre-existing agreement22.

FACTS: J, a youth, learned that his friend T and her sister R were planning to murder their mother by
plying her with alcohol and drowning her, a plan which the sisters ultimately executed and were
convicted for. The police found an MSN chat log between J and T in which J provided information to T
about death by drowning; suggested that the sisters should give their mother codeine pills in addition
to alcohol; and suggested ways to mislead the police. The Crown also led evidence that J supplied
the girls with pills and met T and R after the murder to provide an alibi. The trial judge instructed the
jury that J could be convicted of conspiracy to commit murder under s. 465(1) of the Criminal Code
either as a principal, or as a party under ss. 21(1)(b) or (c) of the Criminal Code. J was convicted of
conspiracy to commit murder. The Court of Appeal dismissed an appeal from the conviction but
reduced J’s sentence.

22 See paragraphs 72-74.

49 Criminal Law and Procedure | FLSC/NCA


ISSUE: Whether a person can be a party to the offence of conspiracy. Whether party liability attaches
to someone who knows of conspiracy and does something for the purpose of furthering unlawful
object. Whether trial judge erred in instructions to jury pertaining to conspiracy. Whether curative
proviso should be applied to uphold conviction.

DECISION: The appeal should be dismissed.

REASONS: In light of the conclusion that party liability does not extend to acts done in furtherance of
the unlawful object of the conspiracy, party liability should not, in the present case, have been put to
the jury. There is no evidence that J aided or abetted the initial formation of the agreement between
R and T to murder their mother or aided or encouraged a new member to join the existing
conspiracy. The trial judge’s error, however, could not possibly have affected the verdict. The
curative proviso under s. 686(1)(b)(iii) of the Criminal Code applies. The evidence implicating J as a
member of the conspiracy was overwhelming and, once the jury rejected J’s defence, a finding of
guilt under s. 465(1) of the Criminal Code was inevitable.

R. V. GAUTHIER, 2013 SCC 32


RATIO: The defence of abandonment must be submitted to the jury only if there is evidence in the
record that is reasonably capable of supporting the necessary inferences in respect of each of the
elements of this defence.

The defence can be raised by an accused who is a party to an offence on the basis:

 that he or she did or omitted to do anything for the purpose of aiding any person to commit
the offence, or
 abetted any person in committing it23, or
 on the basis that he or she had formed with other persons an intention to carry out an
unlawful purpose and to assist each other therein and
 that an offence was committed in carrying out the common purpose 24,

if the evidence shows:

1. that there was an intention to abandon or withdraw from the unlawful purpose;
2. that there was timely communication of this abandonment or withdrawal from the person in
question to those who wished to continue;
3. that the communication served unequivocal notice upon those who wished to continue; and
4. that the accused took, in a manner proportional to his or her participation in the commission
of the planned offence, reasonable steps in the circumstances either to neutralize or
otherwise cancel out the effects of his or her participation or to prevent the commission of
the offence.

There will be circumstances in which timely and unequivocal communication by the accused of his or
her intention to abandon the unlawful purpose will be considered sufficient to neutralize the effects
of his or her participation in the crime.

But there will be other circumstances, primarily where a person has aided in the commission of the
offence, in which it is hard to see how timely communication to the principal offender of the person’s

23 s. 21(1) of the Criminal Code


24 s. 21(2) of the Criminal Code

Aiding and Abetting | Extensions of Criminal Liability 50


intention to withdraw from the unlawful purpose will on its own be considered reasonable and
sufficient.

FACTS: Accused charged with being party, together with her spouse, to murder of their three children.

ISSUE: Essential elements of defence of abandonment in context of forms of participation in crime


provided for in s. 21(1) and s. 21(2) of Criminal Code.

DECISION: The appeal should be dismissed.

REASONS: In this case, G’s evidence that she communicated her withdrawal from the deadly plan
and that her communication was timely and unequivocal is insufficient. The record did not contain
evidence upon which a properly instructed jury acting reasonably could have found that G had
abandoned the common unlawful purpose, and could accordingly have acquitted her, if it believed
the evidence to be true. The defence of abandonment therefore did not meet the air of reality test,
and the trial judge was not required to put the defence to the jury.

COUNSELLING
An accused can be convicted of counseling offences, whether or not the offences counseled are actually
committed. If the offences counseled are committed, CC. s. 22 operates. If they are not committed, CC. s.
464 operates.

For further information on the above, refer to: Roach, pages 141-147.

R. V. HAMILTON , [2005] 2 S.C.R. 432


RATIO: The actus reus for counselling is the deliberate encouragement or active inducement of the
commission of a criminal offence. Mere description of the commission of an offence is insufficient 25.
As well, it is well established that it is not necessary that the person counselled be in fact
persuaded26.

To have a completed actus reus for counselling an offence that was not committed, there must be:

1. an act of counselling;
2. communicated to another person;
3. in respect of the commission of an offence.

The mens rea consists in nothing less than an accompanying intent or conscious disregard of the
substantial and unjustified risk inherent in the counselling: that is, it must be shown that the accused
either intended that the offence counselled be committed, or knowingly counselled the commission
of the offence while aware of the unjustified risk that the offence counselled was in fact likely to be
committed as a result of the accused’s conduct27. Mere recklessness is not sufficient, as if this were
the case, “some may argue that the publication of Shakespeare’s Henry VI, with its famous phrase
“let’s kill all the lawyers”, should be subject to state scrutiny! 28”

25 See R. v. Sharpe, [2001] 1 S.C.R. 45, 2001 SCC 2, at para. 57, per McLachlin C.J.
26 See paragraph 74; R. v. Walia (No. 1) (1975), 9 C.R. (3d) 293 (B.C.C.A.), at pp. 293-95; R. v. Glubisz (1979),
47 C.C.C. (2d) 232 (B.C.C.A.), at pp. 235 and 241-42; R. v. Gonzague (1983), 4 C.C.C. (3d) 505 (Ont. C.A.), at
pp. 508-9
27 See paragraph 29.
28 See paragraph 76.

51 Criminal Law and Procedure | FLSC/NCA


FACTS: Accused sent “teaser” e‑mail on Internet marketing the sale of “Top Secret” files. Teaser
advertised software that would enable purchaser to generate valid credit card numbers. Files sold
included instructions on how to make bombs and how to break into a house. Accused was charged
with counselling four offences that were not committed.

ISSUE: Whether accused had requisite mens rea for the counselling offences.

DECISION: The appeal should be allowed on the count of counselling fraud.

REASONS: The trial judge acquitted the accused on the count of counselling fraud because his
motivation was mercenary as opposed to malevolent. The trial judge’s conclusion that the accused
did not intend to induce the recipients to use those numbers is incompatible with the plain meaning
of the “teaser” e‑mail and with her other findings of fact, including her finding that the accused
understood that the use of the generated numbers was illegal. Her assertion that “[h]is motivation
was monetary” immediately after her reference to these facts demonstrates an error of law as to the
mens rea for counselling the commission of a crime, and warrants a new trial. The trial judge
confounded “motive” and “intent”29.

DISSENT: There is no reason to interfere with the trial judge’s conclusion that the accused did not
have the necessary mens rea. Her consideration of the accused’s motivation must be examined in
the context of the evidence before her, and her reasons must be read as a whole. Here, the Court of
Appeal correctly concluded that the trial judge had considered motive as part of her findings of fact,
but that her decision on the issue of mens rea was based on other facts relating to the accused’s
knowledge. It was on the basis of these other facts that the trial judge found the accused lacked
sufficient knowledge of the consequences of his actions to satisfy the mens rea requirement 30.

ATTEMPTS
As the counseling offence in CC. s. 464 illustrates, not all crimes need to be complete before an offence
arises. There is:

1. the discrete offence of counseling,


2. the offence of conspiracy in which the agreement to commit a crime is a crime, and
3. there is liability for attempting to commit an offence.

Ancio shows the relevant mens rea for attempts, and Deutsch is instructive on when the attempt proceeds
far enough to constitute a crime. You should be aware that the fact that an offence is legally impossible in
the factual circumstances is no defence to an attempt charge, but it is not an offence to try to commit an act
you believe is an offence, when it is not actually an offence. Déry exposes the limits of piggy-backing
incomplete forms of liability.

For further information on the above, refer to: Criminal Code, sections 463, 465, 660; Roach, pages 124-136.

R. V. ANCIO , [1984] 1 S.C.R. 225


RATIO: For attempted murder, nothing short of intent to kill will suffice for the mens rea of that
offence.

29 See paragraphs 40-45.


30 See paragraphs 84-86.

Attempts | Extensions of Criminal Liability 52


FACTS: Ancio, wanting to speak with his estranged wife, broke into an apartment building with a
loaded sawed-off shotgun. Kurely, the man with whom his wife had been living, went to investigate
the sound of breaking glass and threw the chair he was carrying at Ancio when he saw him climbing
the stairs. The gun discharged, missing Kurely, and a struggle followed. Shortly after his arrest,
respondent stated to police that he "had him [Kurely] by the throat and I would have killed him." The
trial judge found Ancio had broken into the apartment building with the intent to use the shotgun to
force his wife to leave and convicted him of attempted murder. The Court of Appeal overturned that
conviction and ordered a new trial.

ISSUE: Is the mens rea in attempted murder limited to an intention to cause death or to cause bodily
harm knowing it to be likely to cause death, or is the mens rea required extended to the intention to
do some action constituting murder as defined by ss.212 or 213 of the Code (now ss.229 and 230)?

DECISION: Appeal dismissed.

REASONS: McIntyre, writing for the majority, held that the mens rea for attempted murder is the
specific intent to kill and a mental state falling short of that level, while it might lead to conviction for
other offences, cannot lead to a conviction for an attempt. The completed offence of murder involves
killing and any intention to complete that offence must include the intention to kill. He held that
despite the arguments of the Crown, there is nothing illogical which arises from the fact that in
certain circumstances a lesser intent will suffice for a conviction for murder. By definition, a person
cannot intend to commit the unintentional killings described in ss.212 and 213 of the Code (now
ss.229 and 230). Any illogic lies in the statutory characterization of unintentional killing as murder.

The crime of attempt developed as, and remains, an offence separate and distinct from murder.
While the Crown must still prove both mens rea and actus reus, the mens rea is the more important
element. The intent to commit the desired offence is a basic element of the offence of attempt, and
indeed, may be the sole criminal element in the offence given that an attempt may be complete
without completion of the offence intended.

R. V. DEUTSCH, [1986] 2 S.C.R. 2


RATIO: The actus reus for attempt must be some step towards the actual commission of the crime
that goes beyond mere acts of preparation.

The distinction between preparation and attempt is qualitative and dependent on the relative
proximity of the act (in time, location and between the acts under the control of the accused
remaining to be accomplished) to the nature of the completed offence.

FACTS: Deutsch ran a company, and put an advertisement in the paper looking for "secretary-sales
assistants to the sales executive". When women came in to interview, he told them that they would
have to have sexual intercourse with clients if it was required to secure a contract. However, this
could result in bonuses of up to $100,000 a year. An undercover police officer went in for an
interview and had the same experience. No one was hired under the ad. Deutsch was charged with
attempting to procure females for illicit intercourse with other persons contrary to s.195(1)(a) of the
Code (now s.212). He was convicted at trial, but a new trial was ordered on appeal.

ISSUE: What constitutes "mere preparation"?

DECISION: Appeal dismissed.

53 Criminal Law and Procedure | FLSC/NCA


REASONS: Le Dain, writing for the majority, states that the general section dealing with the elements
of attempt is s.24. The issue in this case is if there was the necessary intent; were the accused's
actions enough to lead to a conviction for intent, or were they merely preparatory actions? He agrees
with Martin in the Court of Appeal that the holding out of large financial rewards for the applicants
was capable of satisfying the actus reus of an attempt to procure the women to have illicit sexual
intercourse contrary to s.212. In general, the actus reus for attempt must be some step towards the
actual commission of the crime that goes beyond mere acts of preparation. In this case the actual
crime could not be committed until one of the women actually had sex with another person;
however, his offering financial rewards was a step in attempting to make this action occur.

R. V. DÉRY , 2006 SCC 53


RATIO: An attempt to conspire to commit a substantive offence is not an offence under Canadian
law. Criminal liability does not attach to fruitless discussions in contemplation of a substantive
crime that is never committed, nor even attempted, by any of the parties to the discussions.

FACTS: D and S were charged with conspiring to commit theft and conspiring to possess stolen
liquor. The trial judge found that no agreement had been established between the two men to steal
or possess liquor and acquitted them of conspiracy, but found their actions more than merely
preparatory to conspiracy and convicted them of attempting to conspire. A majority of the Court of
Appeal affirmed their convictions. D alone appealed to this Court.

ISSUE: Whether offence of attempted conspiracy to commit substantive offence exists in Canadian
criminal law.

DECISION: Appeal should be allowed.

REASONS: Given that conspiracy is essentially a crime of intention, it is difficult to reach further than
the law of conspiracy already allows. Even if it were possible, it has never been the goal of the
criminal law to catch all crime “in the egg”. In this sense, conspiracies are criminalized when
hatched. And they can only be hatched by agreement. This basic element of conspiracy exposes the
otherwise hidden criminal intentions of the parties to it and this demonstrates their commitment to
a prohibited act. By contrast, the criminal law intervenes later in the progression from thought to
deed where someone acts alone. Overt steps are then thought necessary to disclose and establish
with sufficient certainty the criminal intention that is an essential element of the attempt to commit
an offence.

CORPORATE AND ASSOCIATION LIABILITY


Corporations are liable for the acts of their agents for strict and absolute liability offences. Since these kinds
of offences turn on the actus reus alone, there is no need to use any legal devices to ascribe mens rea to the
corporation and so the Criminal Code corporate liability provisions do not apply to regulatory offences. For
true crimes the Criminal Code sets out standards for corporate and association liability. Section 22.1 applies
to objective fault or negligence offences where an association is charged, and s. 22.2 applies to subjective
mens rea offences charged against an association. See these provisions.

For further information on the above, refer to: Roach, pages 230-243.

SELECT CRIMINAL DEFENCES

Corporate and Association Liability | Select Criminal Defences 54


Not all criminal defences are listed below. For example, s. 25 of the Criminal Code permits law enforcement
personnel to use some force to carry out their duties, and s. 40 permits the defence of property. There are
also procedural defences such as double jeopardy. Charges can be “stayed” pursuant to s. 11 (b) and 24 of
the Charter because of unreasonable delay. You are responsible only for the select defences described below
and those described in assigned cases, such as de minimis non curat lex in R. v. J.A. 2011 SCC 28, above.

INVOLUNTARINESS
INVOLUNTARINESS
Involuntariness involves accused NOT being in CONTROL of actions
 
CONSCIOUS NOT CONSCIOUS
Accused was CONSCIOUS but Accused was NOT CONSCIOUS (or impaired) and t/f NOT in control of actions
NOT in control of actions 
 WHY?

This is a defence of simple Extreme Self-Induced Disease of Mind Some cause other than self-
involuntariness that negate the Intoxication induced intoxication or
actus reus. Accused was mental disorder
conscious but action cannot be
fairly attributed to him.
Examples: physical twitch, “Daviault intoxication”, Considered “mental Automatism proper (non-
spasm, accidental mvmt, “intoxication akin to disorder” and will proceed insane automatism)
physical compulsion by another automatism” under NCRMD provisions
(Larsonneur) (s. 16) Examples: physical blows
(Bleta); sleepwalking
(Parks), but unlikely after
Stone/Luedecke; involuntary
intoxication (King); in very
limited circumstances, psych
blow (Rabey/ Stone)
Burden of proof: Crown must Burden of proof: on Burden of proof: Party Burden of proof: Accused
prove basic physical accused on a balance of raising the issue must satisfy must prove automatism on a
voluntariness beyond a probabilities (REVERSE) s. 16 on a balance of balance of probabilities
reasonable doubt probabilities (REVERSE) (After Stone
  presumption is that accused
is NCRMD, not a/t)

If Crown cannot, result is S 33.1 now governs this and If s. 16 is satisfied, the If accused can show
acquittal is a defence ONLY for verdict will be NCRMD and automatism proper, the result
general intent offences NOT the accused will be dealt is acquittal.
involving interference with with through that system (ie.
bodily integrity. review board)

If accused successful, result
is acquittal

MENTAL DISORDER
Section 16 of the Criminal Code modifies the common law defence of insanity. To have access to this
defence the accused must establish that he has a “mental disorder” as defined by the case law and that it
affected him in one or both of the ways described in s.16 (1). R. v. Cooper provides a definition of mental
disorder, although it has been modified by R. v. Parks (discussed below). Cooper also stresses the

55 Criminal Law and Procedure | FLSC/NCA


significance of the concept of “appreciates” while R. v. Kjeldson describes how the defence works for
sociopathic or psychopathic offenders. R. v. Oommen edifies us about the meaning of “wrong.”

For further information on the above, refer to: Roach, pages 279-302.

MENTAL DISORDER (s. 16)


 Negative defence (No AR or No MR)
 Mental d/o negates the MR
 Excuse
Not Criminally  Prior to 1991, defence was “not guilty by reason of insanity” and would result in an acquittal.
Responsible by Held indefinitely until Swain ruled that violation of s. 7.
reason of Mental  Parliament created NCRMD and new regime with goals of public safety and treatment.
Disorder
(NCRMD)
Effect of NCRMD  Disposition hearing by review board. Accused will be (s. 672.54):
o Released (discharged) without conditions;
o Released (discharged) subject to appropriate conditions; or
o Detained in custody in a hospital subject to appropriate conditions.
 The disposition will turn on factors incl: Need to protect the public, Mental condition of the
accused, Reintegration of the accused, Other needs of the accused.
 If person found NCRMD, can be held in an institution indefinitely
 Has the effect that defence won’t necessarily raise the defence bc person could spend less
time incarcerated if they plead guilty and do time.
1. What is a  “Mental Disorder” means a “disease of the mind” (s. 2 of the Criminal Code)
disease of mind?  “Any illness, disorder, or abnormal condition which impairs the human mind and its
functioning” Excluded, however, are: “self-induced states caused by alcohol or drugs, as
well as transitory mental states such as hysteria or concussion.” (Cooper)
 Should be a broad meaning (Cooper)
 Disease of mind is a legal concept, although it includes a medical component. Judge
determines whether the condition constitutes a disease of the mind. (Simpson)
Who can raise  Accused can whenever they wish up until the point that a conviction is entered (ie. after
defence and when? verdict given but before conviction).
 Crown cannot raise MD until accused has put his mental health at issue in the trial OR in the
space bw rendering of verdict and entering of conviction (Swain) Rationale:
 Accused has the right to be found guilty or not guilty of the charge regardless of if they
have a mental illness.
 If MD brought up, early might increase assumption of guilt.
Burden of Proof 16(3) The burden of proof that an accused was suffering from a mental disorder so as to be
exempt from criminal responsibility is on the party that raises the issue.
Reverse burden. If the Crown wants to raise mental disorder, then burden on them. It’s on a
balance of probabilities
TEST of NCRMD 16.(2) Presumption is no mental disorder
(s. 16 of CCC) 16 (3) REVERSE BURDEN: Burden of proof on party raising the issues on a balance of
probabilities.

16 (1) Did the disorder render the accused incapable of appreciating the nature and quality of
the act or omission OR of knowing that it was wrong? (s. 16(1))

A) To appreciate the nature and quality of the act means


1) knowledge of the act AND

Mental Disorder | Select Criminal Defences 56


2) capacity to measure and foresee the consequences. (Cooper)
 Does not mean:
o Having “appropriate” emotional feelings or remorse. (Kjeldsen) Psychopaths are
then guilty and should be convicted and punished.
o Knowing the penal consequences (Abbey)
o Irresistible impulses

B) To determine whether accused was capable of “knowing the act was wrong”
 ASK: Whether, owing to a disease of the mind, an accused was incapable of rationally
evaluating whether the act was “wrong” in either a legal sense OR as a matter of society’s
moral views (Chaulk and Morrissette; Oommen; Landry).
s. 16 of CCC 16(1) No person is criminally responsible for an act committed or an omission made while
suffering from a mental disorder that rendered the person incapable of appreciating the nature
and quality of the act or omission OR of knowing that it was wrong.

(2) Every person is presumed not to suffer from a mental disorder so as to be exempt from
criminal responsibility by virtue of subsection (1), until the contrary is proved on the balance of
probabilities. (this is the presumption of sanity)

(3) The burden of proof that an accused was suffering from a mental disorder so as to be
exempt from criminal responsibility is on the party that raises the issue. (reverse burden. If the
Crown wants to raise mental disorder, then burden on them. It’s on a balance of probabilities)

R. V. COOPER , [1980] 1 S.C.R. 1149


RATIO: Definition of disease of mind is: “any illness, disorder, or abnormal condition which impairs
the human mind and its functioning” Excluded, however, are: “self-induced states caused by alcohol
or drugs, as well as transitory mental states such as hysteria or concussion.” To appreciate means
knowing what you’re doing and also appreciating its consequences.

FACTS: Cooper, a man described as a "little slow", was an outpatient at a psychiatric hospital. At a
dance at a local church organized for patients of the hospital he took a woman aside, kissed her, and
then strangled her to death because, as he later testified, he was afraid that she would tell on him.
At trial the defence of insanity (now mental disorder) was not raised (at the time Cooper would have
been ordered into an institution for life if this defence was accepted), but Cooper rather relied on the
negation of mens rea in the offence. He also had expert testimony from a forensic psychiatrist saying
that he would not have been able to understand the consequences of his actions or form the mens
rea. Despite this, the trial judge charged the jury with s.16, although the charge was very poor.
Cooper was convicted at trial and his appeal was rejected by the Court of Appeal.

ISSUE: What does it mean to “appreciate the nature and quality of an act or omission” and how does
it compare with “knows the nature and quality of his act”?

DECISION: Appeal allowed, new trial ordered.

REASONS: To ‘know’ the nature and quality of an act may mean merely to be aware of the physical
act while to ‘appreciate’ may involve estimation and understanding of the consequences of that act.
In the case of the appellant, in using his hands to choke the deceased, he may well have known the
nature and quality of that physical act of choking. It is entirely different to suggest, however, that, in
performing the physical act of choking, he was able to appreciate its nature and quality in the sense
of being aware that it could lead to or result in her death.

57 Criminal Law and Procedure | FLSC/NCA


R. V. KJELDSEN, [1981] 2 S.C.R. 617
RATIO: Psychopathic personality disorder, while a disease of the mind, alone is not sufficient to allow
the defence of mental disorder.

FACTS: Accused was psychopath. Accused was out on parole / weekend day pass from previous
murder. Left BC and went to another province. Went into Regina, in taxi driven by woman. Later
phoned taxi cab company, requested same driver, raped and murdered her.

Kjeldsen knew what he was doing and that it was wrong – he just didn’t care.

ISSUES: If an accused suffers from a disease of the mind like psychopathy, but is capable of
appreciating the nature and quality of the act or of knowing that it was wrong, is he exempt from
criminal liability under s. 16(2)?

DECISION: No. Appreciation of the nature and quality of the act does not import a requirement that
the act be accompanied by appropriate feeling about the effect of the act on other people.

REASONS: McIntyre J. - While I am of the view that s. 16(2) exempts from liability an accused who by
reason of the mind has no real understanding of the nature, character and consequences of the act
at the time of its commission, I do not think that the exemption provided by the section extends to
one who has the necessary understanding of the nature, character and consequences of the act, but
merely lacks appropriate feelings for the victim or lacks feelings of remorse or guilt for what he has
done, even though such lack of feeling stems from “disease of the mind”.

No doubt the absence of such feelings is a common characteristic of many persons who engage in
repeated and serious criminal conduct.

R. V. OOMMEN, [1994] 2 S.C.R. 507


RATIO: If the accused lacks the capacity to rationally decide whether the act is right or wrong from a
societal standpoint (and hence to make a rational choice about whether to do it or not) then the
defence of insanity applies31.

FACTS: Accused was in a paranoid psychosis. He knew the difference between right and wrong, but
came to believe that his roommate was part of conspiracy to kill him. He murdered her after
interpreting the buzzer as her signal to kill him.

ISSUE: Did the accused have the rational inquiry to determine whether that act is right or wrong and
hence to make a rational choice about whether to do it or not? What is meant by “knowing that act
was wrong” in 16.132? Abstract knowledge of the act or extend to practical situations?

DECISION: Not guilty.

REASONS: Although Oommen was aware that killing was wrong by society’s standards, he was
unable to perceive that it was wrong in these circumstances, because his delusions led him to
believe that he was in genuine danger, and that his actions were therefore justified.

AUTOMATISM AND INVOLUNTARY ACTS “NEGATIVING” THE ACTUS REUS

31 16.1 is not only concerned with the accused’s understanding of right from wrong, but their ability to apply
that knowledge in the specific circumstances.
32 .1 or (1)

Automatism and Involuntary Acts “Negativing” the Actus Reus | Select Criminal Defences 58
As indicated above, the accused does not satisfy the actus reus requirement unless his act is willed. Some
courts have acquitted individuals who reflexively strike out, using the specious reasoning that their physical
act was not willed, but the legitimacy of this reasoning is questionable. A more sophisticated application of
the voluntariness concept was employed in R. v. Swaby.

It is the “voluntariness” concept that explains the defence of automatism, which operates on the theory that
the accused’s physical motions were not culpable where they are not voluntary or thought-directed or
conscious, as in the sleep-walking case of R. v. Parks. Please note that automatism will not realistically
operate in any case where the accused appears conscious of his conduct – it is reserved to those unusual
cases where there appears to be some disconnect between the actions of the accused and his conscious will.
The result of the Parks decision was controversial enough that the Supreme Court of Canada took procedural
steps to cut the defence back in R. v. Stone, although in R. v. Fontaine some of the excessive language of
Stone was qualified by the Court.

Note that “automatism” is divided into two categories, “insane (or mental disorder) automatism” and “non-
insane (non-mental disorder) automatism.” Where a court finds “insane automatism” the real defence it is
applying is “mental disorder,” since an accused person who is automatistic because of a disease of the mind
cannot appreciate the nature and quality of his act or have the capacity to understand that the act is wrong. If
the defence that applies in “non-insane automatism,” a complete acquittal is appropriate, although Stone
has stacked the deck against this kind of defence succeeding.

For further information on the above, refer to: Roach, pages 302-316.

AUTOMATISM
What is the type of  Negative defence (No AR or no MR)
defence?  Negates the AR
 Results in acquittal
Burden of Proof  Reverse burden: Accused must establish the defence on a balance of probabilities (Stone)
 Possible evidence: involuntariness, psychological ev, severity of trigger, bystanders, states of
a/t at other times, motive for crime exists, other explanation.
1. Rationale: accused can easily fake defence; create consistency with Daviault.
Two forms of 2. Mental Disorder automatism
automatism  Involuntary action that results from a disease of the mind
recognized at law:  Results in review board, NCRMD, not acquitted
3. Non-Mental Disorder automatism
 A state of impaired consciousness, rather than unconsciousness, in which an individual,
though capable of action, has no voluntary control over that action (recognized in Rabey;
Stone)
 Results in acquittal
 Can arise from:
o Physical blow
o Physical condition (e.g. stroke, diabetes)
o Psychological blow (Rabey)
 Contextual objective test: “Trigger of involuntary conduct must be an ordinary
event that could send a normal person in similar circumstances into an
automatic state.” (Rabey)
Will it be Two-stage test (Stone):
NCRMD or 1. The judge must assess whether a proper foundation for a defence of automatism has been
automatism established.
proper? a. Is there evidence on the record that person an air of reality to the defence of

59 Criminal Law and Procedure | FLSC/NCA


automatism?
2. If this evidentiary foundation has been laid, the judge must determine whether the condition
is one of mental disorder or non-mental disorder automatism.
a. Consider for 2:

4 Principles (Stone)
1. If the impaired consciousness resulting in involuntary conduct arises from a disease of the
mind, then the proper defence is mental disorder, not automatism.
2. Judges should start from the presumption that the condition the accused claims to have
suffered is a disease of the mind, and then decide whether the evidence takes the condition
out of the disease of the mind category.
3. A holistic approach should be taken by the trial judge in deciding whether it is a disease of
the mind. This holistic approach is informed by:
 The internal cause factor
o If internal, then NCRMD
 The continuing danger factor
o If continuing danger, then NCRMD
 When not clearly internal cause and no continuing danger, judge can consider policy
factors in deciding bw automatism and NCRMD. (e.g. defence is easily faked, public
upset at an outright acquittal).
 All three are considered and then a determination is made.
4. Instances of psychological blow automatism will be very rare, requiring an “extremely
shocking trigger” that might cause “a normal person” to enter into a state of automatism.
(Stone, Rabey)
 Contextual objective test: would a similarly situated ordinary person react?
Consequences  Policy factors – small chance of jury finding automatism proper bc it seems unfair to
just give someone an acquittal and doesn’t sit right with ppl. Rather have NCRMD
where there is a disposition hearing

R. V. SWABY, [2001] O.J. NO. 2390 (ONT.C.A.)


RATIO: In order for the act to be voluntary the accused must have a choice to either acquiesce to the
criminal behaviour or remove himself from participation.

FACTS: The accused was driving when the police followed him the passenger jumped out and ran
into a field. There he was found with a restricted gun. He plead guilty to possession. The accused was
convicted at trial of being an occupant of a vehicle knowing there was present an unlicensed,
restricted weapon. According to the passenger's testimony the gun was the accused's and he gave it
to him to dispose of. According to the accused the gun belonged to the passenger and he had no
knowledge of it prior to his arrest.

ISSUE: At what point are you acting voluntarily?

DECISION: Conviction set aside, new trial ordered.

REASONS: The R had to prove the coincidence of occupancy of the vehicle and the A's knowledge of
the weapon. Although the section under which the A was charged contained no explicit defence it
must be interpreted so as to exclude the possibility of conviction for what would amount to an
involuntary act.

R v. Théroux - the act must be the voluntary act of the accused for the actus reus to exist.

Automatism and Involuntary Acts “Negativing” the Actus Reus | Select Criminal Defences 60
If one acquires knowledge of an illegal weapon while in a moving vehicle, it cannot be the
law that criminal responsibility instantly attaches. There must be some period of time
afforded to the person who has acquired that knowledge to deal w/ the situation.

It is the conduct of the driver following the coincidence of occupancy and knowledge that
counts, and if the driver acts with appropriate dispatch to get either the gun or himself out of
the vehicle, there is no voluntary act for the criminal law to punish.

R. V. PARKS, [1992] 2 S.C.R. 871


RATIO: Automatism works as a defence and results in an absolute acquittal.

Once the defendant raises automatism as a defence the burden is on the Crown to prove
voluntariness, or alternatively to prove "insane" automatism which results in a non-criminal
responsibility verdict but may result in an alternative disposition under s.672.54.

FACTS: Parks attacked his parents-in-law when he was sleepwalking. He drove 23 kilometers to their
house when he was sleepwalking and stabbed them in their sleep with a kitchen knife. His mother-
in-law died, and his father-in-law was seriously injured. He did not remember any of the actions and
there was no reasonable motive for the murder. Parks did not have any mental conditions, although
several members of his family had sleep problems. Parks had been working long hours at work and
had recently been charged with a theft from his employer. He was acquitted both at trial and at the
Court of Appeal.

ISSUE: Does sleepwalking constitute non-insane automatism or it is a "disease of the mind" under
s.16 of the Criminal Code?

DECISION: Appeal dismissed.

REASONS: Lamer held that the expert evidence showed that Parks was indeed sleepwalking at the
time of the attack, that sleepwalking is not a neurological disorder, and that there is no medical
treatment for sleepwalking aside from good health.

La Forest went into detail analyzing automatism. In determining whether or not automatism springs
from a disease of the mind one should look to determine if it is caused by internal (in the mind) or
external factors. One should also consider whether the condition is continuing. Although these are
not determinative, a finding that automatism is internal and continuing suggests a disease of the
mind. In this case there was no evidence of a recurrence of sleepwalking causing a similar outcome.
Again La Forest states that whether or not something is a disease of the mind is a legal question –
although expert evidence helps, it is not determinative.

There is also non-insane automatism, which is where the automatism is caused by external factors,
it is not continual, and is not linked to any disease of the mind. This applies as a complete defence
resulting in the disposition of an acquittal. Although some critics are against applying this defence to
sleepwalking, La Forest states that it cannot lead to an opening of the floodgates because it is so
rare, and that it must be done to uphold the principles of voluntariness required for a conviction.

Once the defendant raises automatism as a defence the burden is on the Crown to show that the
acts were voluntary. They can also prove that the actions were the result of "insane" automatism.

R. V. STONE, [1999] 2 S.C.R. 290


RATIO:

61 Criminal Law and Procedure | FLSC/NCA


1. Definition: Automatism is “a state of impaired consciousness, rather than unconsciousness,
in which an individual, though capable of action, has no voluntary control over that action.”
2. The key question is voluntariness
3. Burden of proof: The accused must convince the trier of fact of automatism on a balance of
probabilities and, although this limits the accused’s rights under s. 11(d), it is justified.

Will it be NCRMD33 or automatism proper? Two-stage test:

1. The judge must assess whether a proper foundation for a defence of automatism has been
established.
2. If this evidentiary foundation has been laid, the judge must determine whether the condition
is one of mental disorder or non-mental disorder automatism.

When is it mental disorder and when is it automatism? – 4 Principles:

1. If the impaired consciousness resulting in involuntary conduct arises from a disease of the
mind, then the proper defence is mental disorder, not automatism.
2. Judges should start from the presumption that the condition the accused claims to have
suffered is a disease of the mind, and then decide whether the evidence takes the condition
out of the disease of the mind category. (Prof: this is a radical contraction of the defence)
3. A new, holistic approach should be taken by the trial judge in deciding whether it is a
disease of the mind. This holistic approach is informed by:
 The internal cause factor
o If internal, then NCRMD
 The continuing danger factor
o If continuing danger, then NCRMD
 And, when it is not clearly an internal cause nor is there a continuing danger, policy
concerns underlying this Automatism/NCRMD split.
o A judge can decide to call it a DOM if there are policy concerns to do so (e.g.
defence is easily faked, public upset at an outright acquittal). Although it
might look like a/t proper, reasons will lead to NCRMD
 All of the above three factors are considered
4. Instances of psychological blow automatism will be very rare, requiring an “extremely
shocking trigger” that might cause “a normal person” to enter into a state of automatism.
 This is a “contextual objective test.”

FACTS: Stone was driving to see his two sons from a previous marriage with his wife. She did not
want him to see them and as a result of her reticence he was only able to visit with them for 15
minutes. On the drive back she continued to berate him, telling him he was a loser, that he was
terrible in bed, that he had a small penis, and that she was going to go to the police with trumped up
assault charges. He pulled the car over and put his head down. He testified that he blacked out and
felt a "woosh" go through his body. When he came to he had stabbed her 47 times with a hunting
knife that he kept in the car. He hid her body in his truck's tool chest, picked up a six pack, drove
home, left a note for his daughter, and took off to Mexico. After a few weeks in Mexico he decided to
return to Canada and turn himself in.

In his defence, Stone pleaded insane automatism, non-insane automatism, lack of intent, and in the
alternative, provocation. The judge allowed for a defence of insane automatism which was presented

33 Not Criminally Responsible on Account of Mental Disorder

Automatism and Involuntary Acts “Negativing” the Actus Reus | Select Criminal Defences 62
to the jury. The jury convicted him of manslaughter and sentenced him to seven years. The verdict
was upheld by the Court of Appeal.

ISSUE: How do you decided between automatism proper and NCRMD? Which is the appropriate
defence to put forward to jury? Was leaving only mental disorder defence to jury justified?

DECISION: Trial judge did not err in refusing to instruct jury on non-mental defence of automatism.
Stone got pushed into NCRMD (even though the evidence clearly showed that was no disease of the
mind).

REASONS: Bastarache -- Starting proposition for trial judges is that involuntary and automatic
behaviour was caused by a disease of the mind. They should only instruct the jury about non-insane
automatism if there is evidence that establishes an “air of reality” for the defence. Objective test for
the trigger from Rabey does not violate ss. 7 and 11(d) of the Charter. In determining whether it’s
insane or non-insane, trial judge should consider policy factors including public safety. Burden of
Proof on accused on a balance of probability.

PRECEDENT: Suggests a strong preference for finding NCRMD where accused establishes that he
was in a dissociative state and acted involuntarily. This is a radical constriction of automatism
proper. Claims now being funnelled through NCRMD.

R. V. FONTAINE, [2004] 1 S.C.R. 702


RATIO: The evidentiary burden is discharged if the accused raises sufficient evidence that there is an
"air of reality" that the defence might succeed; the judge is not to consider the truth of the evidence
when deciding if the accused has met the burden.

FACTS: Fontaine worked "under the table" at a car garage. Renaud, a former employee, called the
store and said "we're coming to get you, pigs". Dompierre, the eventual victim, came in to the store
and told a co-worker that there was a hit out for Fontaine and himself. Having been informed of this,
when he left work, Fontaine purchased a firearm. That evening, Fontaine saw Renaud outside his
house. During the night, Fontaine smoked marijuana and at some point in the night he freaked out
and shot bullets through his windows and doors, thinking that someone was breaking in to get him.
The next day, Dompiere came into the garage to pay off a debt and Fontaine, thinking he was the
hitman, shot him seven times, killing him. He turned himself into the police. He claimed that he was
"frozen" at the time of the murder and only partially recollected it; he said that his actions were not
his own.

Several different doctors testified, and stated that it was possible that Fontaine suffered from
paranoia that was triggered by his habitual smoking of marijuana. This would make him delusional,
however, it was hard to prove and relied mostly on his evidence. However, the doctor for the defence
found conclusively that Fontaine did have a major psychological disorder and that he was unable to
determine right from wrong.

The trial judge did not put mental-disorder automatism to the jury, who convicted him; the Court of
Appeal overturned this and issued a new trial, stating that the defence should be left to the jury.

ISSUE: How much evidence is needed to pass the first step of the automatism defence as laid out in
R v Stone?

DECISION: Appeal dismissed.

63 Criminal Law and Procedure | FLSC/NCA


REASONS: Fish, writing for a unanimous court, states that it is not up to the judge to determine
whether or not the evidence that was put forward is true. All that they should determine is if it is true,
if it could used to properly raise the defence of automatism. The evidentiary burden on the defendant
is proved if he or she shows some evidence, including expert testimony, which could lead to a
properly instructed jury finding that the defence applies (the air of reality test). It is then up to the jury
as finders of fact to determine if the evidence is true, and if the defence applies.

Applying that to the case at bar, the Court of Appeal was correct in ordering a new trial. The accused
did raise sufficient evidence that, if it were true, could lead to a finding that the defence applies.
Therefore, the jury should have been charged with the defence of mental disorder automatism.

R. V. LUEDECKE, 2008 ONCA 716


RATIO: Contrary to Parks, sleepwalking is presumed to be mental disorder 34.

Strong public policy considerations (from Stone’s two-party test) suggest it needs to be a mental
disorder. Possibly dangerous people may be set free without any order on protection of them and
potential victims. Also floodgates policy concern due to ease of feigning.

FACTS: The accused was charged with sexual assault. He admitted that he engaged in non-
consensual sexual relations with the victim, but claimed that he was asleep when the sexual activity
occurred. At trial, he called a sleep-disorder expert who opined that the accused was in a
parasomniac state at the time. The expert explained that a person who is experiencing a
parasomniac episode acts without any volition, consciousness or capacity to control his or her
behaviour. The accused had a family history of parasomnia and had experienced parasomniac
events in the past, including episodes of engaging in sexual intercourse with girlfriends while asleep.
The expert identified factors that are known to trigger parasomniac events in individuals predisposed
to parasomnia, including sleep deprivation, stress and alcohol consumption. Finally, the expert
testified that, from a medical perspective, he did not regard parasomnia as a mental disease or
disorder. Relying mainly on the uncontradicted evidence of the expert, the trial judge found that the
accused's actions were involuntary and that they were not the product of a mental disorder. He
acquitted the accused. The Crown appealed.

ISSUE: Is sleepwalking a mental disorder?

DECISION: Appeal allowed.

REASONS: In acquitting the accused rather than finding him not criminally responsible ("NCR") on
account of mental disorder, the trial judge failed to apply the proper legal standard. If an accused
establishes that he or she acted involuntarily while in a disassociative state, he or she will almost
always be found NCR. At the pre-verdict stage, social defence concerns dominate, focusing on the
risk posed by the potential recurrence of the conduct in issue. Where that risk exists, the risk
combined with the occurrence of the conduct that led to the criminal proceedings will almost always
justify further inquiry into the accused's dangerousness so as to properly protect the public. At the
post-verdict stage, the emphasis shifts to an individualized assessment of the actual dangerousness
of the NCR accused. Where that personalized assessment does not demonstrate the requisite
significant risk, the NCR accused must receive an absolute discharge. Even where a significant risk
exists, the disposition order must be tailored to the specific circumstances of the individual and
must, to the extent possible, minimize the interference with that individual's liberty. In this case, the

34The trial judge called it non-mental automatism; Doherty JA said that this was inaccurate, must apply
Stone (as this is a continual danger).

Automatism and Involuntary Acts “Negativing” the Actus Reus | Select Criminal Defences 64
trial judge failed to appreciate the significance of the hereditary nature of the accused's condition,
failed to give effect to the accused's well-established history of what the defence expert called
"sexsomnia" and failed to appreciate the significance of the strong likelihood of the recurrence of the
events that triggered the sexsomnia. The trial judge also failed to appreciate that the expert's
medical opinion that parasomnia did not constitute a mental disorder was largely irrelevant to the
determination of whether, for policy reasons, the condition should be classified legally as a disease
of the mind.

INTOXICATION
INTOXICATION (s. 33.1)
Summary  Negative defence (either no AR or MR)
 Normal intoxication (No MR); Extreme Intoxication (No AR or MR)
 Results in acquittal if successful
Competing views  Unjust to punish the morally innocent (no MR).
on intoxication  However, don’t want to concede to the person who gets drunk and commits a crime.
1. Is the offence  Argue for one or the other based on the circumstances involved.
specific intent  Classify it as either specific or general intent; then turn it over to the facts of the case.
or general
intent?  General intent offence
 MR is intending to apply physical force to your body. Nothing intended beyond the
physical act itself. It’s a very simple mens rea. E.g. just making body move
 Assault, assault/bodily harm, assult/peace officer, B&E, crim harassment, incest,
indecent assault, manslaughter, mischief, sexual assault, unlawful confinement, wilful
obstruction/peace officer
 Specific intent offences
 MR involves some extra purpose that is a more complicated.
 Look for: Language that requires planning or foresight, a specific type of knowledge
 Aiding and abetting, arson, assault/intent to resist arrest, assaulting police officer,
attempt at any offence, attempted murder, B&E, murder, possession stolen prop, rec
stolen prop, robbery, theft, sexual exploitation, wilfully causing fire

Purpose of the specific/general distinction is not about logic but more about policy driven
distinctions.
2. Does this 1) Normal Intoxication
involve Normal or  Intoxication affects the capacity to form MR.
Extreme 2) Extreme Intoxication (Daviault)
Intoxication?  Intoxication akin to automatism. No capacity to form AR or MR. Brain isn’t in control and
no longer acting voluntarily.
Rules of Is it a specific intent offence? 
Intoxication 1. For specific intent offences, normal intoxication is a defence. The jury must acquit if there
Defence is a reasonable doubt as to whether the accused formed the requisite MR. (George)
o Burden of proof is on the Crown beyond a reasonable doubt.
o D must establish an air of reality

Is it a general intent offence? 


2. For general intent offences, the form of MR required is so simple and basic that normal
intoxication cannot raise a reasonable doubt as to MR. However, extreme intoxication (akin
to automatism) is a defence.

65 Criminal Law and Procedure | FLSC/NCA


o The burden of proof is on the accused on a balance of probabilities. Expert evidence
normally required. (Daviault)

Does it involved interference with bodily integrity of another person?


Yes  Statutory bar based on s. 33.1

3. S. 33.1 - for general intent offences involving interference with bodily integrity of
another, even extreme intoxication is not an available defence.
o For general intent offences involving interference with bodily integrity of another,
intoxication is NO DEFENCE at all.
S. 33.1 of the CCC (1) It is not a defence to an offence referred to in subsection (3) that the accused, by reason of
self-induced intoxication, lacked the general intent or the voluntariness required to commit the
offence, where the accused departed markedly from the standard of care as described in
subsection (2).
(2) For the purposes of this section, a person departs markedly from the standard of reasonable
care generally recognized in Canadian society and is thereby criminally at fault where the person,
while in a state of self-induced intoxication that renders the person unaware of, or incapable of
consciously controlling, their behaviour, voluntarily or involuntarily interferes or threatens to
interfere with the bodily integrity of another person.
(3) This section applies in respect of an offence under this Act or any other Act of Parliament
that includes as an element an assault or any other interference or threat of interference by a
person with the bodily integrity of another person.

SIMPLE INTOXICATION
Intoxication does not operate as a justification or excuse for criminal conduct. This so-called defence of
intoxication (simple intoxication) operates only if proof of the intoxication helps leave the judge or jury in
reasonable doubt over whether the accused formed the mens rea of an offence classified by the courts as a
“specific intent” offence that requires the accused to do an act for an ulterior purpose. Traditionally
intoxication has not been a defence for “general intent” offences, defined as offences that simply require the
doing of an act without an ulterior purpose. In R. v. George, the Supreme Court classified robbery as a specific
intent offence that allows a defence of simple intoxication but found that the included offence of assault was
a general intent offence that did not allow the defence. Please note that in Canada, the inquiry for the
ordinary intoxication defence is no longer into “capacity to form the intent” as it was in common law England
– the defence applies if intoxication prevents the formation of the specific intent required by the relevant
section.

For further information on the above, refer to: Roach, pages 246-255.

THE QUEEN V. GEORGE, [1960] S.C.R. 871


RATIO: Normal intoxication is never a defence for general intent offences. For specific intent
offences, normal intoxication is a defence and can raise a reasonable doubt as to MR.

FACTS: George was drunk, committed robbery and argued defence of intoxication. Trial judge
acquitted him and also decided not to convict him of the lesser offence of assault. Crown appealed
and argued for a distinction bw crimes where there is simple MR (assault) and those with added MR
(robbery).

CHARGE: Robbery (AR: assault MR: intent to steal).

Simple Intoxication | Select Criminal Defences 66


ISSUE: Can intoxication be a defence for criminal activity?

DECISION: Acquitted of robbery; convicted of assault.

REASONS: The mens rea required for a general intent offense is too vague to be negated by “normal”
intoxication, unlike specific intent offenses.

R. V. ROBINSON , [1996] 1 S.C.R. 683


RATIO: Capacity alone is insufficient and unconstitutional, must have intent (Beard rules do not
apply).

FACTS: Intoxicated man struck victim with a rock then fatally stabbed him.

ISSUE: Is capacity to commit offence in cases of intoxication sufficient, or is specific intent required?

DECISION: Appeal dismissed.

REASONS: Proper charge is that jury consider whether evidence of intoxication, along with all other
evidence in the case, impacted on whether the accused possessed the requisite specific intent.
However, examining evidence of capacity can assist in establishing intent, capacity just cannot be
sufficient on its own to impose moral culpability. Regardless, there must be evidence that
intoxication undermined mens rea, i.e. must be some ‘air of reality’ to defence.

EXTREME INTOXICATION
In R. v. Daviault the Supreme Court held that extreme intoxication verging on automatism could provide a
defence to even general intent offences because it would undermine the voluntariness of the act and it would
be unconstitutional to substitute the act of becoming intoxicated for the basic act and mind of the offence.
The Court indicated that the defence would be rare and would have to be established by the accused with
expert evidence and established on a balance of probabilities but that it could be applied with respect to
general intent offences such as assault and sexual assault. The theory behind the defence is that a person
can become intoxicated enough that his mind may cease to operate sufficiently to make conscious choices
relating to his actions. Scientifically, the premise that this can happen is controversial, although Daviault
recognized that if this were to occur the Charter would require an acquittal since voluntariness is a principle
of fundamental justice. Daviault was so controversial that Parliament immediately enacted s. 33.1 of the
Criminal Code to eradicate the defence in sexual offence and violence cases. This means that, subject to
Charter challenge [Canadian courts are split on whether s. 33.1 is constitutionally valid] extreme intoxication
can only be used for other kinds of offences. Be aware that nothing in s. 33.1 abolishes the defence of simple
intoxication – it limits only the defence of extreme intoxication.

For further information on the above, refer to: Criminal Code, section 33.1; Roach, pages 255-277.

R. V. DAVIAULT, [1994] 3 S.C.R. 63


RATIO: Voluntary intoxication can act as a defence in crimes of general intent only if the intoxication
was such that the person was in a state of automatism. HOWEVER, note that this result has been
NEGATED by 33.1 (if the offense involves an interference with bodily integrity).

FACTS: Accused charged with sexual assault of elderly women. Consumed 8 bottles of beer and 40
of brandy. Blood alcohol was 0.4-0.6. Medical experts testified that at that level of drunkenness he
should be dead. If a person didn’t die of this, it would lead someone to be in a dissociative state.

67 Criminal Law and Procedure | FLSC/NCA


ISSUE: Does voluntary intoxication to a point that closely resembles automatism act as a defence for
crimes of general intent?

DECISION: Extreme intoxication is a defence for general intent offences.

REASONS: Substitution analysis from Bernard rejected. Burden of proof on the accused to establish
this “intoxication akin to automatism” on a balance of probabilities. Expert evidence will normally be
required.

DISSENT: They would have allowed the substitute MR because getting that drunk is so reckless.

R. V. BOUCHARD-LEBRUN, 2011 SCC 58


RATIO: Section 33.1 applies where three conditions are met:

1. the accused was intoxicated at the material time;


2. the intoxication was self‑induced; and
3. the accused departed from the standard of reasonable care generally recognized in
Canadian society by interfering or threatening to interfere with the bodily integrity of another
person.

Where these three things are proved, it is not a defence that the accused lacked the general intent or
the voluntariness required to commit the offence.

Section 33.1 Cr. C. therefore applies to any mental condition that is a direct extension of a state of
intoxication.

FACTS: Accused assaulted two individuals while in a state of toxic psychosis resulting from the
voluntary consumption of drugs.

ISSUE: Whether toxic psychosis whose symptoms are caused by state of self‑induced intoxication
can be “mental disorder”. Whether s. 33.1 of Criminal Code limits scope of defence of not criminally
responsible on account of mental disorder.

DECISION: Appeal dismissed.

REASONS: There is no threshold of intoxication beyond which s. 33.1 Cr. C. does not apply to an
accused, which means that toxic psychosis can be one of the states of intoxication covered by this
provision. It is so covered in the case at bar. The Court of Appeal therefore did not err in law in
holding that s. 33.1 Cr. C. was applicable rather than s. 16 Cr. C.

DEFENCE OF THE PERSON


The self-defence provisions in the Criminal Code were amended in March of 2013, to replace defences which
were widely seen as excessively technical and badly drafted.

The new provisions are discussed in Roach, pp. 324-41. In addition, R. v. Lavallee, [1990] 1 SCR 852
discusses the concept of "reasonable belief" in the context of self-defence, and should still be applicable to
the new provision.

For further information on the above, refer to: Roach, pages 324-341.

SELF DEFENCE (s. 34)

Defence of the Person | Select Criminal Defences 68


 Affirmative defence
 AR and MR are there but shouldn’t be punished.
 Classic case of justification.
 Results in acquittal
 At CL, you can respond proportionately to a threat to your safety and you have a defence of self defence.
Burden of Proof Crown prove elements of offence beyond a reasonable doubt once an air of reality is established.
s. 34 of CCC 34. (1) A person is not guilty of an offence if
(passed on Nov 22, (a) they believe on reasonable grounds that force is being used against them or another person
2012) or that a threat of force is being made against them or another person;
 Subjective element

(b) the act that constitutes the offence is committed for the purpose of defending or protecting
themselves or the other person from that use or threat of force; and

(c) the act committed is reasonable in the circumstances.

 Basically, if you reasonably believe that the threat of force is being applied to you, you
can respond to that in a reasonable fashion.

34(2) In determining whether the act committed is reasonable in the circumstances, the court
shall consider the relevant circumstances of the person, the other parties and the act, including,
but not limited to, the following factors:

(a) the nature of the force or threat;


(b) the extent to which the use of force was imminent and whether there were other means
available to respond to the potential use of force;
(c) the person’s role in the incident;
(d) whether any party to the incident used or threatened to use a weapon;
(e) the size, age, gender and physical capabilities of the parties to the incident;
(f) the nature, duration and history of any relationship between the parties to the incident,
including any prior use or threat of force and the nature of that force or threat;
(f.1) any history of interaction or communication between the parties to the incident;
(g) the nature and proportionality of the person’s response to the use or threat of force; and (h)
whether the act committed was in response to a use or threat of force that the person knew was
lawful.

34(3) Subsection (1) does not apply if the force is used or threatened by another person for the
purpose of doing something that they are required or authorized by law to do in the
administration or enforcement of the law, unless the person who commits the act that constitutes
the offence believes on reasonable grounds that the other person is acting unlawfully.
1. Does the accused believed (subjective) on reasonable grounds (objective) that force or threat
of force is being used against them or another person? (34(1)(a))

2. Was the act committed for the purpose of defending or protecting the accused or the other
person from that use or threat of force? (34(1)(b))

3. Was the act committed reasonable in the circumstances? (34(1)(c))


a. 34(2) In determining whether the act committed is reasonable in the
circumstances, consider the relevant circumstances of the person, the other parties
and the act, including, but not limited to, the following factors:

69 Criminal Law and Procedure | FLSC/NCA


(a) the nature of the force or threat;
(b) the extent to which the use of force was imminent and whether there were other means
available to respond to the potential use of force;
(c) the person’s role in the incident;
(d) whether any party to the incident used or threatened to use a weapon;
(e) the size, age, gender and physical capabilities of the parties to the incident;
(f) the nature, duration and history of any relationship between the parties to the incident,
including any prior use or threat of force and the nature of that force or threat;
(f.1) any history of interaction or communication between the parties to the incident;
(g) the nature and proportionality of the person’s response to the use or threat of force; and
(h) whether the act committed was in response to a use or threat of force that the person knew
was lawful.

34(3) Subsection (1) does not apply if the force is used or threatened by another person for the
purpose of doing something that they are required or authorized by law to do in the
administration or enforcement of the law, unless the person who commits the act that constitutes
the offence believes on reasonable grounds that the other person is acting unlawfully.

R. V. LAVALLEE, [1990] 1 S.C.R. 852


RATIO: Sometimes the jury needs help (from experts) in understanding what is reasonable or not in
circumstances that are foreign to them. Broadens the scope of factors and better contextualizes
experiences of self-defence.

FACTS: 22 year old accused had a long history of abuse from the deceased. Killed him as he was
leaving the room after a fight and threat to “get you”. Trial judge allowed evidence of battered
spouse syndrome and acquitted by jury. CA orders a new trial on grounds that expert evidence should
not have been allowed.

ISSUE: Can you use psychiatric evidence in determining if a defense possess an air of reality? Can
you act in anticipation of violence? How to take gender and lived reality into account when we assess
the reasonable person?

DECISION: Acquittal restored.

REASONS: Wilson –Expert evidence is permissible to give the jury information that it won’t otherwise
have because of the specialised nature of this experience. Battered spouse syndrome may not be
something that can be appreciated without the assistance of expert evidence. Expert evidence
needed to show why a person in that situation may not see leaving or seeking help as a reasonable
alternative.

PRECEDENT: Subsequent cases focus on same thing – enhancing understanding of jurors about
what is reasonable in that situation.

CRITIQUE: Syndromization of women’s experience; women’s experiences must be included in the


objective standard of a reasonable person and have historically been ignored; battered women who
are not the s/t weak, frail, helpless woman not as likely to make use of this defence; defence isn’t
very often successful anyway.

NECESSITY
Necessity | Select Criminal Defences 70
The defence of necessity permits the conduct of the accused to be excused where its elements are met. The
defence is heavily circumscribed.

For further information on the above, refer to: Roach, pages 353-362.

NECESSITY
Summary  Affirmative defence (AR and MR are present)
 Classic excuse defence (although Wilson J in Perka says should be available for justification
 Results in acquittal
Burden of Proof Crown prove elements of offence beyond a reasonable doubt once an air of reality is established.
Legal 1. The situation must be urgent and the peril must be imminent
Requirements for  Modified objective:
Necessity (After o “The accused person must, at the time of the act, honestly believe (subjective), on
Perka) reasonable grounds (objective), that he faces a situation of imminent peril that leaves
no reasonable alternative open.”

2. Compliance with the law must be demonstrably impossible.


 That is, there must be no legal way out. No reasonable legal alternative. (Latimer)

3. There must be proportionality between the harm inflicted and the harm avoided.
 Objective
o Does the community view the harm avoided as at least comparable to the harm
inflicted?
 The two harms must, at minimum, be of comparable gravity. (Latimer)
Exception  Fact that you’re doing something illegal at the time does not disentitle you from raising
defence, unless you created the circumstances of necessity. (E.g competing keg parties;
fasting man who did B&E)

PERKA V. THE QUEEN. [1984] 2 S.C.R. 23235

RATIO: Recognized the defence of necessity. Explicitly references it and gives criteria but classifies it
as an excuse alone.

FACTS: Perka had been sailing boat in international waters. Boat had 33½ tons of weed (~6-
7million). Massive engine trouble and storm put him and everyone else in imminent peril. Redirected
ship and landed on Vancouver island. Charged with possession for trafficking. He says storm was
only reason came to Canada and therefore the reason he broke Canadian law was because he was
at risk of death because of storm – committed offence in your jurisdiction because of necessity. Trial
acquitted him but then new trial ordered.

ISSUE: When is the defence of necessity available?

DECISION: Acquitted on procedural issues. Never actually dealt with necessity with respect to his
acquittal.

REASONS: Dickson C.J. distinguishes between justifications and excuses. Justifications: actions that
are often “praised, as motivated by some great or noble object and therefore punishment often
seems incompatible with the social approval bestowed on the doer. (RBHS 885). Excuses are still

35 Not in syllabus.

71 Criminal Law and Procedure | FLSC/NCA


viewed as wrongful acts but done in circumstances or for reasons that the law appreciates and
therefore doesn’t support punishment. No vindication of the deed (ie. didn’t do right thing) but
recognize that there was a frailty in human nature that led you to break the law. Majority recognizes
defence of necessity but classifies it only as an EXCUSE.

DISSENT: Wilson J – Necessity defence should sometimes be available as a justification. (e.g. parent
has to trespass to protect a child who is in danger) Conflicting legal duties exist, but you’re not a
weak person for taking care of your child.

PRECEDENT: First Canadian case that recognized existence of the common law defense of necessity.
Case wasn’t disposed of on necessity.

R. V. LATIMER, [2001] 1 S.C.R. 3


RATIO: Latimer redefines the test. Must be no legal alternative, must be imminent peril, and the
crimes must be proportionate.

FACTS: Tracey Latimer suffered from severe form of cerebral palsy, subject to frequent seizures. Anti-
seizure meds prevented her from taking pain killers and she was also subject to surgeries. Facing a
surgery to remove her upper thigh bone. Latimer concludes that rather than face final surgery, he
should end her life. Puts her into cab of truck and kills her. Tells police and charged with first degree
murder. At first trial, convicted of second degree murder by jury. New trial ordered because Crown
tampered with jury pool. Second trial tried for second degree murder. Defence argued necessity. Trial
judge decided not to put defence to jury because said no air of reality. Jury asks if they convict him, if
they can have input into sentencing. Judge tells them not to concern themselves with punishment.
But may be some input into sentencing. If parole ineligibility should exceed 10 years. So jury comes
back and finds him guilty and then judge says min is life in prison. Jury upset and says parole
eligibility should be 1 year, even though min is 10-25. Judge uses Charter to do a constitutional
exemption (no longer exists). SCC concludes there is no exemption to mandatory minimum.

ISSUE: Did the trial judge correctly conclude there was no air of reality to the defence of necessity?

DECISION: Guilty. Defence of necessary is not available.

REASONS: No proportionality – infliction of death for ease of pain.

PRECEDENT: Some indications that murder may never be met under defence of necessity.

DURESS
The defence of duress is available under section 17 of the Criminal Code and at common law. Section 17
identifies a limited defence, but the common law and Charter have been used to extend its application.

For further information on the above, refer to: Roach, pages 362-380.

DURESS (s. 17 and CL)


Summary  Affirmative defence
 AR and MR are there but shouldn’t be punished.
 Excuse
 Results in acquittal

 CL of duress applies to all situations EXCEPT where the accused is a principal

Duress | Select Criminal Defences 72


(Paquette), accused of one of the 22 excluded offences under s. 17 (Ruzic).
 Principals who commit any other offence are dealt with under the CL of duress (most).
 If you are a principal offender and accused of one of the excluded offences, you cannot claim
the defence of duress.
 Jury should be entitled to decide whether accused is a principal or a secondary offender. If
conclude that secondary, then decide if person has a defence of duress at common law.
(Mena)
Burden of Proof Crown prove elements of offence beyond a reasonable doubt once an air of reality is established.
Is the accused the YES  s. 17  Is it one of the 22 excluded offences? Yes  No defence of duress
principal offender? No  Still under s. 17 but same as CL
defence of duress (Paquette; Ruzic)
NO  Common law defence of duress
 Non principal offenders and principals outside of 22 offences go through CL defence of
duress
Elements of CL of 1. The accused must be subject to a threat of death or serious injury to himself or to another
Duress person (and though it doesn’t have to be immediate, there must be a close temporal
connection between the threat and its execution (Ruzic))

2. The accused must be committing the offence exclusively as a product of the threat – (that
is, must believe that the threat will be carried out if s/he fails to commit the offence)

3. The threat must be of such gravity that it may well cause a reasonable person placed in the
same situation as the accused to respond to the threat by committing the offence in question.
(subjective/objective – ordinary person in same circumstances)

4. The accused must have a reasonable belief in no reasonable avenue of escape


(objective/subjective – Ruzic; using the context of Belgrade); and…

5. The accused must not be a voluntary member of a criminal association whereby he or she
knew that s/he may be subject to compulsion by threats.
s. 17 of CCC A person who commits an offence under compulsion by threats of immediate death or bodily
harm from a person who is present when the offence is committed is excused for committing the
offence if the person believes that the threats will be carried out and if the person is not a party to
a conspiracy or association whereby the person is subject to compulsion, but this section does not
apply where the offence that is committed is…”

Three Restrictions:
1. The threat has to be of immediate bodily harm (no force or effect, Ruzic)
2. The person posing the threat has to be present (no forece or effect, Ruzic)
3. 22 offences are excluded, including murder, sexual assault, robbery and arson. (“excluded
offences”)
 s. 17 applies to principals only (Paquette)

 Consider making a constitutional argument that those offences. Likely if someone is charged
with one of those offences and has a good duress claim, they will challenge that restriction.

R .V. RYAN, 2013 SCC 3


RATIO: Duress is available when one is compelled to commit a crime against an innocent third party.

73 Criminal Law and Procedure | FLSC/NCA


FACTS: Nicole Doucet Ryan (now Nicole Doucet) alleged that she was subject to repeated abuse and
torment by her husband, Michael Ryan. At trial, the trial judge accepted she was subject to such
abuse. The husband was never called to testify. In September 2007, Ms. Doucet began to think
about having her husband murdered. Over the course of the next seven months, she spoke to at
least three men whom she hoped would kill him. In December 2007 or January 2008, she paid one
man $25,000 to carry out the killing, but he then refused, demanding more compensation. She
approached another person and was contacted by a third, an undercover RCMP officer, posing as a
“hit man”. On March 27, 2008, she met with this individual and agreed to pay him to kill her
husband. The agreed upon price was $25,000, with $2,000 paid in cash that day. The killing was to
take place the coming weekend. Later that same night, she provided an address and a picture of her
husband to the “hit man.” Shortly after, she was arrested and charged with counselling the
commission of an offence not committed contrary to s. 464(a) of the Criminal Code, R.S.C. 1985, c.
C-46.

At trial, there was no issue that the elements of the offence had been proved and the trial judge,
Farrar J. (as he then was), indicated that he was satisfied beyond a reasonable doubt that the
requisite elements of the offence of counselling the commission of an offence had been established.
He based this conclusion on the Ms. Doucet’s admission that the Crown had proved a prima facie
case and on the audio and video tapes of recorded conversations with the undercover officer and a
statement made on arrest. The only issue at trial was whether Ms. Doucet’s otherwise criminal acts
were excused because of duress. The accused had raised that the common law defence of duress
applied. The Crown argued that on the facts of this case, the components of duress were not present.
But it did not argue at trial, as it did later on appeal, that the defence of duress was not available in
law to the accused. The trial judge accepted her version and acquitted her on the basis she had
established she was acting under duress.

The Nova Scotia Court of Appeal unanimously upheld her acquittal.

ISSUE: Availability of duress in the context of a domestic violence.

DECISION: The Court unanimously allowed the Crown appeal. The majority entered a stay while the
dissenting judge, Fish J., would have ordered a new trial, leaving it to the Crown to determine
whether a retrial was in the public interest. The Court accepted the facts found by the trial judge.

REASONS: In this case, given the facts found by the trial judge, the husband would not be an
innocent victim. Rather he would be the author of his own misfortune. Moreover Ms. Doucet was
never compelled to act as she did. The Court alluded to the possibility of invoking self-defence as a
possible defence.

Fish J. found the granting of the stay of proceedings was inappropriate. He would have ordered a new
trial. Any further defence advanced by the accused could be made then.

REACTIONS: Following the release of the decision on January 18, 2013, Michael Ryan, ex-husband of
the accused denied any of the allegations made. He emphasized he was in attendance at court in
response to a subpoena, but the Crown never called him as a witness 36.

A law professor from Dalhousie University called for a public inquiry. He questioned the rationale of
the Crown not to call Mr. Ryan as a witness. He also was critical of the Court in making findings

36 http://www.cbc.ca/news/canada/nova-scotia/story/2013/01/23/ns-hitman-michael-ryan.html

Duress | Select Criminal Defences 74


against the RCMP without having heard anything from the RCMP. He was also critical of the granting
of a stay of proceedings, agreeing with Fish J. that it is an extreme remedy 37.

PROVOCATION
The defence of provocation applies solely to the offence of murder. It is a partial defence, reducing conviction
to manslaughter where its elements are met.

For further information on the above, refer to: Roach, pages 405-427.

PROVOCATION (s. 232)


What type of  Affirmative defence
defence?  AR and MR are there but shouldn’t be punished.
 Only a defence to murder. It does not apply to any other crime.
 Partial defence. If accepted, it reduces charge from murder to manslaughter
Burden of Proof Crown prove elements of offence beyond a reasonable doubt once an air of reality is established.
s. 232 of CCC 232. (1) Culpable homicide that otherwise would be murder may be reduced to manslaughter if
the person who committed it did so in the heat of passion caused by sudden provocation.

(2) A wrongful act or an insult that is of such a nature as to be sufficient to deprive an ordinary
person of the power of self control is provocation for the purposes of this section if the accused
acted on it on the sudden and before there was time for his passion to cool.

232(3) “…no one shall be deemed to have given provocation to another by doing anything that
he had a legal right to do, or by doing anything that the accused incited him to do in order to
provide the accused with an excuse for causing death or bodily harm to any human being.”
 “Legal right” really means something “sanctioned by law, such as a sheriff proceeding to
execute a legal warrant, or a person acting in justified self-defence.” (Tran, para. 27.)
 It does not not have to be an illegal act to be a “wrongful act or insult”.
Three Elements of For a successful defence of provocation to a charge of murder, three things have to be proven:
Provocation 1. The accused was subject to a wrongful act or insult sufficient to deprive an ordinary
person of the power of self-control
 Modified objective test.
o Subjective feature: Age (Hill), gender (Hill), past relationship, racial slurs
(Thibert)
o Objective: ordinary person
 Subjective features inform the gravity of the insult, not the level of self-control we
expect of the ordinary person (Tran)
2. The accused did, in fact, lose self-control in response to this wrongful act or insult
(subjective element); and
3. The accused did so on the sudden and before there was time for his passion to cool.
 Suddenness has two components:
1) the insult or act must be sudden AND
2) the response must be sudden.
Relevance of  Nahar – leaves open door that culture could be considered; Humaid
Culture  Cultural background is not part of the ordinary person standard; would be relevant to the
insult portion
Rationale  “The law recognizes that, as a result of human frailties, the accused reacted inappropriately

37 http://www.cbc.ca/news/canada/nova-scotia/story/2013/02/05/ns-hit-man-case-inquiry-kaiser.html

75 Criminal Law and Procedure | FLSC/NCA


and disproportionately, but understandably to a sufficiently serious wrongful act or insult.”
(Tran (2010), para. 22)
 Theory is that people who act in heat of the moment are less blameworthy. (ie. excuse)
Critiques  Defence privileges the emotions of anger/rage. Don’t privilege other emotions.
 Used as a defence by men who kill their wives in murderous rage.

R. V. TRAN, [2010] 3 S.C.R. 350


RATIO: Adultery is not considered an “insult” sufficient for the ordinary person to lose self-control.

FACTS: Tran killed ex-wife’s new lover and seriously injured her after learning she had a new
boyfriend.

ISSUE: Was the new relationship an “insult” sufficient to deprive the accused of self-control?

DECISION: 2nd degree murder conviction upheld.

REASONS: No air of reality to the defence of provocation. This was not an “insult” and Tran did not
act “on the sudden”. He went there after suspecting the relationship. Only “behavior which comports
with contemporary society’s norms and values will attract the law’s compassion.” 38

ENTRAPMENT
Entrapment is a common law defence that applies even where the accused has committed a crime with the
required fault. It results in a stay of proceedings in cases where a state agent has provided the accused with
an opportunity to commit a crime without either a reasonable suspicion that the accused was involved in
crime or a bona fide inquiry into a particular type of crime in a high crime area. Alternatively even if there is a
reasonable suspicion or a bona fide inquiry, entrapment will apply and result in a stay of proceedings if the
state agent induces the commission of the crime.

For further information on the above, refer to: Roach, pages 42-44.

R. V. MACK, [1988] 2 S.C.R. 903


RATIO: A defence of entrapment is available when:

 the authorities provide a person with an opportunity to commit an offence without acting on
a reasonable suspicion that this person is already engaged in criminal activity or pursuant to
bona fide inquiries;
 although having such a reasonable suspicion or acting in the course of a bona fide inquiry,
they go beyond providing an opportunity and induce the commission of an offence.

A defence of entrapment cannot be advanced until after the crown has proven all of the elements of
the offence. Objective entrapment involving police misconduct, and not the accused's state of mind,
is a question to be decided by the trial judge, and the proper remedy is a stay of proceedings.

The following factors may be considered in determining if the police have gone further than providing
an opportunity:

1. the type of crime being investigated and the availability of other techniques for the police
detection of its commission;

38 See paragraph 30.

Entrapment | Select Criminal Defences 76


2. whether an average person, with both strengths and weaknesses, in the position of the
accused would be induced into the commission of a crime;
3. the persistence and number of attempts made by the police before the accused agreed to
committing the offence;
4. the type of inducement used by the police including: deceit, fraud, trickery or reward;
5. the timing of the police conduct, in particular whether the police have instigated the offence
or became involved in ongoing criminal activity;
6. whether the police conduct involves an exploitation of human characteristics such as the
emotions of compassion, sympathy and friendship;
7. whether the police appear to have exploited a particular vulnerability of a person such as a
mental handicap or a substance addiction;
8. the proportionality between the police involvement, as compared to the accused, including
an assessment of the degree of harm caused or risked by the police, as compared to the
accused, and the commission of any illegal acts by the police themselves;
9. the existence of any threats, implied or express, made to the accused by the police or their
agents;
10. whether the police conduct is directed at undermining other constitutional values.

This list is not exhaustive.

FACTS: The accused was once an addict but had given up narcotics. A police informer persistently
requested the accused to sell drugs over a lengthy period of time. The informer threatened the
accused, and offered large monetary inducements.

ISSUE: Whether or not stay of proceedings should issue on basis of entrapment. Manner in which
entrapment claim should be dealt with by the Courts.

DECISION: Appeal allowed. Entrapment had occurred.

REASONS: In reaching its decision, the Supreme Court of Canada acknowledged that in an
investigation for drug trafficking, traditional investigation devices cannot always be used. However, in
this case, the police went beyond merely providing the accused with the opportunity to commit an
offence. The threatening manner in which the informant acted toward the accused when he would
not become involved in drug dealing and the number of times the informer approached the accused
to become involved was unacceptable.

R. V. BARNES, [1991] 1 S.C.R. 449


RATIO: An exception to [the rule establishing entrapment] arises when the police undertake a bona
fide investigation directed at an area where it is reasonably suspected that criminal activity is
occurring. When such a location is defined with sufficient precision, the police may present any
person associated with the area with the opportunity to commit the particular offence. Such
randomness is permissible within the scope of a bona fide inquiry.

FACTS: "Buy‑and‑bust" program in area of city with trafficking problem. Accused in this area.
Undercover police officer approached accused on hunch that he would have drugs.

ISSUE: Whether appellant was subjected to random virtue‑testing.

DECISION: Appeal dismissed.

77 Criminal Law and Procedure | FLSC/NCA


REASONS: An undercover officer who attempts to buy drugs from someone suspected of selling
drugs does not constitute entrapment.

IGNORANCE OF THE LAW


The general rule, that ignorance of the law is no excuse, is reflected in s 19 of the Criminal Code. This general
principle is subject to exceptions in limited circumstances. In particular, when “colour of right” is specified to
be relevant, a mistake about the law can be relevant. Further, the Supreme Court of Canada has created the
common law defence of “officially induced error”.

For further information on the above, refer to: Criminal Code, section 19; Roach, pages 94-100.

LILLY V. THE QUEEN, [1983] 1 SCR 794


RATIO: An honest belief concerning property rights, whether based on a mistake in fact or in law,
may constitute a colour of right.

FACTS: Appellant, a licensed real estate broker, was convicted of theft of $26,759.58, being sums
deposited “in trust” with respect to real property transactions. The appellant relied, for 18 of the 21
transactions involved in the count on which he was found guilty, on the defence of colour of right
alleging he thought he could lawfully transfer the amounts from the “in trust” account to the
agency’s general account once the offer to purchase the property had been accepted. As to the other
transactions involving a total of $13,500 he relied on an alleged lack of knowledge of the transfers.
The Court of Appeal dismissed appellant’s appeal.

ISSUE: This appeal is to determine whether the trial judge misdirected the jury as to the meaning of
the phrase “colour of right”.

DECISION: Appeal allowed, new trial ordered.

REASONS: In his charge, the trial judge misdirected the jury. The fate of the appellant’s defence of
colour of right was not dependent upon the jury determining when the commissions were payable.
Rather, the defence was dependent upon whether the jurors were satisfied beyond a reasonable
doubt that the appellant had not, at the time of the transfers, an honest belief that he had the right
to that money, and not, as they were told, dependent upon what they thought his rights were.
Further, the conviction cannot stand for it was impossible to know whether the conviction stood
solely on those transactions that did not raise the defence of colour of right.

R. V. JONES, [1991] 3 SCR 110


RATIO: A mistake about the law is no defence to a charge of breaching it.

FACTS: Accused convicted of unlawfully conducting a bingo. Accused mistakingly believed that
lottery provisions in Criminal Code inapplicable to lotteries conducted on Indian reserve

ISSUE: Whether the defence of colour of right is available.

DECISION: Appeal dismissed.

REASONS: The defence of colour of right does not apply to a charge under s. 206(1)(d) of the Code.
First, no authority was cited for the proposition that colour of right is relevant to any crime which
does not embrace the concept within its definition. Second, appellants' mistake was one of law,
rather than of fact. They mistakingly believed that the law did not apply because it was inoperative
on Indian reserves. A mistake about the law is no defence to a charge of breaching it.

Ignorance of the Law | Select Criminal Defences 78


LÉVIS (CITY) V. TÉTREAULT; LÉVIS (CITY) V. 2629-4470 QUÉBEC INC.
RATIO: The court affirmed the analytical framework put forward by Lamer C.J. in Jorgensen 39. To
avail the defence of ‘officially induced error’, the accused must prove six elements:

1. that an error of law or of mixed law and fact was made;


2. that the person who committed the act considered the legal consequences of his or her
actions;
3. that the advice obtained came from an appropriate official;
4. that the advice was reasonable;
5. that the advice was erroneous; and
6. that the person relied on the advice in committing the act.

FACTS: Accused put their motor vehicle back into operation without having paid the required
registration fees. Accused claimed to have been misled by erroneous information obtained from an
official regarding the procedure for paying fees relating to registration.

ISSUE: Whether defence of officially induced error available in Canadian criminal law. If so, whether
accused establishing that conditions under which this defence available have been met.

DECISION: Appeal allowed.

REASONS: As for the defence of officially induced error, although it is available in Canadian criminal
law, the company has not established that the conditions under which it is available have been met.

THE ADVERSARIAL PROCEEDING


THE ADVERSARIAL PROCESS
As indicated, a trial is the opportunity for the Crown prosecutor to prove the specific allegation made in the
charge (information or indictment) beyond a reasonable doubt. The key characteristic of the Canadian
criminal trial is therefore the specific allegation. This is done during a trial.

For further information on the above, refer to: Coughlan, pages 347-370.

It is helpful to understand the trial process to situate what follows:

THE PRESUMPTION OF INNOCENCE AND THE ULTIMATE STANDARD OF PROOF


At a Canadian trial, the accused is presumed to be innocent, a right guaranteed by s.11(d) of the Charter. This
means that ultimately, at the end of the whole case, the Crown must prove the guilt of the accused beyond a
reasonable doubt. This is the Crown’s ultimate burden in both a criminal or regulatory prosecution. The
meaning of proof beyond a reasonable doubt is described in R. v. Lifchus.

For further information on the above, refer to: Roach, pages 50-51.

R. V. LIFCHUS, [1997] 3 S.C.R. 320


RATIO: When defining important criminal terms such as "beyond a reasonable doubt" the judge must
not simply use a plain language definition; they must include descriptions of the important

39 See paragraphs 28-35.

79 Criminal Law and Procedure | FLSC/NCA


underlying concepts of criminal law that must be considered, and the specific degree that must be
proven to be acceptable.

If a different charge is used, a verdict should only be overturned to order a new trial if there is a
reasonable likelihood that the charge led the jury to misapprehend the standard of proof.

FACTS: Lifchus was a stockbroker who was accused of fraud and theft. He was convicted of one and
acquitted of the other. He appealed on the basis that the judge did not properly explain the burden of
proof to the jury. He said that "beyond a reasonable doubt" is simply an everyday idea and that
everyone understands it - a "plain language" approach. The Court of Appeal allowed the appeal
ordering a new trial, which the Crown appealed.

ISSUE: How should a judge charge a jury on the meaning of “beyond a reasonable doubt”?

DECISION: Appeal dismissed.

REASONS: Cory, writing for the majority, agrees that this was not the correct way to describe "beyond
a reasonable doubt" to a jury, because it is not simply the plain understanding of it.

He says that other than the part in question, the trial judge's charge to the jury was clear; however,
the part in question was enough to mislead the jury. The Crown tries to argue that s.686(b)(iii)
applies, however this is rejected as the Court says that you cannot say that the jury would have come
to the same conclusion with the correct instruction.

GUIDELINE: He gives a list of things to include in a charge:

 the standard of proof beyond a reasonable doubt is inextricably intertwined with that
principle fundamental to all criminal trials, the presumption of innocence;
 the burden of proof rests on the prosecution throughout the trial and never shifts to the
accused;
 a reasonable doubt is not a doubt based upon sympathy or prejudice, rather, it is based
upon reason and common sense;
 it is logically connected to the evidence or absence of evidence;
 it does not involve proof to an absolute certainty; it is not proof beyond any doubt nor is it an
imaginary or frivolous doubt; and
 more is required than proof that the accused is probably guilty -- a jury which concludes only
that the accused is probably guilty must acquit;

and a list of things not to include:

 describing the term as an ordinary expression which has no special meaning in the criminal
law context;
 inviting jurors to apply to the task before them the same standard of proof that they apply to
important, or even the most important, decisions in their own lives;
 equating proof "beyond a reasonable doubt" to proof "to a moral certainty;
 qualifying the word "doubt" with adjectives other than "reasonable", such as "serious",
"substantial" or "haunting", which may mislead the jury; and
 instructing jurors that they may convict if they are "sure" that the accused is guilty, before
providing them with a proper definition as to the meaning of the words "beyond a
reasonable doubt".

The Adversarial Process | The Adversarial Proceeding 80


Suggested Charge40:

The accused enters these proceedings presumed to be innocent. That presumption of


innocence remains throughout the case until such time as the Crown has on the evidence
put before you satisfied you beyond a reasonable doubt that the accused is guilty.

What does the expression “beyond a reasonable doubt” mean?

The term “beyond a reasonable doubt” has been used for a very long time and is a part of
our history and traditions of justice. It is so engrained in our criminal law that some think it
needs no explanation, yet something must be said regarding its meaning.

A reasonable doubt is not an imaginary or frivolous doubt. It must not be based upon
sympathy or prejudice. Rather, it is based on reason and common sense. It is logically
derived from the evidence or absence of evidence.

Even if you believe the accused is probably guilty or likely guilty, that is not sufficient. In
those circumstances you must give the benefit of the doubt to the accused and acquit
because the Crown has failed to satisfy you of the guilt of the accused beyond a reasonable
doubt.

On the other hand you must remember that it is virtually impossible to prove anything to an
absolute certainty and the Crown is not required to do so. Such a standard of proof is
impossibly high.

In short if, based upon the evidence before the court, you are sure that the accused
committed the offence you should convict since this demonstrates that you are satisfied of
his guilt beyond a reasonable doubt.

R. V. STARR [2000] 2 S.C.R. 144


RATIO: The reasonable doubt standard falls much closer to absolute certainty than to proof on a
balance of probabilities.

FACTS: In August 1994, Bernard Cook and Darlene Weselowski were drinking with Robert Dennis
Starr in a hotel near Winnipeg. In the late hours of the night Starr parted ways with Cook and
Weselowski. Together, Cook and Weselowski were approached by Jodie Giesbrecht, a sometimes
girlfriend of Cook. During an ensuing conversation Cook told Giesbrecht that he could not go with her
that night because he had to "go and do an Autopac scam with Robert", as he had been given $500
for wrecking a car for insurance purposes.

A few hours later the bodies of both Cook and Weselowski were found on the side of a nearby
highway. They had been shot in the head. Starr was arrested in connection with the murders.

At trial, the Crown advanced the theory that the murders were gang-related, where Starr had used
the insurance fraud scam as a means to get Cook into the countryside to murder him. The case
hinged on the testimony of Giesbrecht and the statement she heard from Cook that night. The judge
found the statement admissible on the "present intentions" or "state of mind" exception to the
hearsay rule.

ISSUE: Whether trial judge explained concept of reasonable doubt to jury in adequate manner.

40 Provided by Cory at paragraph 39. It has subsequently been adopted by all trial jurors since then.

81 Criminal Law and Procedure | FLSC/NCA


DECISION: In a 5 to 4 decision the Court held that the evidence should not be admitted and sent the
case back for a retrial.

REASONS: With regard to the jury instruction, the Court held that the judge failed to properly instruct
the jury on the standard of proof. The judge should have placed “beyond a reasonable doubt”
between absolute certainty and “balance of probabilities”.

R. V. J.H.S., 2008 SCC 30


RATIO: Court affirmed R. v. W.(D.) [1991] 1 S.C.R. 742: where credibility is a central issue at trial.

The passage from W. (D.) at issue in this case, as in so many others, is found at pp. 757-58, where
Cory J. explained:

Ideally, appropriate instructions on the issue of credibility should be given, not only during
the main charge, but on any recharge. A trial judge might well instruct the jury on the
question of credibility along these lines:

1. First, if you believe the evidence of the accused, obviously you must acquit.
2. Second, if you do not believe the testimony of the accused but you are left in
reasonable doubt by it, you must acquit.
3. Third, even if you are not left in doubt by the evidence of the accused, you must ask
yourself whether, on the basis of the evidence which you do accept, you are
convinced beyond a reasonable doubt by that evidence of the guilt of the accused.

If that formula were followed, the oft repeated error which appears in the recharge in this
case would be avoided. The requirement that the Crown prove the guilt of the accused
beyond a reasonable doubt is fundamental in our system of criminal law. Every effort
should be made to avoid mistakes in charging the jury on this basic principle.

Nonetheless, the failure to use such language is not fatal if the charge, when read as a
whole, makes it clear that the jury could not have been under any misapprehension as to the
correct burden and standard of proof to apply . . . .

FACTS: The accused was convicted of sexual assault. The complainant and accused were both
principal witnesses at trial.

ISSUE: Whether trial judge adequately instructed jury on application of principles of reasonable
doubt to issue of credibility.

DECISION: Appeal allowed, conviction restored.

REASONS: In short the W. (D.) questions should not have attributed to them a level of sanctity or
immutable perfection that their author never claimed for them. W. (D.)’s message that it must be
made crystal clear to the jury that the burden never shifts from the Crown to prove every element of
the offence beyond a reasonable doubt is of fundamental importance but its application should not
result in a triumph of form over substance. In R. v. S. (W.D.), [1994] 3 S.C.R. 521, Cory J. reiterated
that the W. (D.) instructions need not be given “word for word as some magic incantation” (p. 533).
In R. v. Avetysan, [2000] 2 S.C.R. 745, 2000 SCC 56, Major J. for the majority pointed out that in any
case where credibility is important “[t]he question is really whether, in substance, the trial judge’s
instructions left the jury with the impression that it had to choose between the two versions of
events” (para. 19). The main point is that lack of credibility on the part of the accused does not
equate to proof of his or her guilt beyond a reasonable doubt.

The Adversarial Process | The Adversarial Proceeding 82


OTHER BURDENS
While the Crown prosecutor must prove guilt beyond a reasonable doubt at the end of the case, there are
other burdens of proof that operate during the criminal process. There are “evidential” burdens that some
rules of law impose in order for a party who wishes a matter to be placed in issue to succeed in having that
matter placed in issue. For example, if at the end of the Crown’s case in chief the defence argues that there
is no “case to meet” and requests a “directed verdict of acquittal” the judge will evaluate whether the Crown
has shown a prima facie case.

This is the same standard that applies where the accused is entitled to and requests a preliminary inquiry to
determine whether there is a case to answer; the preliminary inquiry judge will discharge the accused unless
the Crown can show a prima facie case. The meaning of the prima facie case is discussed in R. v. Arcuri
below.

Even the accused must at times satisfy an evidential burden in order to have a matter placed in issue.
Indeed, if the accused wants to have a defence considered, the accused must show that the defence has an
“air of reality” to it. If the accused succeeds, the judge must consider the defence, and in a jury trial must
direct the jury on the law that applies to that defence: R. v. Cinous and R. v. Fontaine illustrate this.

There are numerous rules of evidence called “presumptions” that operate to assign burdens of proof on the
accused. A presumption is a rule of law that directs judges and jury to assume that a fact is true (known as
the “presumed fact”) in any case where the Crown proves that another fact is true (known as the “basic fact”),
unless the accused can rebut the presumed fact according to the assigned standard of proof.

Those presumptions known as “mandatory presumptions” can be rebutted by the accused simply raising a
reasonable doubt about whether the presumed fact follows from the basic fact. Where a mandatory
presumption is rebutted, the “presumed fact” falls back into issue notwithstanding the presumption, and
must be proved by the Crown in the ordinary way, without the assistance of the presumption.

Other presumptions operate as “reverse onus provisions,” deeming the presumed fact to exist where the
Crown proves the basic fact unless the accused disproves the presumed fact on the balance of probabilities.
A presumption can be easily recognized as a “mandatory presumption” because the legal rule raising the
presumption will use the term “evidence to the contrary” to describe the burden of rebuttal. A presumption
will be interpreted as a “mandatory presumption” where it fails to set out the required standard of rebuttal
because of s. 25 (1) of the Intepretation Act. Many presumptions operate in alcohol driving prosecutions and
are used to determine whether the accused has more than a legal amount of alcohol in his blood while
driving or having care or control of a motor vehicle: See, for example, s. 258 (1) (a), [a reverse onus provision]
and ss. 258 (1) (c), (d.1) and (g), all mandatory presumptions. Presumptions are prima facie contrary to the
Charter and must be saved under s. 1.

For further information on the above, refer to: Roach, pages 50-58.

R. V. OAKES, [1986] 1 S.C.R. 103


RATIO: To test if a section is saved under s.1:

1. the section must fulfill an objective related to concerns which are pressing and substantial
in a free and democratic society; and
2. the means chosen must be reasonable and demonstrably justified.

FACTS: Oakes charged with unlawful possession of narcotic for purpose of trafficking. Oakes claims
s.8 of Narcotics Act violates presumption of innocence contained in s.11(d) of Charter.

83 Criminal Law and Procedure | FLSC/NCA


ISSUES: Is s.8 of the Narcotic Control Act inconsistent with s.11(d) of the Canadian Charter of Rights
and Freedoms and thus of no force and effect? If yes, is this violation of the Charter justified under
s.1?

DECISION: S.8 of the Narcotic Control Act violates the Canadian Charter of Rights and Freedoms, and
is therefore of no force or effect. S.8 imposes a limit of s.11(d) of the Charter which is not reasonable
and not demonstrably justifiable.

REASONS: He finds that the reverse onus created in the Act clearly violates the s.11(d) right to the
presumption of innocence, as it requires the accused to bear the burden of proof. He also states that
it is not exempted merely because it is a federal statute – it is still subject to the Charter.

Applying the Oakes test to the case at bar, it does not pass the rational connection test as the
possession of a small quantity of narcotics does not support the inference of trafficking - "it would be
irrational to infer that a person had an intent to traffic on the basis of his or her possession of a very
small quantity of narcotics". As a result, the section is unconstitutional and the appeal dismissed.

R. V. CINOUS, [2002] 2 S.C.R 3


RATIO: There must be evidence on each element of the defence in order for there to be an air of
reality. Only then should the defence be put to a jury. Ask: Is there evidence on the record upon which
a properly instructed jury acting reasonably could acquit? Regarding self-defence: must reasonably
believe you are subject to a threat of force and have no other means of protecting yourself.

FACTS: Accused charged with murder having shot the deceased in back on head outside a gas
station. He was in a gang and given what he knew about the situation, he became concerned that he
was the target of the killing.

ISSUES: Did Cinous have reasonable grounds to believe he was being threatened? Must an accused
be truly threatened or simply believe he was threatened?

DECISION: Guilty.

REASONS: No air of reality to the defence of self-defence. You don’t actually have to have an assault
happen – just need a “reasonable apprehension” of an assault of a threat of an assault. Here, it
doesn’t matter whether they were actually going to assault Cinous because he believed on
reasonable grounds that there was a threat of assault. There must be no other means of protecting
yourself. Here, the Court ruled that he had other options available, including calling the police and
fleeing.

DISSENT: Binnie – To concede with not calling the police and concede with reasonable criminal
rather than reasonable person is threatening for the public order. This would endorse a sociopathic
approach to dispute resolution. A reasonable person would have seen other options available.

R. V. FONTAINE, [2004] 1 S.C.R. 702


RATIO: The evidentiary burden is discharged if the accused raises sufficient evidence that there is an
"air of reality" that the defence might succeed; the judge is not to consider the truth of the evidence
when deciding if the accused has met the burden.

FACTS: See above.

ISSUE: See above.

The Adversarial Process | The Adversarial Proceeding 84


DECISION: See above.

REASONS: See above.

THE NEUTRAL IMPARTIAL TRIER


Another critical component of the adversarial system is the presence of a neutral, impartial trier of law (to
make legal decisions) and a neutral impartial trier of fact (to make factual findings at the end of the trial). In
Canada, more than 95% of all criminal trials are conducted by a judge alone, so the judge performs the role
both of the trier of law and the trier of fact. Where there is a jury trial, the judge acts as the trier of law, and
the jury as the trier of fact. This means that the judge makes all legal and procedural decisions during the
trial, and directs the jury by training them in the law that applies. The jury then makes the factual decision
and renders the holding. In Canada the appropriate sentence is a question of law, and therefore sentencing is
done by the judge and not by the jury. Indeed, the jury should not be told of the possible sentences for fear
that this will inspire a sympathetic rather than a legal verdict. Requiring the judge to remain neutral and
impartial does not require the judge to remain passive. This is especially so in the case of a self-represented
accused, where a trial judge has a duty to see to it that the accused’s fair trial right is respected. Still, the
essence of the adversarial system is that the parties initiate the proof that is brought forward, not the judge.

For further information on the above, refer to: Coughlan, pages 379-390.

R. V. GUNNING, [2005] 1 S.C.R 627


RATIO: It is for the judge to decide all questions of law and to direct the jury accordingly; but the jury,
who must take its direction on the law from the judge, is the sole arbiter on the facts. The judge also
has the duty, insofar as it is necessary, to assist the jury by reviewing the evidence as it relates to the
issues in the case. The judge is also entitled to give an opinion on a question of fact and express it as
strongly as the circumstances permit, so long as it is made clear to the jury that the opinion is given
as advice and not direction.

Subject to one exception, it is also the exclusive domain of the jury to determine the verdict. An
exception lies where the judge is satisfied that there is no evidence upon which a properly instructed
jury could reasonably convict, in which case, it is the judge’s duty to direct the jury to acquit the
accused. This exception is made in order to safeguard against wrongful convictions. However, there
is no corresponding duty or entitlement to direct a jury to return a verdict of guilty. As Lord Devlin so
aptly put it in Chandler v. Director of Public Prosecutions, [1964] A.C. 763 (H.L.), in answer to the
argument that it should be open to the judge to direct a verdict of guilty in certain circumstances:

With great respect I think that to be an unconstitutional doctrine. It is the conscience of the
jury and not the power of the judge that provides the constitutional safeguard against
perverse acquittal. [pp. 803‑4]41

FACTS: See above.

ISSUE: See above.

DECISION: See above.

REASONS: See above.

R. V. HAMILTON , [2004] O.J. NO. 3252 (ONT. C.A.)

41 See paragraphs 27-28.

85 Criminal Law and Procedure | FLSC/NCA


RATIO: Caution is to be exercised before a trial judge can introduce their own issue.

FACTS: H and M pleaded guilty to importing cocaine. H, a 26-year-old black single mother with no
criminal record, agreed to act as a courier for compensation. She travelled to Jamaica and, before
returning to Canada, swallowed pellets containing 349 grams of cocaine with a street value of about
$70,000. She had no financial interest in the cocaine and no involvement in its proposed
distribution. H had a grade nine education and few employable skills, and was unemployed at the
time of sentencing. M was a black single mother, aged 31 at the time of the offence, with no
criminal record. Before returning to Canada from a trip to Jamaica, she swallowed pellets containing
489 grams of cocaine. At the time of sentencing, M held a low-paying job in the fast food industry.
Both accused were remorseful. The trial judge found that the accused should receive conditional
sentences.

The trial judge introduced hundreds of pages of his own research into societal ills into the
proceedings, and relied upon these materials in support of his conclusion that the accused, because
of their race, gender and poverty, were particularly vulnerable targets to those who sought out
individuals to act as cocaine couriers. He made those findings based on material he had produced
during the sentencing hearing and his own experiences as a judge. The trial judge found that the
personal responsibility of the accused for their crimes was significantly diminished by the effects of
systemic racial and gender bias. In his view, society had to take its share of the responsibility for the
crimes of the accused, and should do so through a mitigation of the penalty imposed on the
accused. He also concluded children would essentially be orphaned if their mothers were jailed. H
received a conditional sentence of 20 months on terms that provided for partial house arrest in the
first year and a curfew for the remainder of the sentence. M received a conditional sentence of two
years less a day on terms that provided for partial house arrest in the first 15 months of the
sentence and a curfew for the rest of the sentence. The Crown appealed.

ISSUE: Did the trial judge err by going beyond the appropriate role of judge?

DECISION: Appeal dismissed.

REASONS: The trial judge assumed the combined role of advocate, witness and judge. He unilaterally
decided to use the sentencing proceedings to raise, explore and [page3 a]ddress various issues
which he believed negatively impacted on the effectiveness and fairness of current sentencing
practices as they related to some cocaine importers. Through his personal experience and personal
research, he became the prime source of information in respect of those issues. He also became the
driving force pursuing those issues during the proceedings.

THE ROLE OF THE PROSECUTOR


The prosecutor is an advocate, but also a quasi-judicial officer. This means that the prosecutor cannot act
solely as an advocate, but must make decisions in the interests of justice and the larger public interest,
including the interests of the accused. The prosecutor has many discretionary decisions that can be made
and should act as a “minister of justice.”

For further information on the above, refer to: Section 5.1, The Lawyer as Advocate in Chapter 5 -
Relationship to the Administration of Justice, Federation of Law Societies of Canada, Model Code Of
Professional Conduct.

BOUCHER V. THE QUEEN, [1955] S.C.R. 16

The Adversarial Process | The Adversarial Proceeding 86


RATIO: It is improper for counsel for the Crown or the defence to express his own opinion as to the
guilt or innocence of the accused The right of the accused to have his guilt or innocence decided
upon the sworn evidence alone uninfluenced by statements of fact by the Crown prosecutor is one of
the most deeply rooted and jealously guarded principles of our law

FACTS: The appellant was found guilty of murder. His appeal to the Court of appeal was unanimously
dismissed. He now appeals to this Court, by special leave, on grounds of misdirection with reference
to reasonable doubt, circumstantial evidence and inflammatory language used by Crown counsel in
his address to the jury.

ISSUE: Was the appellant deprived of a trial according to law by reason of the fact that the crown
counsel used inflammatory language in his address to the jury?

DECISION: Appeal allowed, conviction quashed, new trial ordered.

REASONS: Crown counsel exceeded his duty when he expressed in his address by inflammatory and
vindictive language his personal opinion that the accused was guilty and left with the jury the
impression that the investigation made before the trial by the Crown officers was such that it had
brought them to the conclusion that the accused was guilty.

KRIEGER V. LAW SOCIETY OF ALBERTA, [2002] 2 S.C.R. 372


RATIO: The Court should not interfere with prosecutorial discretion. The Law Society retains
jurisdiction.

FACTS: Krieger was a Crown prosecutor who was subject to a complaint to the Law Society. He
argued that the Law Society could not review the matter since it would interfere with the exercise of
prosecutorial discretion.

ISSUE: What is the role of the office of the Attorney General and its agents in relation to private
lawyers?

DECISION: Appeal allowed, trial judgment restored.

REASONS: Because Crown prosecutors must be members of the Law Society, they are subject to the
Law Society’s code of professional conduct, and all conduct that is not protected by the doctrine of
prosecutorial discretion is subject to the conduct review process. As the disclosure of relevant
evidence is not a matter of prosecutorial discretion but rather a legal duty, the Law Society
possesses the jurisdiction to review an allegation that a Crown prosecutor acting dishonestly or in
bad faith failed to disclose relevant information, notwithstanding that the Attorney General had
reviewed it from the perspective of an employer. A clear distinction exists between prosecutorial
discretion and professional conduct, and only the latter can be regulated by the Law Society.

R. V. NIXON, 2011 SCC 34


RATIO: Subject to the abuse of process doctrine, supervising one litigant’s decision-making process —
rather than the conduct of litigants before the court — is beyond the legitimate reach of the court.

FACTS: The accused drove her motor home through an intersection and struck another vehicle,
killing a husband and wife and injuring their young son. The accused was charged with dangerous
driving causing death, dangerous driving causing bodily harm and parallel charges for impaired
driving. The Crown and the accused entered into a plea agreement, which was subsequently
repudiated by the Crown.

87 Criminal Law and Procedure | FLSC/NCA


ISSUE: Whether the act of repudiation is a matter of tactics or conduct before the court, or a matter
of prosecutorial discretion. Whether the act of repudiation is reviewable on the grounds of abuse of
process.

DECISION: Appeal dismissed.

REASONS: The Crown’s decision in this case to resile from the plea agreement and to continue the
prosecution clearly constituted an act of prosecutorial discretion subject to the principles set out in
Krieger: it is only reviewable for abuse of process.

THE ROLE OF THE DEFENCE


The defence counsel is an officer of the court, and therefore must be respectful and honest with the court
and must not attempt to mislead the court as to the state of the law. Subject to this and the rules of law and
ethics, the defence counsel is obliged to act solely in the interests of the accused, advising the accused on
the implications, and propriety, of pleading guilty, securing advantage of all procedural and constitutional
protections available to the accused that are not properly waived; and if the accused pleads not guilty,
preparing the case fully, challenging the sufficiency of prosecutorial evidence, and advancing all defences
that properly arise.

For further information on the above, refer to: Section 5.1, The Lawyer as Advocate in Chapter 5 -
Relationship to the Administration of Justice, and Section 3.3, Confidentiality in Chapter 3, Relationship To
Clients, in Federation of Law Societies of Canada, Model Code Of Professional Conduct.

GETTING TO THE TRIAL: THE CRIMINAL INVESTIGATION


POLICE POWERS
Police officers are independent of the Crown prosecutor in Canada. This independence is important to permit
the prosecutor to act as a quasi-judicial officer, and not get too close to the mind-set of an investigator. Still,
the police will often seek legal advice from Crown prosecutors, including on the wording of search warrants
and the like. In the interests of securing liberty, the powers of the police are constrained by law, although can
be derived from statute, common law and by implication from statute and common law.

Police powers are also significantly limited by the Charter, most significantly s. 8 (unreasonable search or
seizure) and s. 9 (arbitrary detention). Courts have undertaken a careful balancing of police powers in an
attempt to ensure respect for liberty, without undermining the effectiveness of police investigations and law
enforcement. The law of evidence supports limits on police powers. Although not covered in this examination,
individuals have the right to remain silent in their dealings with the police, what they say cannot be admitted
if it is not “voluntary.” Where there has been an unconstitutional search or arbitrary detention, evidence that
has been obtained as a result may be excluded from consideration.

For further information on the above, refer to: Coughlan, pages 8-24 (general police powers), 63-140 (search
& seizure), 141-157 (detention).

SEARCH AND SEIZURE


POLICE POWERS
When dealing with 1. By what legal authority did the office act? (i.e. statutes)
exercise of police 2. Did the officer comply with the legal authority?

Police Powers | Getting to the Trial: The Criminal Investigation 88


powers, ask: (Collins) 3. Even if so, is the legal authority constitutionally sound?
Section 8 “Everyone has the right to be secure against unreasonable search or seizure.”
REASONABLE EXPECTATION OF PRIVACY
1. Is there a reasonable  Section 8 is only engaged when the accused had a REP. If no REP, then
expectation of privacy? no protection of s. 8 of Charter.
 REP determined on the “totality of the circumstances” (Edwards)

Considerations include:
1. Type of privacy interest at stake (i.e. personal, territorial, informational
privacy – Tessling)
2. Whether the accused has a subjective expectation of privacy and whether
that expectation is objectively reasonable. (Patrick)
3. For property or territorial searches, factors to consider may include
(Edwards):
 Presence at the time of the search
 Possession of control of the property or place searched
 Ownership of the property or place (Belnavis, Edwards)
 Historical use of the property or item
 Ability to regulate access, including the right to admit or exclude others
from the place
REASONABLE EXPECTATION OF NO REASONABLE EXPECTATION OF
PRIVACY PRIVACY
 Your own bodily samples (Stillman)  Garbage bags left for collection at the edge of
 Entry to your home (Feeney) one’s property (Patrick)
 Business records in your place of business  The exterior heat signature of your house
(Hunter v Southam) (Tessling)
 Closed areas of your car when you are the  A girlfriend’s apartment (Edwards)
owner/operator (Blenavis; Harrison)  Regulatory seizure of documents (Thomson
 Rented locker at a bus depot (Buhay) Newspaper)
 A public even in a private hotel room  The contents of a car when one is a passenger
(Wong) (Belnavis)
 Private conversations (Duarte)  Sexual activity in a public washroom(LeBeau and
 Bags, purses, pockets (Grant) Lofthouse)
 Sniffer dog searches of a backpack in a
school or public place (A.M. and Kang-
Brown)
2. Based on this assessment, is there a REP?
If no REP  Section 8 is not engaged and the search is reasonable
If a REP exists  Section 8 is engaged. Determine if the search conducted was reasonable.
REASONABLE SEARCH
In order to be 1. It must be authorized by law
reasonable under s. 8,  Searches incident to investigative detention, dog sniff searches and
a search or seizure searches incident to arrest are common law powers
must satisfy three  Other search powers are derived by statute.
preconditions: 2. The law itself much be reasonable
(Collins) 3. It must be carried out in a reasonable manner
**This is the broader framework for assessing reasonableness under s. 8. Apply

89 Criminal Law and Procedure | FLSC/NCA


to all searches regardless of the context. Hunter v Southam, etc only apply in
criminal and quasi-criminal cases (provides specific ingredients for #2 above).
When is a search  Warrantless searches presumptively breach s. 8 (Hunter v Southam)
reasonable? (Hunter v  When a REP exists, a search is reasonable when:
Southam) 1. Prior authorization from an impartial judicial or quasi-judicial body
(ie. a warrant)
 Standard: reasonable and probable grounds to believe that the proposed
search or seizure will produce evidence of a crime. (Reasonable
suspicion standard sometimes used for less intrusive searches)
 EXCEPTION: Exigent circumstances (e.g. to protect officer safety or
destruction of evidence)
2. There are reasonable and probable grounds, established under oath, to
believe that an offence has been committed and that there is evidence to
be found at the place of the search

3. Conducted reasonably and within the scope of the warrant


3. Assuming a REP  Apply Hunter v Southam:
exists, was the search  When a REP exists, a search is reasonable when:
reasonable? 1. Warrant? No  presumptively unreasonable. Exigent circumstance that
made it impossible to get warrant?
2. Reasonable & probable grounds?
3. Conducted reasonable w/i warrant?

 If all three of the above do not exist, it may still be a legal search if you can
prove the search was reasonable under the circumstances.
o Does it fit into another category of search (e.g. search incident to
arrest, sniffer dog)?
4. Did the Sniffer dog searches (Kang-Brown, A.M.)
circumstances allow  These types of searches are reasonable w/o a warrant if they are based on
for a lower standard of reasonable suspicion (rather than reasonable and probable grounds)
suspicion than Hunter  Seen as being less intrusive and undertaken in response to on-the-spot
v Southam? (Kang- observations. Sniffer dogs are also highly accurate.
Brown, A.M.)  Search must still be conducted reasonably (ie. dog must be properly trained)

What constitutes “reasonable suspicion”?


 “Suspicion” is an expectation that targeted individual is possibly engaged in
criminal activity. “Reasonable” suspicion means something more than
“mere” suspicion and something less than belief based on reasonable and
probable grounds. (Binnie – Kang-Brown)
Requires police officers subjective belief to be backed up by objectively
verifiable indications”
Was there reasonable YES  No Charter breach
suspicion? NO  Breach of s. 8
5. If search IS No Charter breach
reasonable 
5. If search IS NOT Breach of s. 8. Should the evidence be excluded?
reasonable
7. Was there YES  No Charter breach
reasonable suspicion? NO  Breach of s. 8
should the evidence  Apply Grant exclusion of evidence test.

Police Powers | Getting to the Trial: The Criminal Investigation 90


obtained be excluded
under s. 24(2)?

DETENTION
INVESTIGATIVE DETENTION
History/Context General Rule, Pre-Charter and Pre-Mann: Short of arrest, the police never possessed
legal authority at common law to detain anyone against his or her will for questioning or
to pursue an investigation.
Mann: Middle ground emerged bw liberty and arrest; CL police power of investigative
detention.
Clayton: General detention power when “reasonably necessary” in circumstances.
 Progression to more expansive and intrusive police powers has taken place in
Charter-era through expansive reading of the common law rather than legislation.
Generally weakening protection for civil liberties.
1. Was the person  Detention under ss. 9 and 10 of the Charter refers to a suspension of the individual's
detained? To determine, liberty interest by a significant physical or psychological restraint. (Grant)
apply the Grant test.
(applied in Suberu) TEST FOR PSYCHOLOGICAL DETENTION:
1. Psychological detention is established either where the individual has a legal
obligation to comply with the restrictive request or demand, or a reasonable person
would conclude by reason of the state conduct that he or she had no choice but to
comply.
 Test: objective reasonable person
 You are NOT detained when police are simply gathering information or requesting
assistance (Suberu)

2. Where no physical restraint or legal obligation, to determine whether the reasonable


person in the individual's circumstances would conclude that he or she had been deprived
by the state of the liberty of choice, the court may consider:
a) The circumstances giving rise to the encounter as would reasonably be perceived
by the individual. (Consider: whether the police were providing general assistance;
maintaining general order; making general inquiries regarding a particular
occurrence; or, singling out the individual for focussed investigation).
b) The nature of the police conduct (Consider: language used; the use of physical
contact; the place where the interaction occurred; the presence of others; and the
duration of the encounter).
c) The particular characteristics or circumstances of the individual where relevant,
(Consider: age; physical stature; minority status; level of sophistication.)
If NO Detention No Charter breach
2. If DETENTION, what 1. Common Law Power of Investigative Detention (Mann)
are the police entitled to do  Police may detain an individual for investigative purposes if there are reasonable
once a person is detained? grounds to suspect on the totality of the circumstances that the individual is
connected to a particular crime, and that such a detention is necessary.
 Detention must be brief and no duty to answer questions of police.

2. Common Law Power to Search Incident to Investigative Detention (Mann)


 A search incident to investigative detention must be a “protective” search:
a) only a pat-down search
b) must be a search for weapons in order to protect officer or public safety

91 Criminal Law and Procedure | FLSC/NCA


c) Only permissible when there are reasonable grounds to believe that safety
at risk
d) Must be conducted in a reasonable manner

3. General Detention Power (Clayton)


 Police have a common law power to detain an individual so long as detention is
reasonably necessary in the totality of the circumstances.
 To determine whether a detention is “reasonably necessary”, consider:
2. The nature of the situation, including the seriousness of the offence,
3. The information known to the police about the suspect or the crime, and
4. The extent to which the detention was reasonably responsive or tailored to
these circumstances, including its geographic and temporal scope.
 Then balance the seriousness of the risk to public or individual safety with the liberty
interests of members of the public to determine whether the nature of the detention is
no more intrusive than is reasonably necessary.
3. If DETENTION, What Charter
are the rights of the 9. Everyone has the right not to be arbitrarily detained or imprisoned.
detainee? 10. Everyone has the right on arrest or detention
(a) to be informed promptly of the reasons therefore;
(b) to retain and instruct counsel without delay and to be informed of that right; and
(c) to have the validity of the detention determined by way of habeas corpus and to
be released if the detention is not lawful.

s. 7 includes the right to remain silent. It is a residual right included in the principles of
fundamental justice in s. 7. Related to the notion of voluntariness (confessions) and
principle against self incrimination
RIGHT NOT TO BE ARBITRARILY DETAINED (s. 9)
4. Did the detention violate  A detention w/o reasonable suspicious or reasonable necessity is arbitrary and in
s. 9? violation of s. 9
RIGHT TO COUNSEL (s. 10(b))
5. Did the detention violate  Police must advise detainee of right to counsel (informational component)
s. 10(b)? (ie. What (Manninnen)
obligations are placed on  Individuals have a right to counsel immediately upon being detained (Suberu)
the police by s. 10(b) (immediacy only subject to concerns for officer or public safety.)
 Police must give detainee reasonable opportunity to exercise right to retain and
instruct counsel (implementational component) (Manninnen), including access to
legal aid and duty counsel (informational component) (Brydges)
 Police must cease questioning until detainee given reasonable opportunity to exercise
right (Manninnen), or unequivocally waived the right to counsel (Brydges)
 S 10(b) does NOT require a lawyer to be present during interrogation (Sinclair)
 Police are only required to provide additional opportunities to consult counsel when
there are material changes in the circumstances (e.g. additional charges, didn’t
understand waiving right to counsel, new procedures) (Sinclair)
6. Did the detainee use  Must use “reasonable diligence”
reasonable conduct in
exercising their rights?
RIGHT TO SILENCE (s. 7)
7. Did the detention violate 1. Police can carry on persuading and questioning a detainee EVEN after they
s. 7? What are the police have explicitly stated they want to stand on their right to silence AS LONG as

Police Powers | Getting to the Trial: The Criminal Investigation 92


entitled to do? the questioning doesn’t render them involuntary. (Singh)
2. Right to silence applies only after detention (Hebert)
3. The police can continue to question and persuade the accused once he has consulted
counsel (Hebert). They are not obligated to provide access to counsel again unless
there were material changes in the circumstances (Sinclair)
4. The right does not affect voluntary statements made to fellow cell-mates (Hebert)
Use of undercover agents to observe the accused in cells is okay as long as police don’t
engage in active elicitation designed to undermine the accused’s choice not to speak to
police. (Hebert)
Was the statement rendered COMMON LAW CONFESSION RULE (Oickle)
involuntary? If a statement of the accused is made to person of authority (i.e. police), the Crown must
prove beyond a reasonable doubt that the statement was voluntary
 Consider the totality of the circumstances
Confessions can be rendered involuntary because of:
1) Threats or promises that induce the confession (Ibrahim, Boudreau)
2) An atmosphere of oppression that compels a suspect to speak in order to bring
ordeal to end (Hobbins) (deprivation of food, etc)
3) Where suspect lacked “operating mind” (Ward v. the Queen; Horvath v. the Queen;
R. v. Clarkson)
If the common law confessions rule is breached, it results in the automatic exclusion
of the statement
7. Was the detention YES  NO Charter breach
lawful? NO  Charter breach (either s. 7, 9 or 10)
8. If a Charter breach was Apply Grant exclusion of evidence test.
found, should the evidence
be excluded under s. 24(2)?

R. V. GRANT, 2009 SCC 32


RATIO: The test for detention is whether a reasonable person, placed in the position of the accused,
would conclude that their right to choose how to act has been removed by the police, given their
conduct.

When performing an analysis under s.24(2) there are three factors which must be analyzed:

 the seriousness of the Charter-infringing state conduct;


 the impact on the Charter-protected interests of the accused; and
 society's interest in an adjudication of the case on its merits.

FACTS: Three police officers were on patrol for the purposes of monitoring an area near schools with
a history of student assaults, robberies, and drug offences. Grant, a young black man, was walking
down a sidewalk when he came to the attention of two plainclothes officers. As the two officers
drove past, Grant stared at them, while at the same time fidgeting with his coat and pants in a way
that aroused their suspicions. A uniformed officer initiated an exchange with the accused, while
standing on the sidewalk directly in his intended path. He asked him what was going on, and
requested his name and address. At one point, Grant, behaving nervously, adjusted his jacket, which
prompted the officer to ask him to keep his hands in front of him. After a brief period observing the
exchange from their car, the plainclothes officers approached the pair on the sidewalk, identified
themselves to Grant as police officers by flashing their badges and obstructed the way forward. The
uniformed officer then asked the accused whether he had anything he should not have, to which he

93 Criminal Law and Procedure | FLSC/NCA


answered that he had "a small bag of weed" and a firearm. At this point, the officers arrested and
searched the accused, seizing the marijuana and a loaded revolver. They advised him of his right to
counsel and took him to the police station.

At trial, Grant alleged violations of his rights under ss. 8, 9 and 10(b) of the Charter. The trial judge
found no Charter breach and admitted the firearm. The Court of Appeal concluded that a detention
had crystallized during the conversation with the officer before the accused made his incriminating
statements and that the detention was arbitrary and in breach of s.9 of the Charter, however, it held
that the gun should be admitted into evidence under s.24(2).

ISSUE: What constitutes detention? Were Grant’s rights under ss. 8, 9 and 10(b) of the Charter
violated? If so, is the admission of the gun permissible under s.24(2)?

DECISION: Appeal allowed on the trafficking charge, acquittal entered. Appeal dismissed on all other
counts.

REASONS: McLachlin and Charron, writing for the majority, found that existing jurisprudence on
detention was unclear and thus decided to lay out a new framework. They hold that the correct test
for detention is whether a reasonable person, placed in the position of the accused, would conclude
that their right to choose how to act has been removed by the police, given their conduct.

Applying this test to the case at bar, they hold that Grant was detained within the meaning of ss.9
and 10(b) of the Charter before being asked the questions that led him to disclose his possession of
the firearm. Asking him to keep his hands in front of him likely was not sufficient in and of itself, but
a holistic look, including the two plainclothes officers taking up tactical positions, would lead a
reasonable person to conclude they were being detained. They also note the inherent power
imbalance in the situation given Grant's age and inexperience.

They also lay out a test for the admission of evidence obtained under a Charter breach, per s.24(2).
There are three factors which must be analyzed:

1. the seriousness of the Charter-infringing state conduct - requires an assessment of whether


the admission of the evidence would bring the administration of justice into disrepute, and
focuses on the severity of the state conduct that led to the Charter breach (which includes
an analysis of whether the breach was deliberate or willful, and whether the officers were
acting in good faith);
2. the impact on the Charter-protected interests of the accused - focuses on how the accused
person was affected by the state conduct (which includes an analysis of the intrusiveness
into the person's privacy, the direct impact on the right not to be forced to self-incriminate,
and the effect on the person's human dignity); and
3. society's interest in an adjudication of the case on its merits - focuses on how reliable the
evidence is in light of the nature of the Charter breach.

Applying this test to the case at bar, they find that Grant's Charter rights were breached, but the
mistake made by the officers was an understandable one. Under the circumstances, there is no
evidence that admitting the gun would bring the administration of justice into disrepute. The impact
of the infringement on Grant's rights under ss.9 and 10 were significant, as he incriminated himself
under questioning from the officers. The evidence itself is highly reliable, and gun crime is a serious
problem with significant societal repercussions, however Grant argued that the seriousness of the
crime made it more important his right's be respected. On balance, they find that the admission was
acceptable, although it was a close ruling.

Police Powers | Getting to the Trial: The Criminal Investigation 94


Binnie, concurring in part, agrees with a holistic approach but does not like the focus this definition
of detention puts on the perception of psychological pressure, even if filtered through the
"reasonable person" standard

R. V. SUBERU, 2009 SCC 33


RATIO: Individuals must be informed of their rights upon detention (before or concurrently during
arrest), specifically the right to counsel in this case, and must be informed so immediately.

FACTS: S was not read his rights upon first encounter with the police. S was arrested for credit card
fraud. S was immediately read his rights upon second encounter with police and subsequent arrest.

ISSUE: When must an individual be read their ‘rights’?

DECISION: Appeal dismissed in favour of the Crown.

REASONS: RE Detention:

In the present case, while S was momentarily “delayed” when the police asked to speak to him, he
was not subjected to physical or psychological restraint so as to ground a detention within the
meaning of the Charter. S did not testify and the evidence does not support his contention that his
freedom to choose whether or not to cooperate with the police was removed during the period of
time prior to his arrest. The trial judge’s findings on the facts, supported by the evidence, lead to the
view that a reasonable person in the circumstances would have concluded that the initial encounter
was preliminary investigative questioning falling short of detention. Thus, S’s s. 10(b) right to counsel
was not engaged during this period. It was only later, after the officer received additional information
indicating that S was probably involved in the commission of an offence and determined that he
could not let him leave, that the detention crystallized and S’s rights under s. 10 were engaged — a
moment which, on the facts of this case, coincided with his arrest. Upon arresting S, the police
officer promptly and properly informed him of his right to counsel and, therefore, there was no
violation of s. 10(b) of the Charter.

RE Reading of rights:

The police duty to inform an individual of his or her s. 10(b) Charter right to retain and instruct
counsel is triggered at the outset of an investigative detention."

as per "without delay" in s. 10(b) of Charter (interpretive application)

R. V. AUCOIN, 2012 SCC 66


RATIO: A detention will be lawful if it is reasonably necessary on an objective view of the totality of
the circumstances. A search will be reasonable if it is authorized by law, if the law itself is reasonable
and if the manner in which the search was carried out is reasonable.

FACTS: Accused stopped for minor motor vehicle regulatory offence. Police officer conducted
pat‑down search of accused as a prelude to detention in police vehicle. Police officer discovered
drugs in accused’s pockets as a consequence of the pat‑down search.

ISSUE: Was the detention of the accused unlawful? Was the pat-down search unreasonable? Would
admission of the evidence bring the administration of justice into disrepute?

DECISION: Appeal dismissed.

95 Criminal Law and Procedure | FLSC/NCA


REASONS: The officer’s actions, though carried out in good faith, were not reasonably necessary.
Because A’s detention in the back of the cruiser would have been unlawful, it cannot constitute the
requisite basis in law to authorize the warrantless pat‑down search. Nonetheless, the cocaine found
on A was admissible into evidence under s.24(2) of the Charter. There were unusual circumstances
that prompted the police officer’s conduct in this case and he acted in good faith.

R. V. COLE, 2012 SCC 53


RATIO: Computers that are reasonably used for personal purposes — whether found in the workplace
or the home — contain information that is meaningful, intimate, and touching on the user’s
biographical core. Canadians may therefore reasonably expect privacy in the information contained
on these computers, at least where personal use is permitted or reasonably expected. Ownership of
property is a relevant consideration, but is not determinative. Workplace policies are also not
determinative of a person’s reasonable expectation of privacy. Whatever the policies state, one must
consider the totality of the circumstances in order to determine whether privacy is a reasonable
expectation in the particular situation. While workplace policies and practices may diminish an
individual’s expectation of privacy in a work computer, these sorts of operational realities do not in
themselves remove the expectation entirely. A reasonable though diminished expectation of privacy
is nonetheless a reasonable expectation of privacy, protected by s. 8 of the Charter. Accordingly, it is
subject to state intrusion only under the authority of a reasonable law.

FACTS: Pornographic pictures of a child were found on an employer-issued work computer.

ISSUE: Whether the accused had a reasonable expectation of privacy in an employer-issued work
computer. Whether the warrantless search and seizure of laptop computer and disc containing
Internet files breached the accused’s rights under s.8 of the Charter. If so, whether evidence ought to
be excluded pursuant to s.24(2) of the Charter.

DECISION: Appeal allowed, new trial ordered.

REASONS: The police in this case infringed the accused’s rights under s.8 of the Charter. The
accused’s personal use of his work-issued laptop generated information that is meaningful, intimate,
and organically connected to his biographical core. Pulling in the other direction are the ownership of
the laptop by the school board, the workplace policies and practices, and the technology in place at
the school. These considerations diminished the accused’s privacy interest in his laptop, at least in
comparison to a personal computer, but they did not eliminate it entirely. On balance, the totality of
the circumstances support the objective reasonableness of the accused’s subjective expectation of
privacy. While the principal had a statutory duty to maintain a safe school environment, and, by
necessary implication, a reasonable power to seize and search a school-board issued laptop, the
lawful authority of the accused’s employer to seize and search the laptop did not furnish the police
with the same power.

GETTING TO THE TRIAL: TAKING CONTROL OVER THE


ACCUSED
SECURING JURISDICTION OVER THE ACCUSED AND INTERIM RELEASE
The police have specified powers to arrest individuals. So too do non-police officers. The common theme in
the relevant legal provisions is that arrest – taking physical control over the subject - is to be used as a last
Securing Jurisdiction over the Accused and Interim Release | Getting to the Trial: Taking Control 96
Over the Accused
resort when other measures available for ensuring the good conduct and attendance before the criminal
justice process are not practical or desirable.

These less intrusive modes of securing attendance include the appearance notice, the promise to appear,
and the summons. Where an individual is arrested, he or she must be released or given a bail hearing where
it will be decided whether the individual should be released absolutely, subjected to conditions of release, or
held in custody pending the trial.

For further information on the above, refer to: Coughlan, pages 53-54 (gaining jurisdiction over the accused),
185-214 (the arrest, s.10(b)), 165-178 (compelling appearance without arrest), 178-184 (the bail hearing).

ARREST POWERS
General Rule When a police officer and an individual meet, unless there is an arrest, the
individual has the option NOT to interact with the police
How was the person 1. Laying of information
arrested? (ie. How does  Results in summons to court to answer to a charge or warrant for arrest.
an accused become  Police must show court reasonable and probable grounds that person
drawn into system?) committed the crime in order for a warrant to be issued (Storrey).
Otherwise, for less serious offences, summons is issued.

2. Arrest without cause


 Power to make a warrantless arrest turns on the following:
1) Who is making the arrest
a. Citizen’s Arrest (s. 494 CCC)
- May arrest w/o warrant a) person committing an indictable offence b)
person who on reasonable grounds has committed a criminal offence and is
escaping police c)person found committing a criminal offence on or in
relation to your property
b. Police (s. 495 of CCC)
- a) has committed an indictable offence or on reasonable grounds is
about to; b) person found committing a criminal offence
2) What kind of offence it is
a. Indictable
b. Summary conviction
c. Hybrid (Crown has choice on proceeding by either indictable or
sc
What is needed for a With warrant
lawful arrest? Must show judge reasonable and probable grounds that person committed
offence.

Without warrant
1. Criminal Code requires that arresting officer must subjectively have
reasonable and probable grounds on which to arrest.
2. Grounds must also be justifiable from objective perspective. (i.e. Would
a reasonable person in the position of the officer conclude there were
reasonable and probable grounds for the arrest?)
Upon arrest, what are the Common Law Power to Search Incident to Arrest (Cloutier v Langlois)
police entitled to do?  This is a power, not a duty
 Must be for a valid objective in pursuit of the ends of criminal justice:
o Safety of police

97 Criminal Law and Procedure | FLSC/NCA


o Securing evidence
o Preventing escape
 Must not be done in abusive manner
EXCEPTIONS to 1. Bodily samples (Stillman)
search incident to arrest 2. Strip Searches (Golden)
 Police must show reasonable and probable grounds to justify the strip
search, and additional grounds for justifying strip search done in the field,
as opposed to the police station.
What rights do 10. Everyone has the right on arrest or detention
individuals have upon (a) to be informed promptly of the reasons therefore;
arrest? (b) to retain and instruct counsel without delay and to be informed of that
right; and
(c) to have the validity of the detention determined by way of habeas
corpus and to be released if the detention is not lawful.

R. V. HALL, [2002] 3 S.C.R. 309


RATIO:

FACTS: David Scott Hall was charged with the murder of a woman in a high profile case. He applied
for bail pending trial. The judge denied the application--not for reasons of ensuring appearance in
court or protecting the public--but in order "to maintain confidence in the administration of justice".
Paragraph 515(10)(c) of the Criminal Code allows the denial of bail for this reason.

Hall appealed the decision on the basis that section 515(10)(c) violated the right "not to be denied
reasonable bail without just cause" under section section 11(e)of the Charter.

ISSUE: Whether part of s. 515(10)(c) of Criminal Code authorizing denial of bail in order “to maintain
confidence in the administration of justice” infringes the right “not to be denied reasonable bail
without just cause”.

DECISION: Appeal dismissed.

REASONS: Because the impugned phrase confers an open‑ended judicial discretion to refuse bail, it
is inconsistent with both s. 11(e) of the Canadian Charter of Rights and Freedoms, which guarantees
a right “not to be denied reasonable bail without just cause”, and the presumption of innocence.

GETTING READY FOR TRIAL


DISCLOSURE
A key right of the accused, and an important obligation on the Crown, is full disclosure of the fruits of the
investigation (all information gathered by or made known to the police during the investigation) to the
accused. All of the fruits of the investigation are to be disclosed save what is clearly irrelevant or privileged.
The law of privilege is covered by the law of evidence but the most relevant privileges should be flagged here.
Disclosure is to be made before the accused is called upon to elect his mode of trial for s.536 indictable
offences.

The accused may also seek to secure relevant “third party records” – relevant documents that are not the
fruits of the investigation that are under the control of persons other than prosecution and police. Where third

Disclosure | Getting Ready for Trial 98


party records are sought, complex applications must be brought, which differ depending on whether the
charge is a sexual offence prosecution or some other offence.

If issues arise as to whether proper disclosure has been made, the assigned trial judge should ordinarily
resolve them. As a practical matter, this requires early assignment of a trial judge who can address these
matters.

For further information on the above, refer to: Criminal Code, sections 278.1-278.91; Coughlan, pages 216-
250.

R. V. STINCHCOMBE, [1991] 3 S.C.R. 326


RATIO: Crown counsel have an obligation to disclose information obtained through investigation to
the defence. Failure to disclose is a breach of accused’s right to full answer and defence under s.7.

FACTS: William Stinchcombe was a lawyer who was charged with theft and fraud. One of the Crown's
witnesses was a former secretary of Stinchcombe's who had given evidence at the preliminary
inquiry that supported the defence's position. Later a statement was taken from her by an RCMP
officer, however, at trial the defence was denied access to the contents of the statement. When the
Crown decided not to use the statement the defence made a request for it to the judge who refused
to provide it. The accused was eventually convicted.

ISSUE: What is the Crown’s duty with regard to disclosure?

DECISION: Appeal allowed, new trial ordered. Crown counsel have a duty to disclose.

REASONS: Sopinka – Fruits of investigation are not the Crown’s property to secure a conviction, but
the property of the public to be used to ensure that justice is done. The right to make a full answer
and defence is a PFJ so there is a duty to disclose under s. 7 and arguably under s. 10(d). Includes
the disclosure of all relevant, non-privileged information, including evidence that Crown wants to rely
on and evidence that it doesn’t. Exception: Evidence that is privileged (e.g. informer privilege,
solicitor-client privilege).

R. V. O’CONNOR, [1995] 4 S.C.R. 411


RATIO: Crown must disclose to the accused all medical and therapeutic records in its possession
without regard to the privacy and equality interests of complainants in sexual assault trials.

First, the applicant must establish, without seeing them, that the records are likely to be relevant to
the case. Second, the judge must review the records and decide whether to disclose them based on
the balancing the right to make full answer and defence, and the right to privacy.

FACTS: Accused was charged with sexual offences. Defence counsel obtaining pre ‑ trial order
requiring Crown to disclose complainants' entire medical, counselling and school records ‑‑ Trial
judge ordering stay of proceedings owing to non‑disclosure and late disclosure by Crown ‑‑ Court of
Appeal allowing Crown's appeal and ordering new trial

ISSUE: The appeal raised the following issues:

1. When non-disclosure by the Crown justifies an order that the proceedings be stayed, and
2. The appropriate procedure to be followed when an accused seeks production of documents
such as medical or therapeutic records that are in the hands of third parties.

DECISION: Appeal dismissed.

99 Criminal Law and Procedure | FLSC/NCA


REASONS: Stay of Proceedings: La Forest, L’Heureux-Dube, Gonthier and McLachlin JJ - While the
Crown's conduct in this case was shoddy and inappropriate, the non‑disclosure cannot be said to
have violated the accused's right to full answer and defence.

Production of Records in the Possession of the Crown: Cory, Iacobucci JJ. - The principles set out in
the Stinchcombe decision, affirmed in Egger, pertaining to the Crown's duty to disclose must apply to
therapeutic records in the Crown's possession, as found by Lamer C.J. and Sopinka J.

Production of Records in the Possession of Third Parties: Lamer C.J., Sopinka J. - When the defence
seeks information in the hands of a third party (as compared to the state), the onus should be on the
accused to satisfy a judge that the information is likely to be relevant. Upon the production of the
records to the court, the judge should weigh the following factors:

1. the extent to which the record is necessary for the accused to make full answer and
defence;
2. the probative value of the record;
3. the nature and extent of the reasonable expectation of privacy vested in the record;
4. whether production of the record would be premised upon any discriminatory belief or bias;
and
5. the potential prejudice to the complainant's dignity, privacy or security of the person that
would be occasioned by production of the record.

The effect on the integrity of the trial process of producing, or failing to produce, the record,
having in mind the need to maintain consideration in the outcome, is more appropriately dealt
with at the admissibility stage and not in deciding whether the information should be produced.
As for society's interest in the reporting of sexual crimes, there are other avenues available to
the judge to ensure that production does not frustrate the societal interests that may be
implicated by the production of the records to the defence. In applying these factors, it is also
appropriate to bear in mind that production of third party records is always available to the
Crown provided it can obtain a search warrant.

R. V. MCNEIL 2009 SCC 3


RATIO: Records relating to findings of serious misconduct by police officers involved in the
investigation against the accused properly fall within the scope of the first party disclosure package
due to the Crown from police, where the police misconduct is either related to the investigation, or
the finding of misconduct could reasonably impact on the case against the accused.

FACTS: Accused was charged with drug-related offences. The Crown’s main police witness was
involved in drug-related misconduct which led to disciplinary proceedings and criminal charges. The
accused sought the production of the arresting officer’s police disciplinary records and criminal
investigation files

ISSUE: These were the issues on appeal:

 Whether common law production regime for third party records applicable only in cases
where these records attract reasonable expectation of privacy.
 Whether criminal investigation files relating to third party accused attract reasonable
expectation of privacy.
 Whether police records relating to findings of misconduct by officer involved in investigation
against accused fall within scope of first party disclosure package from police to Crown.

Disclosure | Getting Ready for Trial 100


In addition to clarifying these three uncertainties, this case provides an appropriate context within
which to reiterate the respective obligations of the police and the Crown to disclose the fruits of the
investigation under R. v. Stinchcombe, [1991] 3 S.C.R. 326, and to consider the extent to which
relevant police disciplinary records and third party criminal investigation files should form part of this
“first party” disclosure package.

DECISION: Appeal allowed, with the order in the court below set aside. The application having
become moot, the Court made no further order.

REASONS: The Crown’s obligation to disclose all relevant information in its possession to an accused
is well established at common law under the Stinchcombe regime. Under Stinchcombe, the Crown’s
first party disclosure obligation extends only to material relating to the accused’s case in the
possession or control of the prosecuting Crown. A question then arises as to whether the “Crown” for
disclosure purposes encompasses other state authorities. While the roles of the Crown and the
police are separate and distinct, the police have a duty to participate in the disclosure process. The
necessary corollary to the Crown’s disclosure duty under Stinchombe is the obligation of police to
disclose to the Crown all material pertaining to its investigation of the accused. For the purposes of
fulfilling this corollary obligation, the investigating police force, although distinct and independent
from the Crown at law, is not a third party. Rather, it acts on the same first party footing as the
Crown. Records relating to findings of serious misconduct by police officers involved in the
investigation against the accused properly fall within the scope of the first party disclosure package
due to the Crown from police, where the police misconduct is either related to the investigation, or
the finding of misconduct could reasonably impact on the case against the accused.

OBITER: Stated briefly, the procedure to be followed on an O’Connor application for production of
third party records is:

1. The accused first obtains a subpoena duces tecum under ss. 698(1) and 700(1) of the
Criminal Code and serves it on the third party record holder. The subpoena compels the
person to whom it is directed to attend court with the targeted records or materials.
2. The accused also brings an application, supported by appropriate affidavit evidence,
showing that the records sought are likely to be relevant in his or her trial. Notice of the
application is given to the prosecuting Crown, the person who is the subject of the records
and any other person who may have a privacy interest in the records targeted for production.
3. The O’Connor application is brought before the judge seized with the trial, although it may
be heard before the trial commences. If production is unopposed, of course, the application
for production becomes moot and there is no need for a hearing.
4. If the record holder or some other interested person advances a well-founded claim that the
targeted documents are privileged, in all but the rarest cases where the accused’s
innocence is at stake, the existence of privilege will effectively bar the accused’s application
for production of the targeted documents, regardless of their relevance. Issues of privilege
are therefore best resolved at the outset of the O’Connor process.
5. Where privilege is not in question, the judge determines whether production should be
compelled in accordance with the two-stage test established in O’Connor. At the first stage,
if satisfied that the record is likely relevant to the proceeding against the accused, the judge
may order production of the record for the court’s inspection. At the next stage, with the
records in hand, the judge determines whether, and to what extent, production should be
ordered to the accused.

PRELIMINARY INQUIRIES
101 Criminal Law and Procedure | FLSC/NCA
As indicated, at the preliminary inquiry, the judge must determine whether the Crown has presented a prima
facie case. If so, the accused is committed to stand trial and the prosecutor will be called upon to draft an
indictment, which will replace the original information as the new charging document. If the Crown does not
establish a prima facie case, the accused is discharged and the prosecution on the charge that has been laid
ends – in effect, the accused who was “charged” is “discharged.” A discharge at a preliminary inquiry is not,
however, an acquittal. The prosecution can re-lay the charge and try again, but will not do so unless important
new evidence is uncovered. The Attorney General also has the authority to lay a direct indictment, which gives
jurisdiction to a court to try the accused. The direct indictment can be used to re-institute a prosecution after
a preliminary inquiry discharge, or to bypass a preliminary inquiry altogether by indicting the accused directly
to trial.

For further information on the above, refer to: Coughlan, pages 253-277.

R. V. ARCURI [2001] 2 S.C.R 828


RATIO: If evidence is direct, the preliminary inquiry justice need not weigh it. If evidence is
circumstantial, the preliminary inquiry justice must engage in a limited weighing to assess whether a
reasonable jury, properly instructed, could return a finding of guilt.

FACTS: Crown’s case against accused was entirely circumstantial (body found in trunk of his car).
Accused is implicated because of DNA evidence found in bag of cloths left on the side of the road
near murder scene (i.e. sweat on shirt collar & blood on shirt match the accused). In addition, the
shirt, pants & shoes were all the same size as the accused (found in bag). There were questions
about the accused’s opportunity to commit the crime.

ISSUE: Whether the preliminary inquiry judge was required to weigh the Crown’s evidence against
exculpatory evidence called by the accused.

DECISION: Appeal dismissed.

REASONS: McLachlin C.J. - If the Crown’s case is direct, it is true that the matter is always one for the
jury; as by definition there is no inferential gap between direct evidence and the fact to be proved,
there is no inference whose reasonableness the preliminary inquiry justice must assess. If, however,
the Crown relies on circumstantial evidence, then the preliminary inquiry justice must engage in the
limited weighing of the whole of the evidence (i.e. including the defence evidence) to assess whether
a reasonable jury properly instructed could return a finding of guilt 42.

THE JURY TRIAL


If a jury trial is to be held, a trial judge is assigned, and a jury is selected.

For further information on the above, refer to: Coughlan, pages 306-326.

R. V. WILLIAMS, [1998] 1 S.C.R. 1128


RATIO: The prosecution and the defence are entitled to challenge potential jurors for cause on the
ground of partiality. Candidates for jury duty are presumed to be indifferent or impartial and this
presumption must be displaced before they can be challenged and questioned.

42 Paragraph 34.

The Jury Trial | Getting Ready for Trial 102


FACTS: Victor Daniel Williams, an Aboriginal, pleaded not guilty in robbing a Victoria Pizza Parlour.
Williams claimed that someone else had committed the robbery. Williams challenged potential
jurors with allegations of racial bias under s.638 of the Criminal Code.

ISSUE: Whether prospective jurors can be questioned as to racial bias.

DECISION: Appeal allowed, new trial ordered.

REASONS: Judicial directions to act impartially cannot always be assumed to be effective in


countering racial prejudice. Where doubts are raised, the better policy is to err on the side of caution
and permit prejudice to be examined. A motion to challenge for cause therefore need not be
dismissed if there was “no concrete evidence” that any of the prospective jurors could not set aside
their biases. The expectation that jurors usually behave in accordance with their oaths does not
obviate the need to permit challenges for cause where it is established that the community suffers
from widespread prejudice against people of the accused’s race sufficient to create a realistic
potential for partiality.

R. V. FIND, 2001 SCC 32


RATIO: Establishing a realistic potential for juror partiality generally requires satisfying the court on
two matters:

1. that a widespread bias exists in the community; and


2. that some jurors may be incapable of setting aside this bias, despite trial safeguards, to
render an impartial decision.

FACTS: The accused was charged with 21 counts of sexual offences involving complainants ranging
between 6 and 12 years of age at the time of the alleged offences. Prior to jury selection, he applied
to challenge potential jurors for cause, arguing that the nature of the charges against him gave rise
to a realistic possibility that some jurors might be unable to try the case against him impartially and
solely on the evidence before them. The trial judge rejected the application. The accused was tried
and convicted on 17 of the 21 counts. The majority of the Court of Appeal dismissed the accused’s
appeal, upholding the trial judge’s ruling not to permit the accused to challenge prospective jurors
for cause.

ISSUE: Whether charges of sexual assault against children raise realistic possibility of juror partiality
entitling accused to challenge for cause.

DECISION: Appeal dismissed.

REASONS: The material presented by the accused fell short of grounding judicial notice 43 of
widespread bias in Canadian society against an accused in sexual assault trials.

R. V. YUMNU, 2012 SCC 73


RATIO: It is permissible for the Crown, with the assistance of the police, to do limited background
checks to identify potential jurors who, by virtue of their criminal conduct, are not eligible for jury
duty. Information received by the Crown that is relevant to the jury selection process must be turned
over to the defence. Defence counsel, must also make disclosure to both the court and Crown

43 The threshold for judicial notice is strict: a court may properly take judicial notice of facts that are either:
1. so notorious or generally accepted as not to be the subject of debate among reasonable persons; or
2. capable of immediate and accurate demonstration by resort to readily accessible sources of
indisputable accuracy.

103 Criminal Law and Procedure | FLSC/NCA


counsel where they know or have good reason to believe that a potential juror has engaged in
criminal conduct that renders him or her ineligible for jury duty or cannot serve on a particular case
due to matters of obvious partiality.

FACTS: Appellants were convicted of first degree murder and conspiracy to commit murder. Prior to
jury selection, the Crown requested that the police conduct criminal record checks of prospective
jurors and also provide comments on whether any prospective jurors were “disreputable persons”.
None of the information received in response by Crown was disclosed to defence.

ISSUE: Whether it was appropriate to seek such information. Whether there should have been
disclosure of same. Whether there is a reasonable possibility that such conduct affected trial
fairness or gave rise to an appearance of unfairness, such that a miscarriage of justice occurred.

DECISION: Appeal dismissed.

REASONS: Jury vetting by the Crown and police gives rise to a number of concerns:

1. First is the prospect of the Crown and police joining forces to obtain a jury favourable to their
cause.
2. Second is the fundamental precept of our justice system that “justice should not only be
done, but should manifestly and undoubtedly be seen to be done”.
3. Third is juror privacy.

There are, however, countervailing interests at play that warrant some limited checking and some
minimal intrusions into the private lives of potential jurors. Only those persons eligible to serve as
jurors should be permitted to participate in the process. Under provincial statutes and the Criminal
Code, a potential juror’s criminal antecedents, and in some provinces his or her pending charges,
may render that person ineligible for jury duty or result in his or her removal from the jury pool
following a successful challenge for cause. Self‑reporting is one way of screening potential jurors,
but it has proved to be less than satisfactory. Accordingly, absent legislation to the contrary, the
authorities should be permitted to do criminal record checks on potential jurors to determine
whether they are eligible to serve as jurors. In addition, in those provinces where the eligibility
criteria cover persons who have been charged with a criminal offence, this is also something the
authorities may properly check for. It is thus permissible for the Crown, with the assistance of the
police, to do limited background checks using multiple police databases to identify potential jurors
who, by virtue of their criminal conduct, are not eligible for jury duty. The imbalance resulting from
the defence’s inability to conduct such searches is overcome by the disclosure obligations placed on
the Crown. Information received by the Crown that is relevant to the jury selection process must be
turned over to the defence, thereby restoring the balance. In return, defence counsel, as officers of
the court, must make disclosure to both the court and Crown counsel where they know or have good
reason to believe that a potential juror has engaged in criminal conduct that renders him or her
ineligible for jury duty or cannot serve on a particular case due to matters of obvious partiality.

PRE-TRIAL MOTIONS
In either judge alone or jury trials, there will often be preliminary legal issues to be resolved before the trial
gets going. These will ordinarily be dealt with by the assigned trial judge. In a jury trial, it is often convenient
to assign the judge and to dispose of these matters before a jury is selected, or if the motions can be
resolved expeditiously, select the jury and require it to leave the courtroom until the motions are completed.

For further information on the above, refer to: Coughlan, pages 283-306.

Pre-Trial Motions | Getting Ready for Trial 104


SENTENCING
GENERAL PRINCIPLES OF SENTENCING
For the most part, the general principles of sentencing have been codified in the Criminal Code. Judges are
instructed to use alternatives to imprisonment that are reasonable in the circumstances. Mandatory
sentences can be struck down as unconstitutional if they are grossly disproportionate, but judges cannot
create constitutional exemptions from them.

PRINCIPLES OF SENTENCING: s. 718

 s. 718.1: overarching guide is proportionality: must match law to blameworthiness


 s. 718.2(e): all available sanctions other than imprisonment that are reasonable in the
circumstances should be considered for all offenders, with particular attention to the circumstances
of aboriginal offenders
 to deal with disproportionate representation of aboriginal Canadians in CJS

Section 718 set out the general purposes of sentencing44:

“The fundamental purpose of sentencing is to contribute, along with crime prevention initiatives, to
respect for the law, and maintenance of a just, peaceful, and safe society by imposing just sanctions
that have one of more of the following objectives:…

 to denounce unlawful conduct


 to deter the offender and other potential offenders (specific and general deterrence)
 to separate offenders from society, where necessary
 to assist in rehabilitating offenders
 to provide reparations for harm done to victims/the community
 to promote a sense of responsibility in offenders and acknowledgment of the harm done to
victims and to the community”

Deterrence: Theory of deterrence tends to push longer sentences, putting it at odds with theory of
rehabilitation

 Sentencing is not what deters people; it is the likelihood of being caught


 Crimes are not committed at a cost-benefit analysis
 Useful in two situations:
o Incredibly harsh punishment for minor crimes

44 This was supposed to bring transparency and consistency to the Code.

Problems with s.718:

 General critique: did not add to transparency or consistency


 No priority
 Difficult to encompass in one sentence
 Don’t actually name specific sentences
 Different objectives can exist in conflict

105 Criminal Law and Procedure | FLSC/NCA


o Risk of getting caught

Separation: associated with confinement

 Isolated with other criminals, not ideal for rehabilitation


 Not clear it produces the safer society it purports to
 Disparate impact on certain gender or racial groups

Rehabilitation:

 Prisons are not ideal places for this


 Offset by the severity of the crime -> low on the list of priorities
 Requires a lot of funding

Reparations:

 Rare
 How do you compensate for victims of violence and domination?

Responsibility:

 Creates a binary (individual responsibility) and simplifies criminal responsibility


 Odd, because the criminal justice system is centered around the maintenance of innocence
 The system must believe in itself absolutely, but we know as a fact of any system that sometimes we
get it wrong
 Wrongful conviction – you cannot become eligible for parole unless you are penitent
 Sometimes taking responsibility leads to lesser sentences; can have a coercive face
 Can take responsibility without being remorseful

Denunciation: Whatever else the CJS is doing, societal denunciation is always key

 Always sending a message about a set of values & priorities


 Expressive dimension of criminal law

For further information on the above, refer to: Criminal Code, sections 718, 718.01, 718.1, 718.2, 718.3,
719; Roach, pages 475-486.

R. V. NASOGALUAK [2010] 1 S.C.R. 206. NO. 645

RATIO: Sentencing judges cannot mandate a sentence outside of mandatory minimums/maximums


laid out in the Criminal Code except in "exceptional circumstances".

FACTS: The RCMP received a tip about an intoxicated driver which led to a high speed pursuit of
Nasogaluak. When his car finally came to a stop, the police had to remove him forcibly. He resisted.
One officer punched him twice in the head while wrestling him out of the car. Once Nasogaluak was
out of the car, he continued to resist. The officer yelled at him to stop resisting and gave him a third
hard punch in the head. Nasogaluak was pinned face down on the pavement with the officer
straddling his back and another officer kneeling on his thigh. He refused to offer up his hands to be
handcuffed, so a second officer punched him twice in the back, breaking his ribs, which ultimately
led to a punctured lung.

45 See paragraphs 39-49; 63-64.

General Principles of Sentencing | Sentencing 106


At the police detachment following the arrest, Nasogaluak provided breath samples that placed him
over the legal blood alcohol limit. The officers did not report the force they had used during the arrest
and provided little to no information about the incident. Nasogaluak had no obvious signs of injury
and did not expressly request medical assistance and no attempts were made to ensure that he
received medical attention. He, however, twice told an officer that he was hurt. As well, he was
observed crying, was heard to say that he could not breathe, and was observed leaning over and
moaning. He was released the following morning and checked himself into a hospital. He was found
to have suffered broken ribs and a collapsed lung that required emergency surgery.

PROCEDURAL HISTORY: At trial, Nasogaluak entered a guilty plea to charges of impaired driving and
flight from police. At sentencing, the trial judge held that the police had used excessive force in
arresting the accused and breached his rights under s.7 of the Charter. As a remedy under s.24(1) of
the Charter, he reduced the accused's sentence and ordered a 12-month conditional discharge on
each count, served concurrently, with a one-year driving prohibition. The Court of Appeal held that a
sentencing judge has no discretion to reduce a sentence below a statutorily mandated minimum
sentence and ordered the minimum fine for a first offence mandated by s.255(1) of the Criminal
Code.

ISSUE: Is the imposition of a reduced sentence for the commission of a criminal offence an available
and appropriate remedy for the breach of an offender's Charter rights?

DECISION: Appeal dismissed.

REASONS: LeBel, writing for the court, held the principle of proportionality is central to the
sentencing process set out in the Criminal Code and requires that a sentence must speak out
against the offence but may not exceed what is just and appropriate given the moral
blameworthiness of the offender and the gravity of the offence. The determination of a fit sentence
is, subject to some specific statutory rules, an individualized process that requires the judge to weigh
the objectives of sentencing in a manner that best reflects the circumstances of the case. No one
sentencing objective trumps the others and the relative importance of any mitigating or aggravating
factors will push a sentence up or down the scale of appropriate sentences for similar offences.
However, the sentencing judges' discretion to craft a sentence which is tailored to the nature of the
offence and the circumstances of the offender is not without limits. His discretion is limited by case
law and by statutes through the general sentencing principles and objectives enshrined in the Code.
While a sentencing judge can order a sentence outside the general range set by case law as long as
it is in accordance with the principles and objectives of sentencing, they cannot override a clear
statement of legislative intent and reduce a sentence below a statutory mandated minimum, unless
that minimum is found to be unconstitutional.

Although LeBel found that in some exceptional cases a sentence reduction outside statutory limits
may be possible under s.24(1) of the Charter, this did not apply to the case at bar and the sentence
of the Court of Appeal was upheld.

R V. C.A.M., [1996] 1 S.C.R. 500


RATIO: The system of sentencing is not simply a method of imposing penalties or costs upon an
offender to prevent harmful conduct, but rather the system is to impose sentences which "positively
instills the basic set of values shared by all Canadians as expressed by the Criminal Code."

FACTS: In 1992, the respondent, C.A.M., pleaded guilty to numerous counts of sexual assault, incest,
assault with a weapon, in addition to other lesser offences, arising from a largely uncontested

107 Criminal Law and Procedure | FLSC/NCA


pattern of sexual, physical and emotional abuse inflicted upon his children over a number of years.
None of the offences committed by the respondent carried a penalty of life imprisonment.

PROCEDURAL HISTORY: The trial judge, remarking that the offences of the respondent were “as
egregious as any offences that I have ever had the occasion to deal with”, sentenced him to a
cumulative sentence of 25 years, with individual sentences running both consecutively and
concurrently. The British Columbia Court of Appeal, however, reduced the sentence of the respondent
to 18 years and 8 months. Following a line of jurisprudence it had developed in recent years, the
Court of Appeal concluded that where life imprisonment is not available as a penalty, the “principle
of totality” requires trial judges to limit fixed-term cumulative sentences under the Criminal Code of
Canada, R.S.C., 1985, c. C-46, to a term of imprisonment of 20 years, absent special circumstances.

ISSUE: Did the Court of Appeal err in law in holding that there is such a qualified ceiling on fixed-term
sentences under the Criminal Code?

DECISION: Appeal allowed, and the sentence of 25 years' imprisonment restored.

REASONS: There is no evidence in either the Code or the Corrections Act that Parliament intended to
constrain a trial judge's traditionally broad sentencing discretion through the imposition of a
qualified legal ceiling on numerical sentences pegged at 20 years' imprisonment.

R. V. GLADUE [1999] 1 S.C.R. 688


RATIO: Restorative justice must be given particular consideration when dealing with aboriginal
offenders; however it is not the only thing to be considered in these cases, as the traditional punitive
measures must also be weighed with the facts of the case.

Section 718.2(e) of the Code applies to aboriginals in general, not just those who live in aboriginal
communities.

When sentencing an aboriginal offender, the court must consider:

 the unique systemic or background factors which have played a part in bringing the offender
before the court; and
 the types of sentencing procedures and sanctions which may be appropriate in the
circumstances for the offender because of his or her particular aboriginal heritage or
connection.

FACTS: Gladue was accused of second-degree murder after she killed her fiancée with a knife, and
convicted of manslaughter. The two had been arguing about infidelity and insulting one another
when the incident occurred. Both of the parties were drunk at the time. Both parties were
aboriginals. The trial judge sentenced her to three years’ imprisonment. She appealed on sentence
unsuccessfully to the Court of Appeal and continued her appeal to the Supreme Court.

ISSUE: Should s.718.2(e) be understood as being remedial in nature? Should a different framework
be used when sentencing Aboriginal peoples?

DECISION: Appeal dismissed.

REASONS: Cory and Iacobucci delivered the judgment of the court. They first looked at how the trial
judge arrived at the three year sentence. Gladue had already started rehabilitation, and it seemed
that she did not need to be deterred from acting like this in the future. She had also already
apologized to the victim's family, so providing reparations for the family and community were not a

General Principles of Sentencing | Sentencing 108


large issue. She also acknowledged her wrong, so responsibility had already been shown. This is the
main reasoning why the sentence was only for three years, when it could have been up to life
imprisonment.

Gladue claimed that s.718.2(e) of the Code had not been considered in deciding the sentence. This
was the first time that this new provision of the Code was interpreted by the courts. Gladue did not
come from an aboriginal community; however she still claims that this provision applies to her.
Although the Court accepted the Court of Appeal's decision, they had more to say about the specific
provision. They determined that restorative justice is very important, and that this applies to Gladue
even though she does not live on a reservation, because the bigger problem is the disproportionate
number of aboriginals in jail. However, they also say that restorative justice isn’t the only thing that
needs to be considered when sentencing aboriginals, and that some crimes are serious enough to
deserve traditional punishment. They say that allowing a new trial solely on the basis of her
aboriginal status would not be in the public interest.

R. V. FERGUSON 2008 SCC 6


RATIO:
When considering whether a mandatory minimum violates the Charter, courts should consider wheth
er the sentence is grossly disproportionate to the extent that Canadians would find the punishment a
bhorrent and more than merely excessive (per Smith and Wiles). ss. 718 –
718.2 should be applied to consider if it is grossly disproportionate.

When sentencing, TJ is bound by facts found by the jury (a guilty verdict means TJ must assume self‐
defence failed). If there is ambiguity in a jury verdict, TJ should make his own assumptions per s. 724
, and not guess what the jury thought.

Aggravating factors (A’s duty of care to V) must be proved BRD, while mitigating factors are BoP. Cou
rts may not grant constitutional exceptions if they find a sentence grossly disproportionate and must
instead strike down the minimum sentence and replace it with general principles.

FACTS: During an altercation with a detainee held in a cell at an RCMP detachment, the accused, an
RCMP officer, shot and killed the detainee. The accused was charged with second‑degree murder
but was convicted by a jury of the lesser offence of manslaughter. Notwithstanding the mandatory
minimum sentence of four years imposed by s. 236(a) of the Criminal Code for manslaughter with a
firearm, the trial judge imposed a conditional sentence of two years less a day. He granted the
accused a constitutional exemption from the four‑year sentence because, on the circumstances of
this case, he found that the minimum mandatory sentence constituted cruel and unusual
punishment in violation of s. 12 of the Canadian Charter of Rights and Freedoms. The majority of
the Court of Appeal overturned that sentence and held that the mandatory minimum must be
imposed.

ISSUE: Whether the minimum sentence constitutes cruel and unusual punishment in the
circumstances of this case. If so, whether the trial judge was entitled to grant constitutional
exemption from the four‑year minimum and impose a lesser sentence.

DECISION: Appeal dismissed.

REASONS: The appropriateness of the minimum sentence of four years that Parliament has
prescribed for the offence of manslaughter committed with the use of a firearm depends on what
the jury concluded about the accused’s conduct.

109 Criminal Law and Procedure | FLSC/NCA


R. V. MORRISEY, 2000 SCC 39
RATIO: Under section 12 of the Canadian Charter of Rights and Freedoms, there can be exemptions
for mandatory prison sentences where the sentence is unreasonable or has an effect upon the
accused that may be considered harsh 46 . Pre-trial custody can be counted against a minimum
sentence.

FACTS: Marty Morrisey, a 36 year old from Belmont, Nova Scotia, was drinking with two friends in a
cabin. Morrisey and his friend Adrian Teed sawed the barrel off a shotgun. Morrisey told Teed the gun
was for the purpose of committing a robbery when in fact he was intending to kill himself due to
recent relationship problems. Morrisey drove the third bud home, and when he returned to the cabin
Teed was sleeping in a bunk bed. Morrisey leapt onto the bunk bed while holding the loaded
shotgun. He subsequently fell off the bed, likely due to his intoxication, and the gun accidentally
discharged, fatally wounding Teed.

Morrisey was charged with criminal negligence causing death under section 220(a) of the Criminal
Code of Canada.

ISSUE: Whether minimum sentence constitutes cruel and unusual punishment. Whether pre‑trial
custody should be taken into account.

DECISION: The appeal was dismissed in all respects except one. The accused’s sentence was
adjusted to take pre‑trial custody into account.

REASONS: Both the courts below agreed, and the defence has conceded, that a four‑year minimum
sentence would not be cruel and unusual punishment for this offender. An analysis of the gravity of
the offence requires an understanding of both the character of the offender’s actions, and the
consequences of those actions. That the accused’s actions in this case had particularly grave
consequences for the victim is not challenged. With respect to the character of the actions, to
attract criminal liability under s. 220(a) one must demonstrate wanton and reckless disregard for life
and safety. When both aspects of the gravity of the offence factor are considered, it is clear that s.
220(a) involves those who have committed a particularly grave offence. While there are mitigating
factors, these do not offset the aggravating factors in this case, nor do they displace the gravity of
the offence. With respect to the actual effect of the punishment on the offender, although a four‑
year term in a federal penitentiary is unquestionably a serious sentence, there are no special
punitive measures created to punish these offenders, and they would be eligible for parole after 16
months unless the trial judge directs otherwise. Moreover, pre‑trial custody can be counted against

46 Section 12 of the Charter provides a broad protection to Canadians against punishment which is so
excessive as to outrage our society’s sense of decency. The court must be satisfied that the punishment
imposed is grossly disproportionate for the offender, such that Canadians would find the punishment
abhorrent or intolerable. In assessing whether a sentence is grossly disproportionate, the court must first
consider the gravity of the offence, the personal characteristics of the offender and the particular
circumstances of the case in order to determine what range of sentences would have been appropriate to
punish, rehabilitate or deter this particular offender or to protect the public from him or her. As well, a court
is to consider the actual effect of the punishment on the individual, the penological goals and sentencing
principles upon which the sentence is fashioned, the existence of valid alternatives to the punishment
imposed, and a comparison of punishments imposed for other crimes in the same jurisdiction. These
contextual factors must be first evaluated in light of the particular circumstances of the offender before the
court. If the sentence is grossly disproportionate for the individual offender, the court then proceeds to
analyse whether the infringement of s. 12 can be justified under s. 1 of the Charter. If it is not
disproportionate for the individual offender, then the court is still to consider the constitutionality of the
sentence with reasonable hypotheticals.

General Principles of Sentencing | Sentencing 110


a minimum sentence. While it may be ideal to craft a minimum sentencing regime for this crime
that would simultaneously pursue all of the traditional sentencing principles, this is not necessary for
s. 12 purposes. This legislation survives constitutional scrutiny even if the sentence pursues
sentencing principles of general deterrence, denunciation and retributive justice more than the
principles of rehabilitation and specific deterrence.

R. V. PHAM, 2013 SCC 15


RATIO: A sentencing judge may exercise his or her discretion to take collateral immigration
consequences into account, provided that the sentence ultimately imposed is proportionate to the
gravity of the offence and the degree of responsibility of the offender.

An appellate court has the authority to vary a sentence if the sentencing judge was not aware of the
collateral immigration consequences, or if counsel had failed to advise the judge on this issue.

FACTS: The accused was convicted of two drug-related offences, and sentenced to two years’
imprisonment. The sentencing judge was not made aware that such a sentence would result in a loss
of the right to appeal a removal order under the Immigration and Refugee Protection Act. The Court
of Appeal subsequently refused to vary the sentence to two years less a day.

ISSUE: What weight should be attributed to collateral consequences in sentencing? Whether


sentence can be varied by appellate court on basis that accused would face collateral consequences.

DECISION: Appeal allowed, sentence reduced to two years less a day.

PITHY: “The general rule continues to be that a sentence must be fit having regard to the particular
crime and the particular offender. In other words, a sentencing judge may exercise his or her
discretion to take collateral immigration consequences into account, provided that the sentence that
is ultimately imposed is proportionate to the gravity of the offence and the degree of responsibility of
the offender.”47

APPEALS AND REVIEW


APPEALS OF FINAL DECISIONS AND JUDICIAL REVIEW OF INTERIM DECISIONS
Final verdicts can be appealed. Interim decisions cannot be. Interim decisions can, however, be the subject of
judicial review applications where jurisdictional errors occur. Judicial review may be necessary, for example,
to challenge preliminary inquiry results, to seek or quash publication bans, or to suppress or access third
party records; in these cases if we wait until the end of the trial, the damage sought to be prevented may
have already occurred, hence the judicial review application. In the case of appeals, different grounds of
appeal and procedural routes apply, depending on whether an offence has been prosecuted summarily or
indictably.

For further information on the above, refer to: Coughlan, pages 392-423 (appeals), 277-279 (judicial review
re: preliminary inquiries).

47 Paragraph 14.

111 Criminal Law and Procedure | FLSC/NCA


APPENDIX
PROBLEM FLOWCHART

Problem Flowchart | Appendix 112


EXCLUSION OF EVIDENCE
CHARTER CLAIM IN THE CRIMINAL LAW
There is a breach of a Charter right

What caused the breach?
 
A Law State Action

Is the legally prescribed breach justified under s. The remedy is that the evidence obtained by the
1? state action might be excluded under s. 24(2) of
the Charter.
 
Yes No
 
Since it is a The offending law is inconsistent
justified breach, the with the constitution and so is of no
law is force or effect (s. 52)
constitutionally
sound. No remedy
is necessary

OAKES TEST
Government has the burden of proving, on a balance of probabilities, that a breach is a reasonable limit,
prescribed by law, and justified in a free and democratic society.
1. Was the breach “prescribed by law”?
2. Does the offending law pursue a pressing and substantial objective?
3. Is the law proportional?
a. Is the law rationally connected to the pressing and substantial objective?
b. Does the law minimally impair the right in question?
c. Do the salutary benefits of the law outweigh its deleterious effects?
EXCLUSION OF EVIDENCE (POST-GRANT)
Evidence is obtained in breach of a Charter right

Section 24(2)

KEY QUESTION: Would a reasonable person, informed of all the relevant circumstances and the values
underlying the Charter, conclude that the admission of the evidence would bring the administration of
justice into disrepute?

THREE LINES OF INQUIRY:
1. The seriousness of the Charter-infringing state conduct (Worried about message that j.s.
condones serious state misconduct). Consider: inadvertent or minor? Wilful or reckless disregard
for Charter rights? Good faith by police? Part of a pattern of abusce?
2. The impact of the breach on the Charter-protected interests of the accused (Worried about
message that individual rights count for littler) Consider: interests engaged by infringed right;
degree to which violation impacted interests (e.g. high expectation of privacy?)
3. Society’s interest in the adjudication of the case on its merits (Worried about impact of failing
to admit the evidence on the truth-seeking function of the criminal trial) Consider: is the illegally

113 Criminal Law and Procedure | FLSC/NCA


obtained evidence reliable? How important is the evidence to the Crown’s case? Seriousness of
the offence (cuts both ways)

Given the assessments under each of these three lines of inquiry, would the admission of evidence
obtained by Charter breach bring the admin of justice into disrepute?
 
Yes No

Evidence excluded pursuant to s. 24(2) The evidence is admitted
Types of Evidence (from Grant)
a) Statements Made By b) Bodily Evidence – 1 c) Non-Bodily Physical d) Derivative Evidence
Accused – Concern for is fact-specific; 2 – Evidence – 1 fact- – 1 fact specific and
proper police conduct, greater intrusion, more specific, depends on if more serious state
centrality of protected likely to exclude; 3 – conduct was egregious conduct, more likely
interests affected and usually favour or deliberate; 2 privacy undermines public conf;
questionable reliability admission bc evidence is the main interest 2 if evidence
favour exclusion generally reliable engaged, depends on discoverable, more in
REP 3 – usually reliable favour of admission; 3
so tends to weigh in evidence is usually
favour of admission reliable so in favour of
admission

SAMPLE LEGAL MEMORANDA


SHORT LEGAL MEMORANDUM
MEMORANDUM

To: Senior LP Partner


Date: January 24th, 2013
Re: Oaks by Oaks Ltd.: Sale of Goods Problem

1. INTRODUCTION

You have asked me to prepare memorandum analysing the circumstances regarding the sale of
goods between Mr. … and Mr. … The primary parties involved are our client, Mr. of … Ltd. and the other side
Mr. of “… Ltd.” The main issues of the legal analysis is whether Mr. ….. The problem falls under the main area
of … law.
For the purposes of memorandum, the research methodology was limited to an analysis of the authorities
you provided in conjunction with your initial request.

2. FACTS
Our client, Mr. …

3. ISSUES

4. SHORT CONCLUSION

5. DISCUSSON/ANALYSIS

Sample Legal Memoranda | Appendix 114


6. CONCLUSION

TABLE OF AUTHORITIES

JURISPRUDENCE

SECONDARY SOURCES

NOTES (Case law citations here)

1.

2.

3.

LONG LEGAL MEMORANDUM


MEMORANDUM

To: Senior LP Partner


Date: January 24th, 2013
Re: Oaks by Oaks Ltd.: Sale of Goods Problem

1. INTRODUCTION

2. FACTS

3. (A) ISSUES

(B) SUB-ISSUES

4. SHORT CONCLUSION

5. DISCUSSION/ANALYSIS

6. CONCLUSION

MODEL FACTUM
IN THE HIGH COURT OF THE DOMINION OF CANADA
(On Appeal from the Ontario court of Appeal)

BETWEEN:

ORGANIZATION OF ABORIGINAL WOMEN (OAW)

Appellant

-AND-

HER MAJESTY THE QUEEN IN THE RIGHT OF ONTARIO

Respondent

115 Criminal Law and Procedure | FLSC/NCA


_____________________________________________________________________

FACTUM OF THE APPELLANT

ORGANIZATION OF ABORIGINAL WOMEN


_______________________________________________________________________

Counsel for the Appellant

Mr. … Lawyer

PART I: STATEMENT OG FACTS

1. The appellant, the OAW…

JUDICIAL HISTORY

PART II: STATEMENT OF ISSUES

PART III: APPELLANT’S ARGUMENTS

ISSUE r:

PART VI: ORDER SOUGHT

The appellant respectfully submits that this appeal be allowed and that the questions in this appeal be
answered as follows:

(1) … submit issue no 1… ?


Yes

(2) Is the violation justified under section 1 of the Charter?


No

REMEDY

ALL OF WHICH RESPECTFULLY SUBMITTED.

Dated at this day 24 day of January, 2013

COUNSEL FOR THE APPELLANT

PART V: APPENDIX

TABLE OF AUTHORITIES

1. LEGISLATION

2. CASE LAW

3. SECONDARY MATERIALS

Sample Legal Memoranda | Appendix 116


______________________________________________________

117 Criminal Law and Procedure | FLSC/NCA

Вам также может понравиться